You are on page 1of 87

CivRev (Property) Assignment 2

Republic of the Philippines


SUPREME COURT
Manila

FIRST DIVISION

G.R. No. L-57288 April 30, 1984

LEONILA SARMINETO, petitioner,
vs.
HON. ENRIQUE A. AGANA, District Judge, Court of First Instance of Rizal, Seventh Judicial District, Branch XXVIII,
Pasay City, and SPOUSES ERNESTO VALENTINO and REBECCA LORENZO-VALENTINO, respondents.

Mercedes M. Respicio for petitioner.

Romulo R. Bobadilla for private respondents.

MELENCIO-HERRERA, J.:ñé+.£ªwph!1

This Petition for certiorari questions a March 29, 1979 Decision rendered by the then Court of First Instance of Pasay City. The
Decision was one made on memoranda, pursuant to the provisions of RA 6031, and it modified, on October 17, 1977, a
judgment of the then Municipal Court of Paranaque, Rizal, in an Ejectment suit instituted by herein petitioner Leonila
SARMIENTO against private respondents, the spouses ERNESTO Valentino and Rebecca Lorenzo. For the facts, therefore, we
have to look to the evidence presented by the parties at the original level.

It appears that while ERNESTO was still courting his wife, the latter's mother had told him the couple could build a
RESIDENTIAL HOUSE on  a lot of 145 sq. ms., being Lot D of a subdivision in Paranaque (the LAND, for short). In 1967,
ERNESTO did construct a RESIDENTIAL HOUSE on the LAND at a cost of P8,000.00 to P10,000.00. It was probably assumed
that the wife's mother was the owner of the LAND and that, eventually, it would somehow be transferred to the spouses.

It subsequently turned out that the LAND had been titled in the name of Mr. & Mrs. Jose C. Santo, Jr. who, on September
7 ,  1974, sold the same to petitioner SARMIENTO. The following January 6, 1975, SARMIENTO asked ERNESTO and wife to
vacate and, on April 21, 1975, filed an Ejectment suit against them. In the evidentiary hearings before the Municipal Court,
SARMIENTO submitted the deed of sale of the LAND in her favor, which showed the price to be P15,000.00. On the other hand,
ERNESTO testified that the then cost of the RESIDENTIAL HOUSE would be from P30,000.00 to P40,000.00. The figures were
not questioned by SARMIENTO.

The Municipal Court found that private respondents had built the RESIDENTIAL HOUSE in good faith, and, disregarding the
testimony of ERNESTO, that it had a value of P20,000.00. It then ordered ERNESTO and wife to vacate the LAND after
SARMIENTO has paid them the mentioned sum of P20,000.00.

The Ejectment suit was elevated to the Court of First Instance of Pasay where, after the submission of memoranda, said Court
rendered a modifying Decision under Article 448 of the Civil Code. SARMIENTO was required, within 60 days, to exercise the
option to reimburse ERNESTO and wife the sum of 40,000.00 as the value of the RESIDENTIAL HOUSE, or the option to allow
them to purchase the LAND for P25,000.00. SARMIENTO did not exercise any of the two options within the indicated period,
and ERNESTO was then allowed to deposit the sum of P25,000.00 with the Court as the purchase price for the LAND. This is
the hub of the controversy. SARMIENTO then instituted the instant certiorari proceedings.

We agree that ERNESTO and wife were builders in good faith in view of the peculiar circumstances under which they had
constructed the RESIDENTIAL HOUSE. As far as they knew, the LAND was owned by ERNESTO's mother-in-law who, having
stated they could build on the property, could reasonably be expected to later on give them the LAND.

In regards to builders in good faith, Article 448 of the Code provides:têñ.£îhqwâ£

ART. 448. The owner of the land on which anything has been built, sown or planted in good faith,

shall have the right

to appropriate as his own the works, sowing or planting, after payment of the indemnity provided for in articles
546 and 548, or

to oblige the one who built or planted to pay the price of the land, and the one who sowed, the proper rent.

However, the builder or planter cannot be obliged to buy the land if its value is considerably more than that of
the building or trees. In such case, he shall pay reasonable rent, if the owner of the land does not choose to
appropriate the building or trees after proper indemnity. The parties shall agree upon the terms of the lease and
in case of disagreement, the court shall fix the terms thereof. (Paragraphing supplied)

The value of the LAND, purchased for P15,000.00 on September 7, 1974, could not have been very much more than that
amount during the following January when ERNESTO and wife were asked to vacate. However, ERNESTO and wife have not
questioned the P25,000.00 valuation determined by the Court of First Instance.
CivRev (Property) Assignment 2
In regards to the valuation of the RESIDENTIAL HOUSE, the only evidence presented was the testimony of ERNESTO that its
worth at the time of the trial should be from P30,000.00 to P40,000.00. The Municipal Court chose to assess its value at
P20,000.00, or below the minimum testified by ERNESTO, while the Court of First Instance chose the maximum of P40,000.00.
In the latter case, it cannot be said that the Court of First Instance had abused its discretion.

The challenged decision of respondent Court, based on valuations of P25,000.00 for the LAND and P40,000.00 for the
RESIDENTIAL HOUSE, cannot be viewed as not supported by the evidence. The provision for the exercise by petitioner
SARMIENTO of either the option to indemnify private respondents in the amount of P40,000.00, or the option to allow private
respondents to purchase the LAND at P25,000.00, in our opinion, was a correct decision.têñ.£îhqwâ£

The owner of the building erected in good faith on a land owned by another, is entitled to retain the possession
of the land until he is paid the value of his building, under article 453 (now Article 546). The owner, of the land.
upon, the other hand, has the option, under article 361 (now Article 448), either to pay for the building or to sell
his land to the owner of the building. But he cannot, as respondents here did, refuse both to pay for the
building and to sell the land and compel the owner of the building to remove it from the land where it is erected.
He is entitled to such remotion only when, after having chosen to sell his land, the other party fails to pay for
the same. (Emphasis ours)

We hold, therefore, that the order of Judge Natividad compelling defendants-petitioners to remove their
buildings from the land belonging to plaintiffs-respondents only because the latter chose neither to pay for such
buildings nor to sell the land, is null and void, for it amends substantially the judgment sought to be executed
and is, furthermore, offensive to articles 361 (now Article 448) and 453 (now Article 546) of the Civil Code.
(Ignacio vs. Hilario, 76 Phil. 605, 608 [1946]).

WHEREFORE, the Petition for Certiorari is hereby ordered dismissed, without pronouncement as to costs.

SO ORDERED.
CivRev (Property) Assignment 2
Republic of the Philippines
SUPREME COURT
Manila

FIRST DIVISION

G.R. No. L-49219 April 15, 1988

SPOUSES CONCEPCION FERNANDEZ DEL CAMPO and ESTANISLAO DEL CANTO, plaintiffs-appellees,


vs.
BERNARDA FERNANDEZ ABESIA, defendant-appellant.

Geronimo Creer, Jr. for plaintiffs-appellees.

Benedicto G. Cobarde for defendant, defendant-appellant

GANCAYCO, J.:

In this appeal from the decision of the Court of First Instance (CFI) of Cebu, certified to this Court by the Court of Appeals on
account of the question of law involved, the sole issue is the applicability of the provisions of Article 448 of the Civil Code relating
to a builder in good faith when the property involved is owned in common.

This case involves a parcel of land, Lot No. 1161 of the Cadastral Survey of Cebu, with an area of only about 45 square meters,
situated at the corner of F. Flores and Cavan Streets, Cebu City covered by TCT No. 61850. An action for partition was filed by
plaintiffs in the CFI of Cebu. Plaintiffs and defendants are co-owners pro indiviso of this lot in the proportion of and 1/3 share
each, respectively. The trial court appointed a commissioner in accordance with the agreement of the parties. ,the Id
commissioner conducted a survey, prepared a sketch plan and submitted a report to the trial court on May 29, 1976,
recommending that the property be divided into two lots: Lot 1161-A with an area of 30 square meters for plaintiffs and Lot No.
1161-B with an area of 15 square meters for the defendants. The houses of plaintiffs and defendants were surveyed and shown
on the sketch plan. The house of defendants occupied the portion with an area of 5 square meters of Lot 1161-A of plaintiffs.
The parties manifested their conformity to the report and asked the trial court to finally settle and adjudicate who among the
parties should take possession of the 5 square meters of the land in question.

In solving the issue the trial court held as follows:

The Court believed that the plaintiffs cannot be obliged to pay for the value of the portion of the defendants'
house which has encroached an area of five (5) sq. meters of the land alloted to them. The defendants cannot
also be obliged to pay for the price of the said five (5) square meters. The rights of a builder in good faith under
Article 448 of the New Civil Code does (sic) not apply to a case where one co-owner has built, planted or sown
on the land owned in common. "Manresa agreeing with Sanchez Roman, says that as a general rule this article
is not applicable because the matter should be governed more by the provisions on co-ownership than on
accession. Planiol and Ripert are also of the opinion that this article is not applicable to a co-owner who
constructs, plants or sows on the community property, even if the land where the construction, planting or
sowing is made is a third person under the circumstances, and the situation is governed by the rules of co-
ownership. Our Court of Appeals has held that this article cannot be invoked by one co-owner against another
who builds, plants or sows upon their land, since the latter does not do so on land not belonging to him. (C.A.),
O.G. Supp., Aug. 30, 194, p. 126). In the light of the foregoing authorities and considering that the defendants
have expressed their conformity to the partition that was made by the commissioner as shown in the sketch
plan attached to the commissioner's report, said defendants have no other alternative except to remove and
demolish part of their house that has encroached an area of five (5) sq. meters of the land allotted to the
plaintiffs.

WHEREFORE, judgment is hereby rendered assigning Lot 1161-A with an area of thirty (30) sq. meters to the
plaintiffs spouses Concepcion Fernandez Abesia, Lourdes Fernandez Rodil, Genaro Fernandez and Dominga
A. Fernandez, in the respective metes and bounds as shown in the subdivision sketch plan attached to the
Commissioner's Report dated may 29, 1976 prepared by the Commissioner, Geodetic Engineer Espiritu
Bunagan. Further, the defendants are hereby ordered at their expense to remove and demolish part of their
house which has encroached an area of five (5) square meters from Lot 1161-A of the plaintiffs; within sixty
(60) days from date hereof and to deliver the possession of the same to the plaintiffs. For the Commissioner's
fee of P400.00, the defendants are ordered to pay, jointly and severally, the sum of P133.33 and the balance
thereof to be paid by the plaintiffs. The costs of suit shall be paid by the plaintiffs and the defendants in the
proportion of two-thirds (2/3) and one-third (1/3) shares respectively. A certified copy of this judgment shall be
recorded in the office of the Register of Deeds of the City of Cebu and the expense of such recording shall be
taxed as a part of the costs of the action.

Hence, this appeal interposed by the defendants with the following assignments of errors:

THE TRIAL COURT ERRED IN NOT APPLYING THE RIGHTS OF A BUILDER IN GOOD FAITH UNDER
ART. 448 OF THE NEW CIVIL CODE TO DEFENDANTS-APPELLANTS WITH RESPECT TO THAT PART OF
THEIR HOUSE OCCUPYING A PROTION OF THE LOT ASSIGNED TO PLAINTIFFS-APPELLEES.
CivRev (Property) Assignment 2
II

THE TRIAL COURT ERRED IN ORDERING DEFENDANTS-APPELLANTS TO REMOVE AND DEMOLISH AT


THEIR EXPENSE, THAT PART OF THEIR HOUSE WHICH HAS ENCROACHED ON AN AREA OF FIVE
SQUARE METERS OF LOT 1161-A OF PLAINTIFFS-APPELLEES.

Article 448 of the New Civil Code provides as follows:

Art. 448. The owner of the land on which anything has been built, sown, or planted in good faith, shall have the
right to appropriate as his own the works, sowing or planting, after payment of the indemnity provided for in
articles 546 and 548, or to oblige the one who built or planted to pay the price of the land, and the one who
sowed, the proper rent. However, the builder or planter cannot be obliged to buy the land if its value is
considerably more than that of the building or trees. In such case, he shall pay reasonable rent, if the owner of
the land does not choose to appropriate the building or trees after proper indemnity. The parties shall agree
upon the terms of the lease and in case of disagreement, the court shall fix the terms thereof.

The court a quo  correctly held that Article 448 of the Civil Code cannot apply where a co-owner builds, plants or sows on the
land owned in common for then he did not build, plant or sow upon land that exclusively belongs to another but of which he is a
co-owner. The co-owner is not a third person under the circumstances, and the situation is governed by the rules of co-
ownership. 1

However, when, as in this case, the co-ownership is terminated by the partition and it appears that the house of defendants
overlaps or occupies a portion of 5 square meters of the land pertaining to plaintiffs which the defendants obviously built in good
faith, then the provisions of Article 448 of the new Civil Code should apply. Manresa and Navarro Amandi agree that the said
provision of the Civil Code may apply even when there was co-ownership if good faith has been established. 2

Applying the aforesaid provision of the Civil Code, the plaintiffs have the right to appropriate said portion of the house of
defendants upon payment of indemnity to defendants as provided for in Article 546 of the Civil Code. Otherwise, the plaintiffs
may oblige the defendants to pay the price of the land occupied by their house. However, if the price asked for is considerably
much more than the value of the portion of the house of defendants built thereon, then the latter cannot be obliged to buy the
land. The defendants shall then pay the reasonable rent to the plaintiff upon such terms and conditions that they may agree. In
case of disagreement, the trial court shall fix the terms thereof. Of course, defendants may demolish or remove the said portion
of their house, at their own expense, if they so decide.

WHEREFORE, the decision appealed from is hereby MODIFIED by ordering plaintiff to indemnify defendants for the value of the
Id portion of the house of defendants in accordance with Article 546 of the Civil Code, if plaintiffs elect to appropriate the same.
Otherwise, the defendants shall pay the value of the 5 square meters of land occupied by their house at such price as may be
agreed upon with plaintiffs and if its value exceeds the portion of the house that defendants built thereon, the defendants may
choose not to buy the land but defendants must pay a reasonable rental for the use of the portion of the land of plaintiffs As may
be agreed upon between the parties. In case of disagreement, the rate of rental shall be determined by the trial court. Otherwise,
defendants may remove or demolish at their own expense the said portion of their house. No costs.

SO ORDERED.
CivRev (Property) Assignment 2
Republic of the Philippines
SUPREME COURT
Manila

THIRD DIVISION

G.R. No. 72876             January 18, 1991

FLORENCIO IGNAO, petitioner,
vs.
HON. INTERMEDIATE APPELLATE COURT, JUAN IGNAO, substituted by his Legal Heirs, and ISIDRO
IGNAO, respondents.

Dolorfino and Dominguez Law Offices for petitioner.


Ambrosio Padilla, Mempin & Reyes Law Offices for private respondents.

FERNAN, C.J.:

In this petition for review by certiorari, petitioner seeks the reversal of the decision of the Intermediate Appellate Court (now
Court of Appeals) affirming in toto the decision of the Court of First Instance of Cavite, ordering petitioner Florencio Ignao to sell
to private respondents Juan and Isidro Ignao, that part of his property where private respondents had built a portion of their
houses.

The antecedent facts are as follows:

Petitioner Florencio Ignao and his uncles private respondents Juan Ignao and Isidro Ignao were co-owners of a parcel of land
with an area of 534 square meters situated in Barrio Tabon, Municipality of Kawit, Cavite. Pursuant to an action for partition filed
by petitioner docketed as Civil Case No. N-1681, the then Court of First Instance of Cavite in a decision dated February 6, 1975
directed the partition of the aforesaid land, alloting 133.5 square meters or 2/8 thereof to private respondents Juan and Isidro,
and giving the remaining portion with a total area of 266.5 square meters to petitioner Florencio. However, no actual partition
was ever effected.1

On July 17, 1978, petitioner instituted a complaint for recovery of possession of real property against private respondents Juan
and Isidro before the Court of First Instance of Cavite, docketed as Civil Case No. 2662. In his complaint petitioner alleged that
the area occupied by the two (2) houses built by private respondents exceeded the 133.5 square meters previously alloted to
them by the trial court in Civil Case No. N-1681.

Consequently, the lower court conducted an ocular inspection. It was found that the houses of Juan and Isidro actually
encroached upon a portion of the land belonging to Florencio. Upon agreement of the parties, the trial court ordered a licensed
geodetic engineer to conduct a survey to determine the exact area occupied by the houses of private respondents. The survey
subsequently disclosed that the house of Juan occupied 42 square meters while that of Isidro occupied 59 square meters of
Florencio's land or a total of 101 square meters.

In its decision, the trial court (thru Judge Luis L. Victor) ruled that although private respondents occupied a portion of Florencio's
property, they should be considered builders in good faith. The trial court took into account the decision of the Court of First
Instance of Cavite in the action for partition2 and quoted:

. . . . Hence, it is the well-considered opinion of the Court that although it turned out that the defendants had, before
partition, been in possession of more than what rightfully belongs to them, their possession of what is in excess of their
rightful share can at worst be possession in good faith which exempts them from being condemned to pay damages by
reason thereof.3

Furthermore, the trial court stated that pursuant to Article 448 of the Civil Code, the owner of the land (Florencio) should have
the choice to either appropriate that part of the house standing on his land after payment of indemnity or oblige the builders in
good faith (Juan and Isidro) to pay the price of the land. However, the trial court observed that based on the facts of the case, it
would be useless and unsuitable for Florencio to exercise the first option since this would render the entire houses of Juan and
Isidro worthless. The trial court then applied the ruling in the similar case of Grana vs. Court of Appeals,4 where the Supreme
Court had advanced a more "workable solution". Thus, it ordered Florencio to sell to Juan and Isidro those portions of his land
respectively occupied by the latter. The dispositive portion of said decision reads as follows:

WHEREFORE, judgment is hereby rendered in favor of the defendants and—

(a) Ordering the plaintiff Florencio Ignao to sell to the defendants Juan and Isidro Ignao that portion of his property with
an area of 101 square meters at P40.00 per square meter, on which part the defendants had built their houses; and

(b) Ordering the said plaintiff to execute the necessary deed of conveyance to the defendants in accordance with
paragraph (a) hereof.

Without pronouncement as to costs.5


CivRev (Property) Assignment 2
Petitioner Florencio Ignao appealed to the Intermediate Appellate Court. On August 27, 1985, the Appellate Court, Second Civil
Cases Division, promulgated a decision,6 affirming the decision of the trial court.

Hence the instant petition for review which attributes to the Appellate Court the following errors:

1. That the respondent Court has considered private respondents builders in good faith on the land on question, thus
applying Art. 448 of the Civil Code, although the land in question is still owned by the parties in co-ownership, hence,
the applicable provision is Art. 486 of the Civil Code, which was not applied.

2. That, granting for the sake of argument that Art. 448 . . . is applicable, the respondent Court has adjudged the
working solution suggested in Grana and Torralba vs. CA. (109 Phil. 260), which is just an opinion by way of passing,
and not the judgment rendered therein, which is in accordance with the said provision of the Civil Code, wherein the
owner of the land to buy (sic) the portion of the building within 30 days from the judgment or sell the land occupied by
the building.

3. That, granting that private respondents could buy the portion of the land occupied by their houses, the price fixed by
the court is unrealistic and pre-war price.7

The records of the case reveal that the disputed land with an area of 534 square meters was originally owned by Baltazar Ignao
who married twice. In his first marriage, he had four children, namely Justo (the father of petitioner Florencio), Leon and private
respondents Juan and Isidro. In his second marriage, Baltazar had also four children but the latter waived their rights over the
controverted land in favor of Justo. Thus, Justo owned 4/8 of the land which was waived by his half-brothers and sisters plus his
1/8 share or a total of 5/8. Thereafter, Justo acquired the 1/8 share of Leon for P500.00 which he later sold to his son Florencio
for the same amount. When Justo died, Florencio inherited the 5/8 share of his father Justo plus his 1/8 share of the land which
he bought or a total of 6/8 (representing 400.5 square meters). Private respondents, Juan and Isidro, on the other hand, had 1/8
share (66.75 square meters) each of the land or a total of 133.5 square meters.

Before the decision in the partition case was promulgated, Florencio sold 134 square meters of his share to a certain Victa for
P5,000.00 on January 27, 1975. When the decision was handed down on February 6,1975, the lower court alloted 2/8 of the
land to private respondents Juan and Isidro, or a total of 133.5 square meters.

It should be noted that prior to partition, all the co-owners hold the property in common dominion but at the same time each is an
owner of a share which is abstract and undetermined until partition is effected. As cited in Eusebio vs. Intermediate Appellate
Court,8 "an undivided estate is co-ownership by the heirs."

As co-owners, the parties may have unequal shares in the common property, quantitatively speaking. But in a qualitative sense,
each co-owner has the same right as any one of the other co-owners. Every co-owner is therefore the owner of the whole, and
over the whole he exercises the right of dominion, but he is at the same time the owner of a portion which is truly abstract,
because until division is effected such portion is not concretely determined. 9

Petitioner Florencio, in his first assignment of error, asseverates that the court a quo erred in applying Article 448 of the Civil
Code, since this article contemplates a situation wherein the land belongs to one person and the thing built, sown or planted
belongs to another. In the instant case, the land in dispute used to be owned in common by the contending parties.

Article 448 provides:

Art. 448. The owner of the land on which anything has been built, sown or planted in good faith, shall have the right to
appropriate as his own the works, sowing or planting, after payment of the indemnity provided for in articles 546 and
548, or to oblige the one who built or planted to pay the price of the land, and the one who sowed, the proper rent.
However, the builder or planter cannot be obliged to buy the land if its value is considerably more than that of the
building or trees. In such case, he shall pay reasonable rent, if the owner of the land does not choose to appropriate the
building or trees after proper indemnity. The parties shall agree upon the terms of the lease and in case of
disagreement, the court shall fix the terms thereof.

Whether or not the provisions of Article 448 should apply to a builder in good faith on a property held in common has been
resolved in the affirmative in the case of Spouses del Campo vs. Abesia,10 wherein the Court ruled that:

The court a quo correctly held that Article 448 of the Civil Code cannot apply where a co-owner builds, plants or sows
on the land owned in common for then he did not build, plant or sow upon land that exclusively belongs to another but
of which he is a co-owner. The co-owner is not a third person under the circumstances, and the situation is governed by
the rules of co-ownership.

However, when, as in this case, the ownership is terminated by the partition and it appears that the home of defendants
overlaps or occupies a portion of 5 square meters of the land pertaining to plaintiffs which the defendants obviously built
in good faith, then the provisions of Article 448 of the new Civil Code should apply. Manresa and Navarro Amandi agree
that the said provision of the Civil Code may apply even when there is a co-ownership if good faith has been
established.11

In other words, when the co-ownership is terminated by a partition and it appears that the house of an erstwhile co-owner has
encroached upon a portion pertaining to another co-owner which was however made in good faith, then the provisions of Article
448 should apply to determine the respective rights of the parties.
CivRev (Property) Assignment 2
Petitioner's second assigned error is however well taken. Both the trial court and the Appellate Court erred when they
peremptorily adopted the "workable solution" in the case of Grana vs. Court of appeals,12 and ordered the owner of the land,
petitioner Florencio, to sell to private respondents, Juan and Isidro, the part of the land they intruded upon, thereby depriving
petitioner of his right to choose. Such ruling contravened the explicit provisions of Article 448 to the effect that "(t)he owner of the
land . . . shall have the right to appropriate . . .or to oblige the one who built . . . to pay the price of the land . . . ." The law is clear
and unambiguous when it confers the right of choice upon the landowner and not upon the builder and the courts.

Thus, in Quemuel vs. Olaes,13 the Court categorically ruled that the right to appropriate the works or improvements or to oblige
the builder to pay the price of the land belongs to the landowner.

As to the third assignment of error, the question on the price to be paid on the land need not be discussed as this would be
premature inasmuch as petitioner Florencio has yet to exercise his option as the owner of the land.

WHEREFORE, the decision appealed from is hereby MODIFIED as follows: Petitioner Florencio Ignao is directed within thirty
(30) days from entry of judgment to exercise his option to either appropriate as his own the portions of the houses of Juan and
Isidro Ignao occupying his land upon payment of indemnity in accordance with Articles 546 and 548 of the Civil Code, or sell to
private respondents the 101 square meters occupied by them at such price as may be agreed upon. Should the value of the
land exceed the value of the portions of the houses that private respondents have erected thereon, private respondents may
choose not to buy the land but they must pay reasonable rent for the use of the portion of petitioner's land as may be agreed
upon by the parties. In case of disagreement, the rate of rental and other terms of the lease shall be determined by the trial
court. Otherwise, private respondents may remove or demolish at their own expense the said portions of their houses
encroaching upon petitioner's land.14 No costs.

SO ORDERED.
CivRev (Property) Assignment 2
Republic of the Philippines
SUPREME COURT
Manila

THIRD DIVISION

G.R. No. 108894 February 10, 1997

TECNOGAS PHILIPPINES MANUFACTURING CORPORATION, petitioner,


vs.
COURT OF APPEALS (FORMER SPECIAL SEVENTEENTH DIVISION) and EDUARDO UY, respondents.

PANGANIBAN, J.:

The parties in this case are owners of adjoining lots in Parañaque, Metro Manila. It was discovered in a survey, that a portion of
a building of petitioner, which was presumably constructed by its predecessor-in-interest, encroached on a portion of the lot
owned by private respondent. What are the rights and obligations of the parties? Is petitioner considered a builder in bad faith
because, as held by respondent Court, he is "presumed to know the metes and bounds of his property as described in his
certificate of title"? Does petitioner succeed into the good faith or bad faith of his predecessor-in-interest which presumably
constructed the building?

These are the questions raised in the petition for review of the Decision 1 dated August 28, 1992, in CA-G.R. CV No. 28293 of
respondent Court2 where the disposition reads:3

WHEREFORE, premises considered, the Decision of the Regional Trial Court is hereby reversed and set aside
and another one entered —

1. Dismissing the complaint for lack of cause of action;

2. Ordering Tecnogas to pay the sum of P2,000.00 per month as reasonable rental from October 4, 1979 until
appellee vacates the land;

3. To remove the structures and surrounding walls on the encroached area;

4. Ordering appellee to pay the value of the land occupied by the two-storey building;

5. Ordering appellee to pay the sum of P20,000.00 for and as attorney's fees;

6. Costs against appellee.

Acting on the motions for reconsideration of both petitioner and private respondent, respondent Court ordered the deletion of
paragraph 4 of
the dispositive portion in an Amended Decision dated February 9, 1993, as follows: 4

WHEREFORE, premises considered, our decision of August 28, 1992 is hereby modified deleting paragraph 4
of the dispositive portion of our decision which reads:

4. Ordering appellee to pay the value of the land occupied by the two-storey building.

The motion for reconsideration of appellee is hereby DENIED for lack of merit.

The foregoing Amended Decision is also challenged in the instant petition.

The Facts

The facts are not disputed. Respondent Court merely reproduced the factual findings of the trial court, as follows: 5

That plaintiff (herein petitioner) which is a corporation duly organized and existing under and by virtue of
Philippine laws is the registered owner of a parcel of land situated in Barrio San Dionisio, Parañaque, Metro
Manila known as Lot 4331-A (should be 4531-A) of Lot 4531 of the Cadastral Survey of Parañaque, Metro
Manila, covered by Transfer Certificate of Title No. 409316 of the Registry of Deeds of the Province of Rizal;
that said land was purchased by plaintiff from Pariz Industries, Inc. in 1970, together with all the buildings and
improvements including the wall existing thereon; that the defendant (herein private respondent) is the
registered owner of a parcel of land known as Lot No. 4531-B of Lot 4531 of the Cadastral Survey of
Parañaque, LRC (GLRO) Rec. No. 19645 covered by Transfer Certificate of Title No. 279838, of the Registry of
Deeds for the Province of Rizal; that said land which adjoins plaintiff's land was purchased by defendant from a
CivRev (Property) Assignment 2
certain Enrile Antonio also in 1970; that in 1971, defendant purchased another lot also adjoining plaintiffs land
from a certain Miguel Rodriguez and the same was registered in defendant's name under Transfer Certificate
of Title No. 31390, of the Registry of Deeds for the Province of Rizal; that portions of the buildings and wall
bought by plaintiff together with the land from Pariz Industries are occupying a portion of defendant's adjoining
land; that upon learning of the encroachment or occupation by its buildings and wall of a portion of defendant's
land, plaintiff offered to buy from defendant that particular portion of defendant's land occupied by portions of its
buildings and wall with an area of 770 square meters, more or less, but defendant, however, refused the offer.
In 1973, the parties entered into a private agreement before a certain Col. Rosales in Malacañang, wherein
plaintiff agreed to demolish the wall at the back portion of its land thus giving to defendant possession of a
portion of his land previously enclosed by plaintiff's wall; that defendant later filed a complaint before the office
of Municipal Engineer of Parañaque, Metro Manila as well as before the Office of the Provincial Fiscal of Rizal
against plaintiff in connection with the encroachment or occupation by plaintiff's buildings and walls of a portion
of its land but said complaint did not prosper; that defendant dug or caused to be dug a canal along plaintiff's
wall, a portion of which collapsed in June, 1980, and led to the filing by plaintiff of the supplemental complaint in
the above-entitled case and a separate criminal complaint for malicious mischief against defendant and his wife
which ultimately resulted into the conviction in court of defendant's wife for the crime of malicious mischief; that
while trial of the case was in progress, plaintiff filed in Court a formal proposal for settlement of the case but
said proposal, however, was ignored by defendant.

After trial on the merits, the Regional Trial Court6 of Pasay City, Branch 117, in Civil Case No. PQ-7631-P, rendered a decision
dated December 4, 1989 in favor of petitioner who was the plaintiff therein. The dispositive portion
reads: 7

WHEREFORE, judgment is hereby rendered in favor of plaintiff and against defendant and ordering the latter to
sell to plaintiff that portion of land owned by him and occupied by portions of plaintiff's buildings and wall at the
price of P2,000.00 per square meter and to pay the former:

1. The sum of P44,000.00 to compensate for the losses in materials and properties incurred
by plaintiff through thievery as a result of the destruction of its wall;

2. The sum of P7,500.00 as and by way of attorney's fees; and

3. The costs of this suit.

Appeal was duly interposed with respondent Court, which as previously stated, reversed and set aside the decision of the
Regional Trial Court and rendered the assailed Decision and Amended Decision. Hence, this recourse under Rule 45 of the
Rules of Court.

The Issues

The petition raises the following issues:8

(A)

Whether or not the respondent Court of Appeals erred in holding the petitioner a builder in bad faith because it
is "presumed to know the metes and bounds of his property."

(B)

Whether or not the respondent Court of Appeals erred when it used the amicable settlement between the
petitioner and the private respondent, where both parties agreed to the demolition of the rear portion of the
fence, as estoppel amounting to recognition by petitioner of respondent's right over his property including the
portions of the land where the other structures and the building stand, which were not included in the
settlement.

(C)

Whether or not the respondent Court of Appeals erred in ordering the removal of the "structures and
surrounding walls on the encroached area" and in withdrawing its earlier ruling in its August 28, 1992 decision
for the petitioner "to pay for the value of the land occupied" by the building, only because the private
respondent has "manifested its choice to demolish" it despite the absence of compulsory sale where the builder
fails to pay for the land, and which "choice" private respondent deliberately deleted from its September 1, 1980
answer to the supplemental complaint in the Regional Trial Court.

In its Memorandum, petitioner poses the following issues:

A.

The time when to determine the good faith of the builder under Article 448 of the New Civil Code, is
reckoned during the period when it was actually being built; and in a case where no evidence was
presented nor introduced as to the good faith or bad faith of the builder at that time, as in this case, he must
be presumed to be a "builder in good faith," since "bad faith cannot be presumed."9
CivRev (Property) Assignment 2
B.

In a specific "boundary overlap situation" which involves a builder in good faith, as in this case, it is now well
settled that the lot owner, who builds on the adjacent lot is not charged with "constructive notice" of the
technical metes and bounds contained in their torrens titles to determine the exact and precise extent of his
boundary perimeter. 10

C.

The respondent court's citation of the twin cases of Tuason & Co. v. Lumanlan and Tuason &
Co. v. Macalindong is not the "judicial authority" for a boundary dispute situation between adjacent torrens titled
lot owners, as the facts of the present case do not fall within nor square with the involved principle of a
dissimilar case. 11

D.

Quite contrary to respondent Uy's reasoning, petitioner Tecnogas continues to be a builder in good faith, even
if it subsequently built/repaired the walls/other permanent structures thereon while the case a quo was pending
and even while respondent sent the petitioner many letters/filed cases thereon. 12

D.(E.)

The amicable settlement between the parties should be interpreted as a contract and enforced only in
accordance with its explicit terms, and not over and beyond that agreed upon; because the courts do not have
the power to create a contract nor expand its scope. 13

E.(F.)

As a general rule, although the landowner has the option to choose between: (1) "buying the building built in
good faith", or (2) "selling the portion of his land on which stands the building" under Article 448 of the Civil
Code; the  first option is not absolute, because an exception thereto, once it would be impractical for the
landowner to choose to exercise the first alternative, i.e. buy that portion of the house standing on his land, for
the whole building might be rendered useless. The workable solution is for him to select the second alternative,
namely, to sell to the builder that part of his land on which was constructed a portion of the house. 14

Private respondent, on the other hand, argues that the petition is "suffering from the following flaws: 15

1. It did not give the exact citations of cases decided by the Honorable Supreme Court that allegedly
contradicts the ruling of the Hon. Court of Appeals based on the doctrine laid down in Tuason
vs. Lumanlan case citing also Tuason vs. Macalindong case (Supra).

2. Assuming that the doctrine in the alleged Co Tao vs. Chico case is contradictory to the doctrine in Tuason
vs. Lumanlan and Tuason vs. Macalindong, the two cases being more current, the same should prevail.

Further, private respondent contends that the following "unmistakably" point to the bad faith of petitioner: (1) private
respondent's purchase of the two lots, "was ahead of the purchase by petitioner of the building and lot from Pariz Industries"; (2)
the declaration of the General Manager of Tecnogas that the sale between petitioner and Pariz Industries "was not registered"
because of some problems with China Banking Corporation; and (3) the Deed of Sale in favor of petitioner was registered in its
name only in "the month of May 1973." 16

The Court's Ru1ing

The petition should be granted.

Good Faith or Bad Faith

Respondent Court, citing the cases of J.M. Tuason & Co., Inc. vs. Vda. de Lumanlan  17 and J.M. Tuason &
Co., Inc. vs. Macalindong, 18 ruled that petitioner "cannot be considered in good faith" because as a land owner, it is "presumed
to know the metes and bounds of his own property, specially if the same are reflected in a properly issued certificate of title. One
who erroneously builds on the adjoining lot should be considered a builder in (b)ad (f)aith, there being presumptive knowledge of
the Torrens title, the area, and the extent of the boundaries." 19

We disagree with respondent Court. The two cases it relied upon do not support its main pronouncement that a registered owner
of land has presumptive knowledge of the metes and bounds of its own land, and is therefore in bad faith if he mistakenly builds
on an adjoining land. Aside from the fact that those cases had factual moorings radically different from those obtaining here,
there is nothing in those cases which would suggest, however remotely, that bad faith is imputable to a registered owner of land
when a part of his building encroaches upon a neighbor's land, simply because he is supposedly presumed to know the
boundaries of his land as described in his certificate of title. No such doctrinal statement could have been made in those cases
because such issue was not before the Supreme Court. Quite the contrary, we have rejected such a theory in Co Tao
vs. Chico, 20 where we held that unless one is versed in the science of surveying, "no one can determine the precise extent or
location of his property by merely examining his paper title."
CivRev (Property) Assignment 2
There is no question that when petitioner purchased the land from Pariz Industries, the buildings and other structures were
already in existence. The record is not clear as to who actually built those structures, but it may well be assumed that petitioner's
predecessor-in-interest, Pariz Industries, did so. Article 527 of the Civil Code presumes good faith, and since no proof exists to
show that the encroachment over a narrow, needle-shaped portion of private respondent's land was done in bad faith by the
builder of the encroaching structures, the latter should be presumed to have built them in good faith. 21 It is presumed that
possession continues to be enjoyed in the same character in which it was acquired, until the contrary is proved. 22 Good faith
consists in the belief of the builder that the land he is building on is his, and his ignorance of any defect or flaw in his
title. 23 Hence, such good faith, by law, passed on to Pariz's successor, petitioner in this case. Further, "(w)here one derives title
to property from another, the act, declaration, or omission of the latter, while holding the title, in relation to the property, is
evidence against the former." 24 And possession acquired in good faith does not lose this character except in case and from the
moment facts exist which show that the possessor is not unaware that he possesses the thing improperly or wrongfully. 25 The
good faith ceases from the moment defects in the title are made known to the possessor, by extraneous evidence or by suit for
recovery of the property by the true owner. 26

Recall that the encroachment in the present case was caused by a very slight deviation of the erected wall (as fence) which was
supposed to run in a straight line from point 9 to point 1 of petitioner's lot. It was an error which, in the context of the attendant
facts, was consistent with good faith. Consequently, the builder, if sued by the aggrieved landowner for recovery of possession,
could have invoked the provisions of Art. 448 of the Civil Code, which reads:

The owner of the land on which anything has been built, sown or planted in good faith, shall have the right to
appropriate as his own the works, sowing or planting, after payment of the indemnity provided for in articles 546
and 548, or to oblige the one who built or planted to pay the price of the land, and the one who sowed, the
proper rent. However, the builder or planter cannot be obliged to buy the land if its value is considerably more
than that of the building or trees. In such case, he shall pay reasonable rent, if the owner of the land does not
choose to appropriate the building or trees after proper indemnity. The parties shall agree upon the terms of the
lease and in case of disagreement, the court shall fix the terms thereof.

The obvious benefit to the builder under this article is that, instead of being outrightly ejected from the land, he can
compel the landowner to make a choice between the two options: (1) to appropriate the building by paying the
indemnity required by law, or (2) sell the land to the builder. The landowner cannot refuse to exercise either option and
compel instead the owner of the building to remove it from the land. 27

The question, however, is whether the same benefit can be invoked by petitioner who, as earlier stated, is not the builder of the
offending structures but possesses them as buyer.

We answer such question in the affirmative.

In the first place, there is no sufficient showing that petitioner was aware of the encroachment at the time it acquired the property
from Pariz Industries. We agree with the trial court that various factors in evidence adequately show petitioner's lack of
awareness thereof. In any case, contrary proof has not overthrown the presumption of good faith under Article 527 of the Civil
Code, as already stated, taken together with the disputable presumptions of the law on evidence. These presumptions state,
under Section 3 (a) of Rule 131 of the Rules of Court, that the person is innocent of a crime or wrong; and under Section 3 (ff) of
Rule 131, that the law has been obeyed. In fact, private respondent Eduardo Uy himself was unaware of such intrusion into his
property until after 1971 when he hired a surveyor, following his purchase of another adjoining lot, to survey all his newly
acquired lots. Upon being apprised of the encroachment, petitioner immediately offered to buy the area occupied by its building
— a species of conduct consistent with good faith.

In the second place, upon delivery of the property by Pariz Industries, as seller, to the petitioner, as buyer, the latter acquired
ownership of the property. Consequently and as earlier discussed, petitioner is deemed to have stepped into the shoes of the
seller in regard to all rights of ownership over the immovable sold, including the right to compel the private respondent to
exercise either of the two options provided under Article 448 of the Civil Code.

Estoppel

Respondent Court ruled that the amicable settlement entered into between petitioner and private respondent estops the former
from questioning the private respondent's "right" over the disputed property. It held that by undertaking to demolish the fence
under said settlement, petitioner recognized private respondent's right over the property, and "cannot later on compel" private
respondent "to sell to it the land since" private respondent "is under no obligation to sell." 28

We do not agree. Petitioner cannot be held in estoppel for entering into the amicable settlement, the pertinent portions of which
read: 29

That the parties hereto have agreed that the rear portion of the fence that separates the property of the
complainant and respondent shall be demolished up to the back of the building housing the machineries which
demolision (sic) shall be undertaken by the complainant at anytime.

That the fence which serve(s) as a wall housing the electroplating machineries shall not be demolished in the
mean time which portion shall be subject to negotiation by herein parties.

From the foregoing, it is clear that petitioner agreed only to the demolition of a portion of the wall separating the adjoining
properties of the parties — i.e. "up to the back of the building housing the machineries." But that portion of the fence which
served as the wall housing the electroplating machineries was not to be demolished. Rather, it was to "be subject to negotiation
by herein parties." The settlement may have recognized the ownership of private respondent but such admission cannot be
equated with bad faith. Petitioner was only trying to avoid a litigation, one reason for entering into an amicable settlement.
CivRev (Property) Assignment 2
As was ruled in Osmeña vs. Commission on Audit, 30

A compromise is a bilateral act or transaction that is expressly acknowledged as a juridical agreement by the
Civil Code and is therein dealt with in some detail. "A compromise," declares Article 2208 of said Code, "is a
contract whereby the parties, by making reciprocal concessions, avoid a litigation or put an end to one already
commenced."

xxx xxx xxx

The Civil Code not only defines and authorizes compromises, it in fact encourages them in civil actions. Art.
2029 states that "The Court shall endeavor to persuade the litigants in a civil case to agree upon some fair
compromise." . . .

In the context of the established facts, we hold that petitioner did not lose its rights under Article 448 of the Civil Code on the
basis merely of the fact that some years after acquiring the property in good faith, it learned about — and aptly recognized — the
right of private respondent to a portion of the land occupied by its building. The supervening awareness of the encroachment by
petitioner does not militate against its right to claim the status of a builder in good faith. In fact, a judicious reading of said Article
448 will readily show that the landowner's exercise of his option can only take place after the builder shall have come to know of
the intrusion — in short, when both parties shall have become aware of it. Only then will the occasion for exercising the option
arise, for it is only then that both parties will have been aware that a problem exists in regard to their property rights.

Options of Private Respondent

What then is the applicable provision in this case which private respondent may invoke as his remedy: Article 448 or Article
450 31 of the Civil Code?

In view of the good faith of both petitioner and private respondent, their rights and obligations are to be governed by Art. 448.
The essential fairness of this codal provision has been pointed out by Mme. Justice Ameurfina Melencio-Herrera, citing Manresa
and applicable precedents, in the case of Depra vs. Dumlao, 32 to wit:

Where the builder, planter or sower has acted in good faith, a conflict of rights arises between the owners, and
it becomes necessary to protect the owner of the improvements without causing injustice to the owner of the
land. In view of the impracticality of creating a state of forced co-ownership, the law has provided a just solution
by giving the owner of the land the option to acquire the improvements after payment of the proper indemnity,
or to oblige the builder or planter to pay for the land and the sower to pay the proper rent. It is the owner of the
land who is authorized to exercise the option, because his right is older, and because, by the principle of
accession, he is entitled to the ownership of the accessory thing. (3 Manresa 213; Bernardo vs. Bataclan, 37
Off. Gaz. 1382; Co Tao vs. Chan Chico, G.R. No. 49167, April 30, 1949; Article applied; see Cabral, et al. vs.
Ibanez [S.C.] 52 Off. Gaz. 217; Marfori vs. Velasco, [C.A.] 52 Off. Gaz. 2050).

The private respondent's insistence on the removal of the encroaching structures as the proper remedy, which respondent Court
sustained in its assailed Decisions, is thus legally flawed. This is not one of the remedies bestowed upon him by law. It would be
available only if and when he chooses to compel the petitioner to buy the land at a reasonable price but the latter fails to pay
such price. 33 This has not taken place. Hence, his options are limited to: (1) appropriating the encroaching portion of petitioner's
building after payment of proper indemnity, or (2) obliging the latter to buy the lot occupied by the structure. He cannot exercise
a remedy of his own liking.

Neither is petitioner's prayer that private respondent be ordered to sell the land 34 the proper remedy. While that was dubbed as
the "more workable solution" in Grana and Torralba vs. The Court of Appeals, et al., 35 it was not the relief granted in that case
as the landowners were directed to exercise "within 30 days from this decision their option to either buy the portion of the
petitioners' house on their land or sell to said petitioners the portion of their land on which it stands." 36 Moreover, in Grana and
Torralba, the area involved was only 87 square meters while this case involves 520 square meters 37. In line with the case
of Depra vs. Dumlao, 38 this case will have to be remanded to the trial court for further proceedings to fully implement the
mandate of Art. 448. It is a rule of procedure for the Supreme Court to strive to settle the entire controversy in a single
proceeding leaving no root or branch to bear the seeds of future
litigation. 39

Petitioner, however, must also pay the rent for the property occupied by its building as prescribed by respondent Court from
October 4, 1979, but only up to the date private respondent serves notice of its option upon petitioner and the trial court; that is,
if such option is for private respondent to appropriate the encroaching structure. In such event, petitioner would have a right of
retention which negates the obligation to pay rent. 40 The rent should however continue if the option chosen is compulsory sale,
but only up to the actual transfer of ownership.

The award of attorney's fees by respondent Court against petitioner is unwarranted since the action appears to have been filed
in good faith. Besides, there should be no penalty on the right to litigate. 41

WHEREFORE, premises considered, the petition is hereby GRANTED and the assailed Decision and the Amended Decision
are REVERSED and SET ASIDE. In accordance with the case of Depra vs. Dumlao, 42 this case is REMANDED to the Regional
Trial Court of Pasay City, Branch 117, for further proceedings consistent with Articles 448 and 546 43 of the Civil Code, as
follows:

The trial court shall determine:


CivRev (Property) Assignment 2
a) the present fair price of private respondent's 520 square-meter area of land;

b) the increase in value ("plus value") which the said area of 520 square meters may have acquired by reason
of the existence of the portion of the building on the area;

c) the fair market value of the encroaching portion of the building; and

d) whether the value of said area of land is considerably more than the fair market value of the portion of the
building thereon.

2. After said amounts shall have been determined by competent evidence, the regional trial court shall render judgment as
follows:

a) The private respondent shall be granted a period of fifteen (15) days within which to exercise his option
under the law (Article 448, Civil Code), whether to appropriate the portion of the building as his own by paying
to petitioner its fair market value, or to oblige petitioner to pay the price of said area. The amounts to be
respectively paid by petitioner and private respondent, in accordance with the option thus exercised by written
notice of the other party and to the court, shall be paid by the obligor within fifteen (15) days from such notice of
the option by tendering the amount to the trial court in favor of the party entitled to receive it;

b) If private respondent exercises the option to oblige petitioner to pay the price of the land but the latter rejects
such purchase because, as found by the trial court, the value of the land is considerably more than that of the
portion of the building, petitioner shall give written notice of such rejection to private respondent and to the trial
court within fifteen (15) days from notice of private respondent's option to sell the land. In that event, the parties
shall be given a period of fifteen (15) days from such notice of rejection within which to agree upon the terms of
the lease, and give the trial court formal written notice of the agreement and its provisos. If no agreement is
reached by the parties, the trial court, within fifteen (15) days from and after the termination of the said period
fixed for negotiation, shall then fix the terms of the lease provided that the monthly rental to be fixed by the
Court shall not be less than two thousand pesos (P2,000.00) per month, payable within the first five (5) days of
each calendar month. The period for the forced lease shall not be more than two (2) years, counted from the
finality of the judgment, considering the long period of time since 1970 that petitioner has occupied the subject
area. The rental thus fixed shall be increased by ten percent (10%) for the second year of the forced lease.
Petitioner shall not make any further constructions or improvements on the building. Upon expiration of the two-
year period, or upon default by petitioner in the payment of rentals for two (2) consecutive months, private
respondent shall be entitled to terminate the forced lease, to recover his land, and to have the portion of the
building removed by petitioner or at latter's expense. The rentals herein provided shall be tendered by
petitioner to the trial court for payment to private respondent, and such tender shall constitute evidence of
whether or not compliance was made within the period fixed by the said court.

c) In any event, petitioner shall pay private respondent an amount computed at two thousand pesos
(P2,000.00) per month as reasonable compensation for the occupancy of private respondent's land for the
period counted from October 4, 1979, up to the date private respondent serves notice of its option to
appropriate the encroaching structures, otherwise up to the actual transfer of ownership to petitioner or, in case
a forced lease has to be imposed, up to the commencement date of the forced lease referred to in the
preceding paragraph;

d) The periods to be fixed by the trial court in its decision shall be non-extendible, and upon failure of the party
obliged to tender to the trial court the amount due to the obligee, the party entitled to such payment shall be
entitled to an order of execution for the enforcement of payment of the amount due and for compliance with
such other acts as may be required by the prestation due the obligee.

No costs.

SO ORDERED.
CivRev (Property) Assignment 2
Republic of the Philippines
SUPREME COURT
Manila

THIRD DIVISION

G.R. No. 79688             February 1, 1996

PLEASANTVILLE DEVELOPMENT CORPORATION, petitioner,


vs.
COURT OF APPEALS, WILSON KEE, C.T. TORRES ENTERPRISES, INC. and ELDRED JARDINICO, respondents.

DECISION

PANGANIBAN, J.:

Is a lot buyer who constructs improvements on the wrong property erroneously delivered by the owner's agent, a builder in good
faith? This is the main issue resolved in this petition for review on certiorari to reverse the Decision1 of the Court of Appeals2 in
CA-G.R. No. 11040, promulgated on August 20, 1987.

By resolution dated November 13, 1995, the First Division of this Court resolved to transfer this case (along with several others)
to the Third Division. After due deliberation and consultation, the Court assigned the writing of this Decision to the
undersigned  ponente.

The Facts

The facts, as found by respondent Court, are as follows:

Edith Robillo purchased from petitioner a parcel of land designated as Lot 9, Phase II and located at Taculing Road,
Pleasantville Subdivision, Bacolod City. In 1975, respondent Eldred Jardinico bought the rights to the lot from Robillo. At that
time, Lot 9 was vacant.

Upon completing all payments, Jardinico secured from the Register of Deeds of Bacolod City on December 19, 1978 Transfer
Certificate of Title No. 106367 in his name. It was then that he discovered that improvements had been introduced on Lot 9 by
respondent Wilson Kee, who had taken possession thereof.

It appears that on March 26, 1974, Kee bought on installment Lot 8 of the same subdivision from C.T. Torres Enterprises, Inc.
(CTTEI), the exclusive real estate agent of petitioner. Under the Contract to Sell on Installment, Kee could possess the lot even
before the completion of all installment payments. On January 20, 1975, Kee paid CTTEI the relocation fee of P50.00 and
another P50.00 on January 27, 1975, for the preparation of the lot plan. These amounts were paid prior to Kee's taking actual
possession of Lot 8. After the preparation of the lot plan and a copy thereof given to Kee, CTTEI through its employee, Zenaida
Octaviano, accompanied Kee's wife, Donabelle Kee, to inspect Lot 8. Unfortunately, the parcel of land pointed by Octaviano was
Lot 9. Thereafter, Kee proceeded to construct his residence, a store, an auto repair shop and other improvements on the lot.

After discovering that Lot 9 was occupied by Kee, Jardinico confronted him. The parties tried to reach an amicable settlement,
but failed.

On January 30, 1981, Jardinico's lawyer wrote Kee, demanding that the latter remove all improvements and vacate Lot 9. When
Kee refused to vacate Lot 9, Jardinico filed with the Municipal Trial Court in Cities, Branch 3, Bacolod City (MTCC), a complaint
for ejectment with damages against Kee.

Kee, in turn, filed a third-party complaint against petitioner and CTTEI.

The MTCC held that the erroneous delivery of Lot 9 to Kee was attributable to CTTEI. It further ruled that petitioner and CTTEI
could not successfully invoke as a defense the failure of Kee to give notice of his intention to begin construction required under
paragraph 22 of the Contract to Sell on Installment and his having built a sari-sari store without the prior approval of petitioner
required under paragraph 26 of said contract, saying that the purpose of these requirements was merely to regulate the type of
improvements to be constructed on the Lot.3

However, the MTCC found that petitioner had already rescinded its contract with Kee over Lot 8 for the latter's failure to pay the
installments due, and that Kee had not contested the rescission. The rescission was effected in 1979, before the complaint was
instituted. The MTCC concluded that Kee no longer had any right over the lot subject of the contract between him and petitioner.
Consequently, Kee must pay reasonable rentals for the use of Lot 9, and, furthermore, he cannot claim reimbursement for the
improvements he introduced on said lot.

The MTCC thus disposed:

IN VIEW OF ALL THE FOREGOING, judgment is hereby rendered as follows:

1. Defendant Wilson Kee is ordered to vacate the premises of Lot 9, covered by TCT No. 106367 and to remove all
structures and improvements he introduced thereon;
CivRev (Property) Assignment 2
2. Defendant Wilson Kee is ordered to pay to the plaintiff rentals at the rate of P15.00 a day computed from the time this
suit was filed on March 12, 1981 until he actually vacates the premises. This amount shall bear interests (sic) at the rate
of 12 per cent (sic) per annum.

3. Third-Party Defendant C.T. Torres Enterprises, Inc. and Pleasantville Subdivision are ordered to pay the plaintiff
jointly and severally the sum of P3,000.00 as attorney's fees and P700.00 as cost and litigation expenses. 4

On appeal, the Regional Trial Court, Branch 48, Bacolod City (RTC) ruled that petitioner and CTTEI were not at fault or were not
negligent, there being no preponderant evidence to show that they directly participated in the delivery of Lot 9 to Kee 5 . It found
Kee a builder in bad faith. It further ruled that even assuming arguendo that Kee was acting in good faith, he was, nonetheless,
guilty of unlawfully usurping the possessory right of Jardinico over Lot 9 from the time he was served with notice to vacate said
lot, and thus was liable for rental.

The RTC thus disposed:

WHEREFORE, the decision appealed from is affirmed with respect to the order against the defendant to vacate the
premises of Lot No. 9 covered by Transfer Certificate of Title No. T-106367 of the land records of Bacolod City; the
removal of all structures and improvements introduced thereon at his expense and the payment to plaintiff (sic) the sum
of Fifteen (P15.00) Pesos a day as reasonable rental to be computed from January 30, 1981, the date of the demand,
and not from the date of the filing of the complaint, until he had vacated (sic) the premises, with interest thereon at 12%
per annum. This Court further renders judgment against the defendant to pay the plaintiff the sum of Three Thousand
(P3,000.00) Pesos as attorney's fees, plus costs of litigation.

The third-party complaint against Third-Party Defendants Pleasantville Development Corporation and C.T. Torres
Enterprises, Inc. is dismissed. The order against Third-Party Defendants to pay attorney's fees to plaintiff and costs of
litigation is reversed.6

Following the denial of his motion for reconsideration on October 20, 1986, Kee appealed directly to the Supreme Court, which
referred the matter to the Court of Appeals.

The appellate court ruled that Kee was a builder in good faith, as he was unaware of the "mix-up" when he began construction of
the improvements on Lot 8. It further ruled that the erroneous delivery was due to the negligence of CTTEI, and that such wrong
delivery was likewise imputable to its principal, petitioner herein. The appellate court also ruled that the award of rentals was
without basis.

Thus, the Court of Appeals disposed:

WHEREFORE, the petition is GRANTED, the appealed decision is REVERSED, and judgment is rendered as follows:

1. Wilson Kee is declared a builder in good faith with respect to the improvements he introduced on Lot 9, and is entitled
to the rights granted him under Articles 448, 546 and 548 of the New Civil Code.

2. Third-party defendants C.T. Torres Enterprises, Inc. and Pleasantville Development Corporation are solidarily liable
under the following circumstances:

A.       If Eldred Jardinico decides to appropriate the improvements and, thereafter, remove these structures,
the third-party defendants shall answer for all demolition expenses and the value of the improvements thus
destroyed or rendered useless;

b. If Jardinico prefers that Kee buy the land, the third-party defendants shall answer for the amount
representing the value of Lot 9 that Kee should pay to Jardinico.

3. Third-party defendants C.T. Torres Enterprises, Inc. and Pleasantville Development Corporation are ordered to
pay in solidum the amount of P3,000.00 to Jardinico as attorney's fees, as well as litigation expenses.

4. The award of rentals to Jardinico is dispensed with.

Furthermore, the case is REMANDED to the court of origin for the determination of the actual value of the
improvements and the property (Lot 9), as well as for further proceedings in conformity with Article 448 of the New Civil
Code.7

Petitioner then filed the instant petition against Kee, Jardinico and CTTEI.

The Issues

The petition submitted the following grounds to justify a review of the respondent Court's Decision, as follows:

1. The Court of Appeals has decided the case in a way probably not in accord with law or the the (sic) applicable
decisions of the Supreme Court on third-party complaints, by ordering third-party defendants to pay the demolition
expenses and/or price of the land;
CivRev (Property) Assignment 2
2. The Court of Appeals has so far departed from the accepted course of judicial proceedings, by granting to private
respondent-Kee the rights of a builder in good faith in excess of what the law provides, thus enriching private
respondent Kee at the expense of the petitioner;

3. In the light of the subsequent events or circumstances which changed the rights of the parties, it becomes imperative
to set aside or at least modify the judgment of the Court of Appeals to harmonize with justice and the facts;

4. Private respondent-Kee in accordance with the findings of facts of the lower court is clearly a builder in bad faith,
having violated several provisions of the contract to sell on installments;

5. The decision of the Court of Appeals, holding the principal, Pleasantville Development Corporation (liable) for the
acts made by the agent in excess of its authority is clearly in violation of the provision of the law;

6. The award of attorney's fees is clearly without basis and is equivalent to putting a premium in (sic) court litigation.

From these grounds, the issues could be re-stated as follows:

(1) Was Kee a builder in good faith?

(2) What is the liability, if any, of petitioner and its agent, C.T. Torres Enterprises, Inc.? and

(3) Is the award of attorney's fees proper?

The First Issue: Good Faith

Petitioner contends that the Court of Appeals erred in reversing the RTC's ruling that Kee was a builder in bad faith.

Petitioner fails to persuade this Court to abandon the findings and conclusions of the Court of Appeals that Kee was a builder in
good faith. We agree with the following observation of the Court of Appeals:

The roots of the controversy can be traced directly to the errors committed by CTTEI, when it pointed the wrong
property to Wilson Kee and his wife. It is highly improbable that a purchaser of a lot would knowingly and willingly build
his residence on a lot owned by another, deliberately exposing himself and his family to the risk of being ejected from
the land and losing all improvements thereon, not to mention the social humiliation that would follow.

Under the circumstances, Kee had acted in the manner of a prudent man in ascertaining the identity of his property. Lot
8 is covered by Transfer Certificate of Title No. T-69561, while Lot 9 is identified in Transfer Certificate of Title No. T-
106367. Hence, under the Torrens system of land registration, Kee is presumed to have knowledge of the metes and
bounds of the property with which he is dealing. . . .

xxx       xxx       xxx

But as Kee is a layman not versed in the technical description of his property, he had to find a way to ascertain that
what was described in TCT No. 69561 matched Lot 8. Thus, he went to the subdivision developer's agent and applied
and paid for the relocation of the lot, as well as for the production of a lot plan by CTTEI's geodetic engineer. Upon
Kee's receipt of the map, his wife went to the subdivision site accompanied by CTTEI's employee, Octaviano, who
authoritatively declared that the land she was pointing to was indeed Lot 8. Having full faith and confidence in the
reputation of CTTEI, and because of the company's positive identification of the property, Kee saw no reason to suspect
that there had been a misdelivery. The steps Kee had taken to protect his interests were reasonable. There was no
need for him to have acted ex-abundantia cautela, such as being present during the geodetic engineer's relocation
survey or hiring an independent geodetic engineer to countercheck for errors, for the final delivery of subdivision lots to
their owners is part of the regular course of everyday business of CTTEI. Because of CTTEI's blunder, what Kee had
hoped to forestall did in fact transpire. Kee's efforts all went to naught. 8

Good faith consists in the belief of the builder that the land he is building on is his and his ignorance of any defect or flaw in his
title 9 . And as good faith is presumed, petitioner has the burden of proving bad faith on the part of Kee 10 .

At the time he built improvements on Lot 8, Kee believed that said lot was what he bought from petitioner. He was not aware that
the lot delivered to him was not Lot 8. Thus, Kee's good faith. Petitioner failed to prove otherwise.

To demonstrate Kee's bad faith, petitioner points to Kee's violation of paragraphs 22 and 26 of the Contract of Sale on
Installment.

We disagree. Such violations have no bearing whatsoever on whether Kee was a builder in good faith, that is, on his state of
mind at the time he built the improvements on Lot 9. These alleged violations may give rise to petitioner's cause of action
against Kee under the said contract (contractual breach), but may not be bases to negate the presumption that Kee was a
builder in good faith.

Petitioner also points out that, as found by the trial court, the Contract of Sale on Installment covering Lot 8 between it and Kee
was rescinded long before the present action was instituted. This has no relevance on the liability of petitioner, as such fact does
not negate the negligence of its agent in pointing out the wrong lot. to Kee. Such circumstance is relevant only as it gives
Jardinico a cause of action for unlawful detainer against Kee.
CivRev (Property) Assignment 2
Petitioner next contends that Kee cannot "claim that another lot was erroneously pointed out to him" because the latter agreed to
the following provision in the Contract of Sale on installment, to wit:

13. The Vendee hereby declares that prior to the execution of his contract he/she has personally examined or inspected
the property made subject-matter hereof, as to its location, contours, as well as the natural condition of the lots and from
the date hereof whatever consequential change therein made due to erosion, the said Vendee shall bear the expenses
of the necessary fillings, when the same is so desired by him/her. 11

The subject matter of this provision of the contract is the change of the location, contour and condition of the lot due to erosion. It
merely provides that the vendee, having examined the property prior to the execution of the contract, agrees to shoulder the
expenses resulting from such change.

We do not agree with the interpretation of petitioner that Kee contracted away his right to recover damages resulting from
petitioner's negligence. Such waiver would be contrary to public policy and cannot be allowed. "Rights may be waived, unless
the waiver is contrary to law, public order, public policy, morals, or good customs, or prejudicial to a third person with a right
recognized by law." 12

The Second Issue:  Petitioner's Liability

Kee filed a third-party complaint against petitioner and CTTEI, which was dismissed by the RTC after ruling that there was no
evidence from which fault or negligence on the part of petitioner and CTTEI can be inferred. The Court of Appeals disagreed and
found CTTEI negligent for the erroneous delivery of the lot by Octaviano, its employee.

Petitioner does not dispute the fact that CTTEI was its agent. But it contends that the erroneous delivery of Lot 9 to Kee was an
act which was clearly outside the scope of its authority, and consequently, CTTEI I alone should be liable. It asserts that "while
[CTTEI] was authorized to sell the lot belonging to the herein petitioner, it was never authorized to deliver the wrong lot to
Kee" 13 .

Petitioner's contention is without merit.

The rule is that the principal is responsible for the acts of the agent, done within the scope of his authority, and should bear the
damage caused to third persons 14 . On the other hand, the agent who exceeds his authority is personally liable for the
damage 15

CTTEI was acting within its authority as the sole real estate representative of petitioner when it made the delivery to Kee. In
acting within its scope of authority, it was, however, negligent. It is this negligence that is the basis of petitioner's liability, as
principal of CTTEI, per Articles 1909 and 1910 of the Civil Code.

Pending resolution of the case before the Court of Appeals, Jardinico and Kee on July 24, 1987 entered into a deed of sale,
wherein the former sold Lot 9 to Kee. Jardinico and Kee did not inform the Court of Appeals of such deal.

The deed of sale contained the following provision:

1. That Civil Case No. 3815 entitled "Jardinico vs. Kee" which is now pending appeal with the Court of Appeals,
regardless of the outcome of the decision shall be mutually disregarded and shall not be pursued by the parties herein
and shall be considered dismissed and without effect whatso-ever; 16

Kee asserts though that the "terms and conditions in said deed of sale are strictly for the parties thereto" and that "(t)here is no
waiver made by either of the parties in said deed of whatever favorable judgment or award the honorable respondent Court of
Appeals may make in their favor against herein petitioner Pleasantville Development Corporation and/or private respondent C.T.
Torres Enterprises; Inc." 17

Obviously, the deed of sale can have no effect on the liability of petitioner. As we have earlier stated, petitioner's liability is
grounded on the negligence of its agent. On the other hand, what the deed of sale regulates are the reciprocal rights of Kee and
Jardinico; it stressed that they had reached an agreement independent of the outcome of the case.

Petitioner further assails the following holding of the Court of Appeals:

2. Third-party defendants C.T. Torres Enterprises, Inc. and Pleasantville Development Corporation are solidarily liable
under the following circumstances:

a. If Eldred Jardinico decides to appropriate the improvements and, thereafter, remove these structures, the
third-party defendants shall answer for all demolition expenses and the value of the improvements thus
destroyed or rendered useless;

b. If Jardinico prefers that Kee buy the land, the third-party defendants shall answer for the amount
representing the value of Lot 9 that Kee should pay to Jardinico. 18

Petitioner contends that if the above holding would be carried out, Kee would be unjustly enriched at its expense. In other words,
Kee would be able to own the lot, as buyer, without having to pay anything on it, because the aforequoted portion of respondent
Court's Decision would require petitioner and CTTEI jointly and solidarily to "answer" or reimburse Kee therefor.
CivRev (Property) Assignment 2
We agree with petitioner.

Petitioner' s liability lies in the negligence of its agent CTTEI. For such negligence, the petitioner should be held liable for
damages. Now, the extent and/or amount of damages to be awarded is a factual issue which should be determined after
evidence is adduced. However, there is no showing that such evidence was actually presented in the trial court; hence no
damages could flow be awarded.

The rights of Kee and Jardinico vis-a-vis  each other, as builder in good faith and owner in good faith, respectively, are regulated
by law (i.e., Arts. 448, 546 and 548 of the Civil Code). It was error for the Court of Appeals to make a "slight modification" in the
application of such law, on the ground of "equity". At any rate, as it stands now, Kee and Jardinico have amicably settled through
their deed of sale their rights and obligations with regards to Lot 9. Thus, we delete items 2 (a) and (b) of the dispositive portion
of the Court of Appeals' Decision [as reproduced above] holding petitioner and CTTEI solidarily liable.

The Third Issue: Attorney's Fees

The MTCC awarded Jardinico attorney's fees and costs in the amount of P3,000.00 and P700.00, respectively, as prayed for in
his complaint. The RTC deleted the award, consistent with its ruling that petitioner was without fault or negligence. The Court of
Appeals, however, reinstated the award of attorney's fees after ruling that petitioner was liable for its agent's negligence.

The award of attorney's fees lies within the discretion of the court and depends upon the circumstances of each case 19 . We
shall not interfere with the discretion of the Court of Appeals. Jardinico was compelled to litigate for the protection of his interests
and for the recovery of damages sustained as a result of the negligence of petitioner's agent 20 .

In sum, we rule that Kee is a builder in good faith. The disposition of the Court of Appeals that Kee "is entitled to the rights
granted him under Articles 448, 546 and 548 of the New Civil Code" is deleted, in view of the deed of sale entered into by Kee
and Jardinico, which deed now governs the rights of Jardinico and Kee as to each other. There is also no further need, as ruled
by the appellate Court, to remand the case to the court of origin "for determination of the actual value of the improvements and
the property (Lot 9), as well as for further proceedings in conformity with Article 448 of the New Civil Code."

WHEREFORE , the petition is partially GRANTED. The Decision of the Court of Appeals is hereby MODIFIED as follows:

(1) Wilson Kee is declared a builder in good faith;

(2) Petitioner Pleasantville Development Corporation and respondent C.T. Torres Enterprises, Inc. are declared
solidarily liable for damages due to negligence; however, since the amount and/or extent of such damages was not
proven during the trial, the same cannot now be quantified and awarded;

(3) Petitioner Pleasantville Development Corporation and respondent C.T. Torres Enterprises, Inc. are ordered to pay in
solidum the amount of P3,000.00 to Jardinico as attorney's fees, as well as litigation expenses; and

(4) The award of rentals to Jardinico is dispensed with.

SO ORDERED.
CivRev (Property) Assignment 2
Republic of the Philippines
SUPREME COURT

THIRD DIVISION

G.R. No. 157044 October 5, 2005

RODOLFO V. ROSALES, (represented by his heirs, Rodolfo, Jr., Romeo Allan, Lillian Rhodora, Roy Victor, Roger Lyle
and Alexander Nicolai, all surnamed Rosales) and LILY ROSQUETA-ROSALES, Petitioners
vs.
MIGUEL CASTELLTORT, JUDITH CASTELLTORT, and LINA LOPEZ-VILLEGAS, assisted by her Attorney-in-Fact, Rene
Villegas, Respondents.

DECISION

CARPIO MORALES,  J.:

The present petition for review on certiorari assails the October 2, 2002 Decision 1 and February 6, 2003 Resolution2 of the Court
of Appeals (CA) in CA G.R. CV No. 64046 and seeks to reinstate the April 21, 1999 Decision 3 of the Regional Trial Court (RTC)
of Calamba, Laguna, Branch 34 in Civil Case No. 2229-95-C.

Spouses-petitioners Rodolfo V. Rosales and Lily Rosqueta-Rosales (petitioners) are the registered owners of a parcel of land
with an area of approximately 315 square meters, covered by Transfer Certificate of Title (TCT) No. 36856 4 and designated as
Lot 17, Block 1 of Subdivision Plan LRC Psd-55244 situated in Los Baños, Laguna.

On August 16, 1995, petitioners discovered that a house was being constructed on their lot, without their knowledge and
consent, by respondent Miguel Castelltort (Castelltort). 5

It turned out that respondents Castelltort and his wife Judith had purchased a lot, Lot 16 of the same Subdivision Plan, from
respondent Lina Lopez-Villegas (Lina) through her son-attorney-in-fact Rene Villegas (Villegas) but that after a survey thereof by
geodetic engineer Augusto Rivera, he pointed to Lot 17 as the Lot 16 the Castelltorts purchased.

Negotiations for the settlement of the case thus began, with Villegas offering a larger lot near petitioners’ lot in the same
subdivision as a replacement thereof.6 In the alternative, Villegas proposed to pay the purchase price of petitioners’ lot with legal
interest.7 Both proposals were, however, rejected by petitioners 8 whose counsel, by letter9 of August 24, 1995, directed
Castelltort to stop the construction of and demolish his house and any other structure he may have built thereon, and desist from
entering the lot.

Petitioners subsequently filed on September 1, 1995 a complaint10 for recovery of possession and damages with prayer for the
issuance of a restraining order and preliminary injunction against spouses-respondents Miguel and Judith Castelltort before the
RTC of Calamba, Laguna, docketed as Civil Case No. 2229-95-C.

To the complaint, the Castelltorts claimed in their Answer with Counterclaim 11 that they were builders in good faith.

Lina, represented by her son-attorney-in-fact Villegas, soon filed a Motion for Intervention 12 before the RTC which was granted
by Order13 of December 19, 1995.

In her Answer to the complaint,14 Lina alleged that the Castelltorts acted in good faith in constructing the house on petitioners’ lot
as they in fact consulted her before commencing any construction thereon, they having relied on the technical description of the
lot sold to them, Lot 16, which was verified by her officially designated geodetic engineer.

Nevertheless, Lina proposed to give petitioners a lot containing an area of 536 square meters together with the house and
duplex structure built thereon or, if petitioners choose, to encumber the 536 square meter lot as collateral "to get immediate
cash" through a financing scheme in order to compensate them for the lot in question. 15

Ruling out good faith, the RTC, by Decision of April 21, 1999, found for petitioners in this wise:

In the instant case, there is no well-founded belief of ownership by the defendants of the land upon which they built their house.
The title or mode of acquisition upon which they based their belief of such ownership stemmed from a Contract to Sell (Exhibit
"P") of which they were not even parties, the designated buyer being Elizabeth Yson Cruz and the sale even subjected to the
judicial reconstitution of the title. And by their own actions, particularly defendant Miguel Castelltort, defendants betrayed this
very belief in their ownership when realizing the inutility of anchoring their ownership on the basis of the Contract of Sale,
defendant Miguel Castelltort in his testimony declared Elizabeth Yson Cruz as his wife (tsn, pp. 7-8, March 24, 1998) despite an
admission in their answer that they are the spouses named as defendants (tsn, p. 8, January 12, 1998) and which declaration is
an utter falsehood as the Contract to Sell itself indicates the civil status of said Elizabeth Yson Cruz to be single.

Even if we are to concede that defendants built their house in good faith on account of the representation of attorney-in-fact
Rene Villegas, their failure to comply with the requirements of the National Building Code, particularly the procurement of a
building permit, stained such good faith and belief.

xxx
CivRev (Property) Assignment 2
From any and all indications, this deliberate breach is an unmitigated manifestation of bad faith. And from the evidence thus
adduced, we hold that defendants and the intervenor were equally guilty of negligence which led to the construction of the
defendants’ house on plaintiffs’ property and therefore jointly and severally liable for all the damages suffered by the
plaintiffs.16 (Underscoring supplied)

The dispositive portion of the trial court’s Decision reads, quoted verbatim:

ACCORDINGLY, in view of all the foregoing, judgment is hereby rendered in favor of plaintiffs and against the defendants,
ordering the latter to surrender the possession of the property covered by TCT No. 36856 of the Register of Deeds of Laguna
including any and all improvements built thereon to the plaintiffs.

Defendants and intervenors are likewise jointly and severally directed to pay to plaintiffs the following damages:

a) TWO THOUSAND (P2,000.00) PESOS per month from February 1995 by way of reasonable compensation for the use of
plaintiffs’ property until the surrender of the same;

b) FIFTY THOUSAND (P50,000.00) PESOS by way of moral damages;

c) THIRTY THOUSAND (P30,000.00) PESOS as exemplary damages;

d) TWENTY THOUSAND (P20,000.00) PESOS as attorney’s fees and cost of suit.

The counterclaim interposed by the defendants in their responsive pleading is hereby dismissed for lack of merit.

SO ORDERED.17

Respondents thereupon filed their respective appeals with the CA.

Petitioner Rodolfo Rosales, in the meantime, died on December 7, 2001. His heirs Rodolfo, Jr., Romeo Allan, Lillian Rhodora,
Roy Victor, Roger Lyle and Alexander Nicolai, all surnamed Rosales, filed their Appearance 18 as his substitute.

By Decision of October 2, 2002, the CA granted the appeal and set aside the April 21, 1999 RTC Decision. The dispositive
portion of the Decision reads, quoted verbatim:

WHEREFORE, premises considered, the instant appeal is hereby GRANTED and the assailed decision of the court a
quo REVERSED AND SET ASIDE. In accordance with the cases of Technogas Philippines Manufacturing Corp. vs. Court of
Appeals and Depra vs. Dumlao, applying Article 448 of the Civil Code, this case is REMANDED to the Regional Trial Court of
Calamba, Laguna, Branch 34, for further proceedings, as follows:

1. to determine the present fair price of appellees’ 315 square meter area of land and the amount of the expenses actually spent
by the appellants for building the house as of 21 August 1995, which is the time they were notified of appellees’ rightful claim
over Lot 17.

2. to order the appellees to exercise their option under the law (Article 448, Civil Code), whether to appropriate the house as
their own by paying to the appellants the amount of the expenses spent for the house as determined by the court a quo in
accordance with the limitations as aforestated or to oblige the appellants to pay the price of the land.

In case the appellees exercise the option to oblige the appellants to pay the price of the land but the latter reject such purchase
because, as found by the court, the value of the land is considerably more than that of the house, the court shall order the
parties to agree upon the terms of a forced lease, and give the court a quo a formal written notice of such agreement and its
provisos. If no agreement is reached by the parties, the court a quo shall then fix the terms of the forced lease, provided that the
monthly rental to be fixed by the Court shall not be less that Two Thousand Pesos (P2,000.00) per month, payable within the
first five (5) days of each calendar month and the period thereof shall not be more than two (2) years, counted from the finality of
the judgment.

Upon the expiration of the forced lease, or upon default by the appellants in the payment of rentals for two (2) consecutive
months, the appellees shall be entitled to terminate the forced lease, to recover their land, and to have the improvement
removed by the appellants at the latter’s expense. The rentals herein provided shall be tendered by the appellants to the court
for payment to the appellees, and such tender shall constitute evidence of whether or not compliance was made within the
period fixed by the court.

In any event, the appellants shall pay the appellees the amount of Two Thousand Pesos (P2,000.00) as reasonable
compensation for their occupancy of the encroached property from the time said appellants’ good faith cease (sic) to exist until
such time the possession of the property is delivered to the appellees subject to the reimbursement of the aforesaid expenses in
favor of the appellants or until such time the payment of the purchase price of the said lot be made by the appellants in favor of
the appellees in case the latter opt for the compulsory sale of the same.

SO ORDERED.19 (Emphasis in the original)

In reversing the trial court, the CA held:

xxx
CivRev (Property) Assignment 2
x x x A perusal of the records readily reveals that said court instead relied on flimsy, if not immaterial, allegations of the
appellees, which have no direct bearing in the determination of whether the appellants are builders in bad faith.

For one, the pivotal issue to be resolved in this case, i.e. whether appellant Miguel is a builder in good faith, was ignored by the
court a quo. The instant case does not in any way concern the personal and property relations of spouses-appellants and
Elizabeth Yson Cruz which is an altogether different matter that can be ventilated by the concerned parties through the
institution of a proper action. xxx The court a quo should have focused on the issue of whether appellant Miguel built, in good
faith, the subject house without notice of the adverse claim of the appellees and under the honest belief that the lot which he
used in the construction belongs to him. xxx

xxx As it is, appellant Miguel relied on the title which the intervenor showed to him which, significantly, has no annotation that
would otherwise show a prior adverse claim. Thus, as far as appellant Miguel is concerned, his title over the subject lot, as well
as the title of the intervenor thereto, is clean and untainted by an adverse claim or other irregularities.

For another, the appellants’ failure to secure a building permit from the Municipal Engineer’s Office on their construction on Lot
17 does not impinge on the good faith of the appellants. In fact, it can be told that a building permit was actually filed by
appellant Miguel with respect to Lot 16 and it was only due to the confusion and misapprehension by the intervenor of the exact
parameters of the property which caused appellant’s belief that Lot 17 [the questioned lot], is his. This fact bolsters appellant
Miguel’s good faith in building his house on appellees’ lot under the mistaken belief that the same is his property. Otherwise, he
should have secured a building permit on Lot 17 instead or should not have bothered to take the necessary measures to obtain
a building permit on Lot 16 in the first place.

By and large, the records show that, as testified to by Engr. Rebecca T. Lanuang, appellant Miguel had already applied for a
building permit as early as February 1994 and was in fact issued a temporary building permit  pending the completion of the
requirements for said permit. Although the building permit was belatedly issued in January 1996, this does not in any way
detract from appellant Miguel’s good faith.

xxx

In holding the appellants as builders in bad faith, the court a quo defied law and settled jurisprudence considering that the factual
basis of its findings and the incontrovertible evidence in support thereof prove that the appellant Miguel, in good faith, built the
house on appellees’ land without knowledge of an adverse claim or any other irregularities that might cast a doubt as to the
veracity of the assurance given to him by the intervenor. Having been assured by the intervenor that the stone monuments were
purposely placed, albeit wrongfully, by the land surveyor in said land to specifically identify the lot and its inclusive boundaries,
the appellants cannot be faulted for having relied on the expertise of the land surveyor who is more equipped and experienced in
the field of land surveying. Although under the Torrens system of land registration, the appellant is presumed to have knowledge
of the metes and bounds of the property with which he is dealing, appellant however, considering that he is a layman not versed
in the technical description of his property, cannot be faulted in his reliance on the survey plan that was delivered to him by the
intervenor and the stone monuments that were placed in the encroached property.

xxx

Peremptorily, contrary to the flawed pronouncements made by the court a quo that appellant Miguel is deemed as a builder in
bad faith on the basis of a mere assertion that he built his house without initially satisfying himself that he owns the said
property, this Court finds reason to maintain good faith on the part of the appellant. Admittedly, the appellants’ house
erroneously encroached on the property of the appellees due to a mistake in the placement of stone monuments as indicated in
the survey plan, which error is directly attributable to the fault of the geodetic engineer who conducted the same. This fact alone
negates bad faith on the part of appellant Miguel.

xxx

Moreover, it is quite illogical for appellant Miguel to knowingly build his house on a property which he knew belongs to another
person. x x x

xxx

In view of the good faith of both parties in this case, their rights and obligations are to be governed by Article 448,
which has been applied to improvements or portions of improvements built by mistaken belief on land belonging to the
adjoining owner. x x x

x x x20 (Emphasis and underscoring supplied)

Petitioners’ Motion for Reconsideration21 dated October 22, 2002 having been denied by the CA by Resolution of March 13,
2002, the present petition was filed raising the following issues:

I.

WHETHER OR NOT THE HONORABLE COURT OF APPEALS COMMITTED A GRAVE ABUSE OF DISCRETION IN MAKING
A FINDING THAT IS CONTRARY TO THE ADMISSIONS BY THE PARTIES

II.
CivRev (Property) Assignment 2
WHETHER OR NOT THE HONORABLE COURT OF APPEALS COMMITTED A REVERSIBLE ERROR OF LAW IN
CONCLUDING THAT THE TRIAL COURT, IN DECIDING THE CASE, RELIED ON FLIMSY, IF NOT IMMATERIAL,
ALLEGATIONS OF THE PETITIONERS, WHICH HAVE NO DIRECT BEARING IN THE DETERMINATION OF WHETHER THE
RESPONDENTS ARE BUILDERS IN GOOD FAITH

III.

WHETHER OR NOT THE HONORABLE COURT OF APPEALS COMMITTED A REVERSIBLE ERROR OF LAW IN
RENDERING A DECISION THAT IS UNENFORCEABLE AGAINST BOTH RESPONDENT JUDITH CASTELLTORT AND
THIRD-PARTY ELIZABETH CRUZ22

Petitioners initially hammer against respondents’ proving that Castelltort and a certain Elizabeth Cruz are the builders of the
house on the subject property, they faulting them with estoppel for alleging in their Answer before the trial court that "they
(respondents Castelltort and Judith) caused the construction of their house which they bought from a certain Lina Lopez-
Villegas."

Petitioners rely on the following doctrine established in Elayda v. Court of Appeals:23

"an admission made in the pleadings cannot be controverted by the party making such admission and are conclusive as to him
and that all proofs submitted by him contrary thereto or inconsistent therewith, should be ignored, whether objection is
interposed by the party or not x x x"

Petitioners’ contention is hardly relevant to the case at bar. Whether it was Castelltort and Judith or Castelltort and Elizabeth
Cruz who purchased the property from Lina is not material to the outcome of the instant controversy. As found by the CA:

The fact remains that appellant [Castelltort] is the builder of the house on Lot 17 xxx The court a quo should have focused on the
issue of whether appellant Miguel built, in good faith, the subject house without notice of the adverse claim of the appellees and
under the honest belief that the lot which he used in the construction belongs to him. xxx it cannot be gainsaid that appellant
Miguel has a title over the land that was purchased from the intervenor x x x 24

At all events, as this Court held in the case of Gardner v. Court of Appeals:25

In its Resolution reversing the original Decision, respondent Court discredited the testimony of Ariosto SANTOS for being at
variance with the allegations in his Answer. The fact, however, that the allegations made by Ariosto SANTOS in his pleadings
and in his declarations in open Court differed will not militate against the findings herein made nor support the reversal by
respondent Court. As a general rule, facts alleged in a party’s pleading are deemed admissions of that party and binding upon it,
but this is not an absolute and inflexible rule. An Answer is a mere statement of fact which the party filing it expects to prove, but
it is not evidence. As Ariosto SANTOS himself, in open Court, had repudiated the defenses he had raised in his Answer and
against his own interest, his testimony is deserving of weight and credence. 26 (Underscoring supplied)

The issue determinative of the controversy in the case at bar hinges on whether Castelltort is a builder in good faith.

A builder in good faith is one who builds with the belief that the land he is building on is his, or that by some title one has the
right to build thereon, and is ignorant of any defect or flaw in his title. 27

Article 527 of the Civil Code provides that good faith is always presumed, and upon him who alleges bad faith on the part of a
possessor rests the burden of proof.28

In the case at bar, Lot 16 was sold by Lina, through her attorney-in-fact Villegas, to Castelltort and a certain Elizabeth Cruz 29 for
a consideration of ₱500,000.00. While prior to the sale, what Villegas showed Castelltort as evidence of his mother Lina’s
ownership of the property was only a photocopy of her title TCT No. (T-42171) T-18550 30 he explaining that the owner’s
duplicate of the title was lost and that judicial reconstitution thereof was ongoing, Castelltort acted in the manner of a prudent
man and went to the Registry of Deeds of Laguna to procure a certified true copy of the TCT. 31 The certified true copy bore no
annotation indicating any prior adverse claim on Lot 16.

The records indicate that at the time Castelltort began constructing his house on petitioners’ lot, he believed that it was the Lot
16 he bought and delivered to him by Villegas.

In his cross-examination, Villegas testified:

Q: You said the surveyor placed a mujon along boundary of the property?

A: Yes.

Q: When were the mujons placed in the boundary of the property?

A: These mujons were the basis for my locating the property in pointing to Mr. Castelltort.

xxx

Q: Is it not a fact that before Miguel Castelltort started constructing that house he sought your advice or permission to construct
the same over that particular lot?
CivRev (Property) Assignment 2
A: Yes.

Q: And you gave your consent?

A: Yes, because based on my knowledge also that that was the lot  as pointed by Engr. Rivera.

xxx

Q: Was there any remarkable difference between lot 16 and 17 at the time that this particular lot was sold to Miguel Castelltort
and Elizabeth Cruz?

xxx

A: Both lots 16 and 17 are practically the same. The (sic) have the same frontage. There is only a difference of 4 square meters,
one is 311 square meters and the other 315 square meters. Both sides were fenced, as drawn they were facing the same road.
They are practically the same.

Q: But at the time or immediately before Mr. Castelltort started the construction of the house, was there any remarkable
distinction between these two properties?

A: None.32 (Emphasis and underscoring supplied)

The confusion in the identification of Lot 16 was eventually traced to the error committed by geodetic engineer Augusto Rivera’s
employees in placing stone monuments on petitioners’ property, instead of on Lot 16, the lot sold to Castelltort, based on the
survey made by the engineer in 1992.

The engineer so testified:

Q: Now, aside from inspecting personally the site, what else did your men or assistants do?

A: After computing the subdivision lots, they went back to the field to plant those subdivision corners with concrete monuments.

Q: Which is (sic) also called as "mohons"?

A: Yes, sir.

Q: Now, can you point to this Honorable Court where exactly did your men place these additional mohons and how many?

A: Later on we discovered that they placed the mohons in the adjoining lot, lot 17.

xxx

Q: x x x when again did you meet Mr. Rene Villegas or after how many months or year?

A: Maybe after a year, sir.

Q: And you met him again because he had a problem regarding the property of one Engr. Rosales?

A: Yes, sir.

Q: And when he confided to you this matter, did you go to the site of Lot 16 or 17?

A: Yes, sir.

Q: And what did you see there?

A: A house being constructed then I rechecked the location of the house and it turned out to be in Lot 17.

xxx

Q: Considering that you found out that a mistake was actually made by your assistants Dennis Orencio, Mario Carpio and
Sovejano when you allowed them to proceed on their own to make this computation, did you confront these men of yours
afterwards?

A: Yes, sir.

Q: In what manner?

A: I actually reprimanded them verbally and also I dismissed Mario Carpio from my office.
CivRev (Property) Assignment 2
xxx

Q: And did you investigate how your men committed this mistake of planting these monuments on another lot when corners 4 &
1 were clearly planted on the ground?

A: I myself rechecked it and found out that they committed an error.

xxx

Q: And now, you are saying that your men committed a mistake by placing thereon monuments by planting these monuments
not on Lot 16 but on Lot 17?

A: When I investigated how did they commit (sic) a mistake it came to be like this. Before when we surveyed first this in 1992, at
that time Dante Villegas contracted my services there was a fence here then when we went back, the road was already removed
so they committed an error that this point is Lot 19, they thought that it was Lot 19, the back portion.

xxx

Q: In this particular case, did you find out how your men checked the succeeding lots, how they determine (sic) the exact
location of lot 16?

A: They just relied on one side of the subdivision.

Q: By just counting the number of lots?

A: Yes, sir.

Q: Without making any actual measurement?

A: They made an actual measurement but the reference point is not the one, the correct one because they also checked it with
the other corner of the road going back.

xxx

Q: And how did they commit a mistake when you said they checked the lot at the back of Lot 16?

A: Because they were quite confident since we had already relocated the property two years ago so they thought that they get
(sic) the right lot without checking the other side of the subdivision.

xxx

Q: Now, you said that when you went to the place because you heard from Rene Villegas that there was a mistake you no
longer could find the monuments on lines 1 and 4 and according to you the reason is that a fence was already constructed?

A: Yes, sir.

Q: For clarification, is this line 1 & 4 on Lot 16 a common line 1 &4 on Lot 17?

A: Yes, sir a common line.

Q: In other words, this line 1 &4 devides (sic) Lot 16 & 17?

A: Yes, sir.

Q: So that when these monuments were placed on lines 1 & 4 somebody could mistake it for Lot 17 also because there were
monuments now 1 &4 for lot 16 since these are common lines for

Lot 17 also with Lot 16, it could also be construed that these are monuments for Lot 17?

A: Yes, sir possible.33 (Underscoring supplied)

As correctly found by the CA, both parties having acted in good faith at least until August 21, 1995, the applicable provision in
this case is Article 448 of the Civil Code which reads:

Art. 448. The owner of the land on which anything has been built, sown or planted in good faith, shall have the right to
appropriate as his own the works, sowing or planting, after payment of the indemnity provided for in Articles 546 and 548, or to
oblige the one who built or planted to pay the price of the land, and the one who sowed, the proper rent. However, the builder or
planter cannot be obliged to buy the land if its value is considerably more than that of the building or trees. In such case, he shall
pay reasonable rent, if the owner of the land does not choose to appropriate the building or trees after proper indemnity. The
parties shall agree upon the terms of the lease and in case of disagreement, the court shall fix the terms thereof.
CivRev (Property) Assignment 2
Under the foregoing provision, the landowner can choose between appropriating the building by paying the proper indemnity or
obliging the builder to pay the price of the land, unless its value is considerably more than that of the structures, in which case
the builder in good faith shall pay reasonable rent.34 If the parties cannot come to terms over the conditions of the lease, the
court must fix the terms thereof.

The choice belongs to the owner of the land, a rule that accords with the principle of accession, i.e., that the accessory follows
the principal and not the other way around. Even as the option lies with the landowner, the grant to him, nevertheless, is
preclusive.35 The landowner cannot refuse to exercise either option and compel instead the owner of the building to remove it
from the land.36

The raison d’etre for this provision has been enunciated thus:

Where the builder, planter or sower has acted in good faith, a conflict of rights arises between the owners, and it becomes
necessary to protect the owner of the improvements without causing injustice to the owner of the land. In view of the
impracticability of creating a state of forced co-ownership, the law has provided a just solution by giving the owner of the land the
option to acquire the improvements after payment of the proper indemnity, or to oblige the builder or planter to pay for the land
and the sower the proper rent. He cannot refuse to exercise either option. It is the owner of the land who is authorized to
exercise the option, because his right is older, and because, by the principle of accession, he is entitled to the ownership of the
accessory thing.37

Possession acquired in good faith does not lose this character except in the case and from the moment facts exist which show
that the possessor is not unaware that he possesses the thing improperly or wrongfully. 38 The good faith ceases or is legally
interrupted from the moment defects in the title are made known to the possessor, by extraneous evidence or by suit for
recovery of the property by the true owner. 39

In the case at bar, Castelltort’s good faith ceased on August 21, 1995 when petitioners personally apprised him of their title over
the questioned lot. As held by the CA, should petitioners then opt to appropriate the house, they should only be made to pay for
that part of

the improvement built by Castelltort on the questioned property at the time good faith still existed on his part or until August 21,
1995.

The CA, however, failed to qualify that said part of the improvement should be pegged at its current fair market value consistent
with this Court’s pronouncement in Pecson v. Court of Appeals.40

And, as correctly found by the CA, the commencement of Castelltort’s payment of reasonable rent should start on August 21,
1995 as well, to be paid until such time that the possession of the property is delivered to petitioners, subject to the
reimbursement of expenses, that is, if such option is for petitioners to appropriate the house.

This Court quotes the CA’s ratiocination with approval:

x x x Generally, Article 448 of the Civil Code provides that the payment of reasonable rent should be made only up to the date
appellees serve notice of their option as provided by law upon the appellants and the court a quo; that is, if such option is for
appellees to appropriate the encroaching structure. In such event, appellants would have a right to retain the land on which they
have built in good faith until they are reimbursed the expenses incurred by them. This is so because the right to retain the
improvements while the corresponding indemnity is not paid implies the tenancy or possession in fact of the land on which it is
built, planted or sown.

However, considering that appellants had ceased as builders in good faith at the time that appellant Miguel was notified of
appellees’ lawful title over the disputed property, the payment of reasonable rent should accordingly commence at that time
since he can no longer avail of the rights provided under the law for builders in good faith. 41

If the option chosen by petitioners is compulsory sale, however, the payment of rent should continue up to the actual transfer of
ownership.42

Respecting petitioners’ argument that the appellate court erred in rendering a decision that is "unenforceable against Judith who
is not the owner of the house and Elizabeth Cruz who was found to be a part owner of the house built on their lot but is not a
party to the case," the same does not lie.

While one who is not a party to a proceeding shall not be affected or bound 43 by a judgment rendered therein,44 like Elizabeth
Cruz, this does not detract from the validity and enforceability of the judgment on petitioners and respondents Castelltorts.

WHEREFORE, the petition is DENIED. The Decision dated October 2, 2002 and Resolution dated February 6, 2003 of the Court
of Appeals are AFFIRMED with MODIFICATION such that the trial court shall include for determination the increase in value
("plus value") which petitioners’ 315 square meter lot may have acquired by reason of the existence of that portion of the house
built before respondents Miguel and Judith Castelltort were notified of petitioners’ rightful claim on said lot, and the current fair
market value of said portion.

SO ORDERED.
CivRev (Property) Assignment 2
SECOND DIVISION

G.R. No. 141463            August 6, 2002

VICTOR ORQUIOLA and HONORATA ORQUIOLA, petitioners,


vs.
HON. COURT OF APPEALS, HON. VIVENCIO S. BACLIG, Presiding Judge, Regional Trial Court, Branch 77, Quezon
City, THE SHERIFF OF QUEZON CITY and HIS/HER DEPUTIES and PURA KALAW LEDESMA, substituted by TANDANG
SORA DEVELOPMENT CORPORATION, respondents.

QUISUMBING, J.:

This petition for review seeks the reversal of the decision 1 of the Court of Appeals dated January 28, 1999 in CA-G.R. SP No.
47422, which dismissed the petition to prohibit Judge Vivencio Baclig of the Regional Trial Court of Quezon City, Branch 77,
from issuing a writ of demolition against petitioners, and the sheriff and deputy sheriff of the same court from implementing an
alias writ of execution. Also assailed is the resolution2 of the Court of Appeals dated December 29, 1999 which denied
petitioners’ motion for reconsideration.

The facts are as follows:

Pura Kalaw Ledesma was the registered owner of Lot 689, covered by TCT Nos. 111267 and 111266, in Tandang Sora,
Quezon City. This parcel of land was adjacent to certain portions of Lot 707 of the Piedad Estates, namely, Lot 707-A and 707-
B, registered in the name of Herminigilda Pedro under TCT Nos. 16951 and 16952, respectively. On October 29, 1964,
Herminigilda sold Lot 707-A and 707-B to Mariano Lising who then registered both lots and Lot 707-C in the name of M.B. Lising
Realty and subdivided them into smaller lots.1âwphi1.nêt

Certain portions of the subdivided lots were sold to third persons including herein petitioners, spouses Victor and Honorata
Orquiola, who purchased a portion of Lot 707-A-2, Lot 5, Block 1 of the subdivision plan (LRC), Psd-42965. The parcel is now
#33 Doña Regina St., Regina Village, Tandang Sora, Quezon City. The other portions were registered in the name of the heirs
of Pedro, heirs of Lising, and other third persons.

Sometime in 1969, Pura Kalaw Ledesma filed a complaint, docketed as Civil Case No. Q-12918, with the Regional Trial Court of
Quezon City against Herminigilda Pedro and Mariano Lising for allegedly encroaching upon Lot 689. During the pendency of the
action, Tandang Sora Development Corporation replaced Pura Kalaw Ledesma as plaintiff by virtue of an assignment of Lot 689
made by Ledesma in favor of said corporation. Trial continued for three decades.

On August 21, 1991, the trial court finally adjudged defendants Pedro and Lising jointly and severally liable for encroaching on
plaintiff’s land and ordered them:

(a) to solidarily pay the plaintiff Tandang Sora Dev. Corp. actual damages in the amount of P20,000 with interest from
date of filing of the complaint;

(b) to remove all construction, including barbed wires and fences, illegally constructed by defendants on plaintiff’s
property at defendants’ expense;

(c) to replace the removed concrete monuments removed by defendants, at their own expense;

(d) to pay attorney’s fees in the amount of FIVE THOUSAND PESOS (P5,000.00) with interest computed from the date
of filing of the complaint;

(e) to relocate the boundaries to conform with the Commissioners’ Report, particularly, Annexes "A" and "B" thereof, at
the expense of the defendants.3

As a result, in February 1998, the Deputy Sheriff of Quezon City directed petitioners, through an alias writ of execution, to
remove the house they constructed on the land they were occupying.

On April 2, 1998, petitioners received a Special Order dated March 30, 1998, from the trial court stating as follows:

Before the Court for resolution is the "Ex-Parte Motion For The Issuance of A Writ of Demolition," filed by plaintiff,
through counsel, praying for the issuance of an Order directing the Deputy Sheriff to cause the removal and/or
demolition of the structures on the plaintiff’s property constructed by defendants and/or the present occupants. The
defendants-heirs of Herminigilda Pedro filed their comment on the said Motion.

Considering that the decision rendered in the instant case had become final and executory, the Court, in its Order of
November 14, 1997, directed the issuance of an alias writ of execution for the enforcement of the said decision.
However, despite the service of the said writ to all the defendants and the present occupants of the subject property,
they failed to comply therewith, as per the Partial Sheriff’s Return, dated February 9, 1998, issued by the Deputy Sheriff
of this branch of the Court. Thus, there is now a need to demolish the structures in order to implement the said decision.

WHEREFORE, the defendants are hereby directed to remove, at their expense, all constructions, including barbed
wires and fences, which defendants constructed on plaintiff’s property, within fifteen (15) days from notice of this Order;
otherwise, this Court will issue a writ of demolition against them.
CivRev (Property) Assignment 2
SO ORDERED.4

To prohibit Judge Vivencio Baclig of the Regional Trial Court of Quezon City from issuing a writ of demolition and the Quezon
City sheriff from implementing the alias  writ of execution, petitioners filed with the Court of Appeals a petition for prohibition with
prayer for a restraining order and preliminary injunction on April 17, 1998. 5 Petitioners alleged that they bought the subject parcel
of land in good faith and for value, hence, they were parties in interest. Since they were not impleaded in Civil Case No. Q-
12918, the writ of demolition issued in connection therewith cannot be enforced against them because to do so would amount to
deprivation of property without due process of law.

The Court of Appeals dismissed the petition on January 28, 1999. It held that as buyers and successors-in-interest of Mariano
Lising, petitioners were considered privies who derived their rights from Lising by virtue of the sale and could be reached by the
execution order in Civil Case No. Q-12918. Thus, for lack of merit, the petition was ordered dismissed. 6

Petitioners’ motion for reconsideration was denied. Hence, this petition, where petitioners aver that:

I.

THE HONORABLE COURT OF APPEALS ERRED IN HOLDING THAT THE DECISION IN CIVIL CASE NO. Q-12918
CAN ALSO BE ENFORCED AGAINST THE PETITIONERS EVEN IF THEY WERE NOT IMPLEADED AS PARTIES
THERETO.

II.

THE HONORABLE COURT OF APPEALS ERRED IN NOT UPHOLDING PETITIONERS’ TITLE DESPITE THEIR
BEING BUILDER IN GOOD FAITH AND INNOCENT PURCHASER AND FOR VALUE.

III.

PETITIONERS ARE ENTITLED TO INJUNCTIVE RELIEF CONSIDERING THAT THEY STAND TO SUFFER GRAVE
AND IRREPARABLE INJURY IF ALIAS WRIT OF EXECUTION AND THE SPECIAL ORDER ISSUED BY THE COURT
A QUO IN CIVIL CASE NO. Q-12918 FOR THE DEMOLITION OF ALL THE STRUCTURES ON THE DISPUTED
PROPERTY WERE ENFORCED AGAINST THE PETITIONERS WHO WERE NOT EVEN GIVEN THEIR DAY IN
COURT.7

For our resolution are the following issues: (1) whether the alias writ of execution may be enforced against petitioners; and (2)
whether petitioners were innocent purchasers for value and builders in good faith.

On the first issue, petitioners claim that the alias writ of execution cannot be enforced against them. They argue that the
appellate court erred when it relied heavily on our ruling in Vda. de Medina vs. Cruz8  in holding that petitioners are successors-
in-interest of Mariano Lising, and as such, they can be reached by the order of execution in Civil Case No. Q-12918 even though
they were not impleaded as parties thereto. Petitioners submit that Medina is not applicable in this case because the
circumstances therein are different from the circumstances in the present case.

In Medina, the property in dispute was registered under Land Registration Act No. 496 in 1916 and Original Certificate of Title
No. 868 was issued in the name of Philippine Realty Corporation (PRC). In 1949, Benedicta Mangahas and Francisco Ramos
occupied and built houses on the lot without the PRC’s consent. In 1959, PRC sold the lot to Remedios Magbanua. Mangahas
and Ramos opposed and instituted Civil Case No. C-120 to annul the sale and to compel PRC to execute a contract of sale in
their favor. The trial court dismissed the complaint and ordered Mangahas and Ramos to vacate the lot and surrender
possession thereof to Magbanua. The judgment became final and executory. When Magbanua had paid for the land in full, PRC
executed a deed of absolute sale in her favor and a new title was consequently issued in her name. Magbanua then sought the
execution of the judgment in Civil Case No. C-120. This was opposed by petitioner Medina who alleged that she owned the
houses and lot subject of the dispute. She said that she bought the houses from spouses Ricardo and Eufrocinia de Guzman,
while she purchased the lot from the heirs of the late Don Mariano San Pedro y Esteban. The latter held the land by virtue of
a Titulo de Composicion Con El Estado Num. 4136, dated April 29, 1894. In opposing the execution, Medina argued that the trial
court did not acquire jurisdiction over her, claiming that she was not a party in Civil Case No. C-120, thus, she could not be
considered as "a person claiming under" Ramos and Mangahas.

When Medina  reached this Court, we held that the decision in Civil Case No. C-120, which had long become final and
executory, could be enforced against petitioner even though she was not a party thereto. We found that the houses on the
subject lot were formerly owned by Mangahas and Ramos who sold them to spouses de Guzman, who in turn sold them to
Medina. Under the circumstances, petitioner was privy to the two judgment debtors Mangahas and Ramos, and thus Medina
could be reached by the order of execution and writ of demolition issued against the two. As to the lot under dispute, we
sustained Magbanua’s ownership over it, she being the holder of a Torrens title. We declared that a Torrens title is generally
conclusive evidence of ownership of the land referred to therein, and a strong presumption exists that a Torrens title was
regularly issued and valid. A Torrens title is incontrovertible against any informacion possessoria, or other title existing prior to
the issuance thereof not annotated on the Torrens title. Moreover, persons dealing with property covered by a Torrens certificate
of title are not required to go beyond what appears on its face.

Medina markedly differs from the present case on major points. First, the petitioner in Medina  acquired the right over the houses
and lot subject of the dispute after the original action was commenced and became final and executory. In the present case,
petitioners acquired the lot before the commencement of Civil Case No. Q-12918. Second, the right over the disputed land of the
predecessors-in-interest of the petitioner in Medina  was based on a title of doubtful authenticity, allegedly a Titulo de
Composicion Con El Estado  issued by the Spanish Government in favor of one Don Mariano San Pedro y Esteban, while the
right over the land of the predecessors-in-interest of herein petitioners is based on a fully recognized Torrens title. Third,
CivRev (Property) Assignment 2
petitioners in this case acquired the registered title in their own names, while the petitioner in Medina merely relied on the title of
her predecessor-in-interest and tax declarations to prove her alleged ownership of the land.

We must stress that where a case like the present one involves a sale of a parcel of land under the Torrens system, the
applicable rule is that a person dealing with the registered property need not go beyond the certificate of title; he can rely solely
on the title and he is charged with notice only of such burdens and claims as are annotated on the title. 9 It is our view here that
the petitioners, spouses Victor and Honorata Orquiola, are fully entitled to the legal protection of their lot by the Torrens system,
unlike the petitioner in the Medina  case who merely relied on a mere Titulo de Composicion.

Coming now to the second issue, were petitioners purchasers in good faith and for value? A buyer in good faith is one who buys
the property of another without notice that some other person has a right to or interest in such property. He is a buyer for value if
he pays a full and fair price at the time of the purchase or before he has notice of the claim or interest of some other person in
the property.10 The determination of whether one is a buyer in good faith is a factual issue which generally is outside the
province of this Court to determine in a petition for review. An exception is when the Court of Appeals failed to take into account
certain relevant facts which, if properly considered, would justify a different conclusion. 11 The instant case is covered by this
exception to the general rule. As found by the Court of Appeals and not refuted by private respondent, petitioners purchased the
subject land in 1964 from Mariano Lising.12 Civil Case No. Q-12918 was commenced sometime in 1969. The Court of Appeals
overlooked the fact that the purchase of the land took place prior to the institution of Civil Case No. Q-12918. In other words, the
sale to petitioners was made before Pura Kalaw Ledesma claimed the lot. Petitioners could reasonably rely on Mariano Lising’s
Certificate of Title which at the time of purchase was still free from any third party claim. Hence, considering the circumstances
of this case, we conclude that petitioners acquired the land subject of this dispute in good faith and for value.

The final question now is: could we consider petitioners builders in good faith? We note that this is the first time that petitioners
have raised this issue. As a general rule, this could not be done. Fair play, justice, and due process dictate that parties should
not raise for the first time on appeal issues that they could have raised but never did during trial and even during proceedings
before the Court of Appeals.13 Nevertheless, we deem it proper that this issue be resolved now, to avoid circuitous litigation and
further delay in the disposition of this case. On this score, we find that petitioners are indeed builders in good faith.

A builder in good faith is one who builds with the belief that the land he is building on is his, and is ignorant of any defect or flaw
in his title.14 As earlier discussed, petitioner spouses acquired the land in question without knowledge of any defect in the title of
Mariano Lising. Shortly afterwards, they built their conjugal home on said land. It was only in 1998, when the sheriff of Quezon
City tried to execute the judgment in Civil Case No. Q-12918, that they had notice of private respondent’s adverse claim. The
institution of Civil Case No. Q-12918 cannot serve as notice of such adverse claim to petitioners since they were not impleaded
therein as parties.

As builders in good faith and innocent purchasers for value, petitioners have rights over the subject property and hence they are
proper parties in interest in any case thereon.15 Consequently, private respondents should have impleaded them in Civil Case
No. Q-12918. Since they failed to do so, petitioners cannot be reached by the decision in said case. No man shall be affected by
any proceeding to which he is a stranger, and strangers to a case are not bound by any judgment rendered by the court. In the
same manner, a writ of execution can be issued only against a party and not against one who did not have his day in court. Only
real parties in interest in an action are bound by the judgment therein and by writs of execution and demolition issued pursuant
thereto.16 In our view, the spouses Victor and Honorata Orquiola have valid and meritorious cause to resist the demolition of
their house on their own titled lot, which is tantamount to a deprivation of property without due process of law.1âwphi1.nêt

WHEREFORE, the petition is GRANTED. The decision of the Court of Appeals dated January 28, 1999, and its resolution dated
December 29, 1999, in CA-G.R. SP No. 47422, are REVERSED and SET ASIDE. Respondents are hereby enjoined from
enforcing the decision in Civil Case No. Q-12918 through a writ of execution and order of demolition issued against petitioners.
Costs against private respondent.

SO ORDERED.
CivRev (Property) Assignment 2
Republic of the Philippines
SUPREME COURT
Manila

SECOND DIVISION

G.R. No. 125683 March 2, 1999

EDEN BALLATAN and SPS. BETTY MARTINEZ and CHONG CHY LING, petitioners,
vs.
COURT OF APPEALS, GONZALO GO, WINSTON GO, LI CHING YAO, ARANETA INSTITUTE OF AGRICULTURE and
JOSE N. QUEDDING, respondents.

PUNO, J.:

This is a petition for review on certiorari of the decision of the Court of Appeals dated March 25, 1996 in CA-G.R. CV No. 32472
entitled "Eden Ballatan., et. al., plaintiffs-appellees v. Gonzalo Go and Winston Go, appellants and third-party plaintiffs-
appellants v. Li Ching Yao, et. al., third-party defendants."1

The instant case arose from a dispute over forty-two (42) square meters of residential land belonging to petitioners. The parties
herein are owners of adjacent lots located at Block No. 3, Poinsettia Street, Araneta University Village, Malabon, Metro Manila.
Lot No. 24, 414 square meters in area, is registered in the name of petitioners Eden Ballatan and spouses Betty Martinez and
Chong Chy Ling. 2 Lots Nos. 25 and 26, with an area of 415 and 313 square meters respectively, are registered in the name of
respondent Gonzalo Go, Sr. 3 On Lot No. 25, respondent Winston Go, son of Gonzalo Go, Sr., constructed his house. Adjacent
to Lot No. 26 is Lot No. 27, 417 square meters in area, and is registered in the name of respondent Li Ching Yao. 4

In 1985, petitioner Ballatan constructed her house on Lot No. 24. During the construction, she noticed that the concrete fence
and side pathway of the adjoining house of respondent Winston Go encroached on the entire length of the eastern side of her
property. 5 Her building contractor formed her that the area of her lot was actually less than that described in the title. Forthwith,
Ballatan informed respondent Go of this discrepancy and his encroachment on her property. Respondent Go, however, claimed
that his house, including its fence and pathway, were built within the parameters of his father's lot; and that this lot was surveyed
by Engineer Jose Quedding, the authorized surveyor of the Araneta Institute of Agriculture (AIA), the owner-developer of the
subdivision project.

Petitioner Ballatan called the attention of the IAI to the discrepancy of the land area in her title and the actual land area received
from them. The AIA authorized another survey of the land by Engineer Jose N. Quedding.

In a report dated February 28, 1985, Engineer Quedding found that the lot area of petitioner Ballatan was less by few meters
and that of respondent Li Ching Yao, which was three lots away, increased by two (2) meters. Engineer Quedding declared that
he made a verification survey of Lots Nos. 25 and 26 of respondents Go in 1983 and allegedly found the boundaries to have
been in their proper position. He, however, could not explain the reduction in Ballatan's area since he was not present at the
time respondents Go constructed their boundary walls. 6

On June 2, 1985, Engineer Quedding made a third relocation survey upon request of the parties. He found that Lot No. 24 lost
approximately 25 square meters on its eastern boundary that Lot No. 25, although found to have encroached on Lot No. 24, did
not lose nor gain any area; that Lot No. 26 lost some three (3) square meters which, however, were gained by Lot No. 27 on its
western boundary. 7 In short, Lots Nos. 25, 26 and 27 moved westward to the eastern boundary of Lot No. 24.

On the basis of this survey, on June 10, 1985, petitioner Ballatan made a written demand on respondents Go to remove and
dismantle their improvements on Lot No. 24. Respondents Go refused. The parties including Li Ching Yao, however, met
several times to reach an agreement one matter.

Failing to agree amicably, petitioner Ballatan brought the issue before the barangay. Respondents Go did not appear. Thus, on
April 1, 1986, petitioner Ballatan instituted against respondents Go Civil Case No. 772-MN for recovery of possession before the
Regional Trial Court, Malabon, Branch 169. The Go' s filed their "Answer with Third-Party Complaint" impleading as third-party
defendants respondents Li Ching Yao, the AIA and Engineer Quedding.

On August 23, 1990, the trial court decided in favor of petitioners. It ordered the Go's to vacate the subject portion of Lot No. 24,
demolish their improvements and pay petitioner Ballatan actual damages, attorney's fees and the costs of the suit. It dismissed
the third-party complaint against: (1) AIA after finding that the lots sold to the parties were in accordance with the technical
description a verification plan covered by their respective titles; (2) Jose N. Quedding, there being no privity of relation between
him and respondents Go and his erroneous survey having been made at the instance of AIA, not the parties; and (3) Li Ching
Yao for failure to prove that he committed any wrong in the subject encroachment. 8 The court made the following disposition:

WHEREFORE, judgment is hereby rendered in favor of the plaintiffs and against the defendants, ordering the
latter:

1. To demolish and remove all improvements existing and encroaching on plaintiff's lot;
CivRev (Property) Assignment 2
2. To clear, vacate and deliver possession of the encroached area to the plaintiffs;

3. To pay plaintiffs jointly and severally the following:

a) P7,800.00 for the expenses paid to the surveyors;

b) P5,000.00 for plaintiffs' transportation;

4. To pay plaintiffs, jointly and severally, attorney's fees equivalent to 25% of the current market value of the
subject matter in litigation at the time of execution; and

5. To pay the costs of suit.

The third-party complaint filed by third-party plaintiff Gonzalo Go and Winston Go against third-party
defendants Araneta Institute of Agriculture, Jose N. Quedding and Li Ching Yao is hereby DISMISSED, without
pronouncement as to costs.

SO ORDERED.

Respondents Go appealed. On March 25, 1996, the Court of Appeals modified the decision of the trial court. It affirmed the
dismissal of the third-party complaint against the AIA but reinstated the complaint against Li Ching Yao and Jose Quedding.
Instead of ordering respondents Go to demolish their improvements on the subject land, the appellate court ordered them to pay
petitioner Ballatan, and respondent Li Ching Yao to pay respondents Go, a reasonable amount for that portion of the lot which
they encroached, the value to be fixed at the time of taking. It also ordered Jose Quedding to pay respondents Go attorney's
fees of P5,000.00 for his erroneous survey. The dispositive portion of the decision reads:

WHEREFORE, premises considered, the decision appealed from is hereby AFFIRMED insofar as the dismissal
of the third-party complaint against Araneta Institute of Agriculture is concerned but modified in all other
aspects as follows:

1) Defendants-appellants are hereby ordered to pay plaintiffs-appellees the reasonable value of the forty-two
(42) square meters of their lot at the time of its taking;

2) Third-party defendant Li Ching Yao is hereby ordered to pay defendants-appellants the reasonable value of
the thirty-seven (37) square meters of the latter's lot at the time of its taking; and

3) Third-party defendant Jose N. Quedding is hereby ordered to pay to defendants-appellants the amount of
P5,000.00 as attorney's fees.

LET THE RECORD of the case be remanded to the Regional Trial Court of Malabon for further proceedings
and reception of evidence for the determination of the reasonable value of Lots Nos. 24 and 26.

SO ORDERED.9

Hence, this petition. Petitioners allege that:

RESPONDENT COURT OF APPEALS ERRED ON QUESTIONS OF LAW AND GRAVELY ABUSED ITS
DISCRETION AMOUNTING TO LACK OF JURISDICTION WHEN:

1. IT APPLIED EQUITY OR EQUITABLE SOLUTIONS TO THE INSTANT CASE IN UTTER DISREGARD AND
IN VIOLATION OR GROSS IGNORANCE OF EXISTING LAWS AND JURISPRUDENCE VESTING BASIC
PROPERTY RIGHTS TO HEREIN PETITIONERS. RESPONDENT COURT HAS NO POWER TO APPLY/USE
EQUITY IN THE PRESENCE OF EXISTING LAWS TO THE CONTRARY.

2. UNDER THE GUISE OF APPLYING EQUITY BUT IN EFFECT A VERY APPARENT PARTIALITY AND
FAVOR TO RESPONDENTS GO, IT ORDERED PAYMENT OF THE ENCROACHED AREA AT THE VALUE
AT THE TIME OF ITS TAKING AND NOT THE VALUE AT THE TIME OF PAYMENT, THEREBY ENRICHING
THE GO'S BUT DEPRIVING PETITIONERS OF THE FRUITS OR INCREASE IN VALUE OF THEIR
PROPERTY TO WHICH THEY ARE ENTITLED UNDER THE LAW AS THE REGISTERED OWNERS WITH
TORRENS TITLE IN THEIR NAMES.

3. WHEN IT DID NOT DISMISS THE THIRD-PARTY COMPLAINT DUE TO NON-PAYMENT OF ANY FILING
OR DOCKET FEE.

4. WHEN IT DENIED PETITIONERS THE RECOVERY OF THE NECESSARY EXPENSES IN PROTECTING


THEIR RIGHTS IN THIS CASE. 10

Petitioners question the admission by respondent Court of Appeals of the third-party complaint by respondents Go against the
AIA, Jose Quedding and Li Ching Yao. Petitioners claim that the third-party complaint should not have been considered by the
Court of Appeals for lack of jurisdiction due to third-party plaintiffs' failure to pay the docket and filing fees before the trial court.
CivRev (Property) Assignment 2
The third-party complaint in the instant case arose from the complaint of petitioners against respondents Go. The complaint filed
was for accion publiciana, i.e., the recovery of possession of real property which is a real action. The rule in this jurisdiction is
that when an action is filed in court, the complaint must be accompanied the payment of the requisite docket and filing fees. 11 In
real actions, the docket and filing fees are based on the value of the property and the amount of damages claimed, if any 12 If the
complaint is filed but the fees are not paid at the time of filing, the court acquires jurisdiction upon full payment of the fees within
a reasonable time as the court may grant, barring prescription. 13 Where the fees prescribed for the real action have been paid
but the fees of certain related damages are not, the court, although having jurisdiction over the real action, may not have
acquired jurisdiction over the accompnying claim for damages. 14 Accordingly, the court may expunge those claims for damages,
or allow, on motion, a reasonable time for amendment of the complaint so as to allege the precise amount of damages and
accept payment of the requisite legal fee. 15 If there are unspecified claims, the determination of which may arise after the filing
of the complaint or similar pleading, the additional filing fee thereon shall constitute a lien on the judgment award. 16 The same
rule also applies to third-party claims and other similar pleadings. 17

In the case at bar, the third-party complaint filed by respondents Go was incorporated in their answer to the complaint. The third-
party complaint sought the same remedy as the principal complaint but added a prayer for attorney's fees and costs without
specifying their amounts, thus:

ON THE THIRD PARTY COMPLAINT

1. That summons be issued against Third-Party Defendants Araneta Institute of Agriculture, Jose N. Quedding
and Li Ching Yao;

2. That after hearing, they be sentenced to indemnify the Third-Party Plaintiffs for whatever is adjudged against
the latter in favor of the Plaintiffs;

3. That Third-Party Defendants be ordered to pay attorney's fees as may be proved during trial;

4. That Third-Party Defendants be ordered to pay the costs.

Other just and equitable reliefs are also prayed for. 18

The Answer with Third-Party Complaint was admitted by the trial court without the requisite payment of filing fees, particularly on
the Go's prayer for damages. 19 The trial court did not award the Go's any damages. It dismissed the third-party complaint. The
Court of Appeals, however, granted the third-party complaint in part by ordering third-party defendant Jose N. Quedding to pay
the Go's the sum of P5,000.00 as attorney's fees.

Contrary to petitioners' claim, the Court of Appeal did not err in awarding damages despite the Go's failure to specify the amount
prayed for and pay the corresponding additional filing fees thereon. The claim for attorney's fees refers to damages arising after
the filing of the complaint against the Go's. The additional filing fee on this claim is deemed to constitute a lien on the judgment
award. 20

The Court of Appeals found that the subject portion is actually forty-two (42) square meters in area, not forty-five (45), as initially
found by the trial court; that this forty-two (42) square meter portion is on the entire eastern side of Lot No. 24 belonging to
petitioners; that this said portion is found the concrete fence and pathway that extends from respondent Winston Go's house on
adjacent Lot No. 25; that inclusive of the subject portion, respondents Go did not gain nor lose any portion of Lots Nos. 25 and
26; that instead, Lot No. 27, on which respondent Li Ching Yao built his house, encroached on the land of respondents Go,
gaining in the process thirty-seven (37) square meters of the latter's land. 21

We hold that the Court of Appeals correctly dismissed the third-party complaint against AIA.. The claim that the discrepancy in
the lot areas was due to AIA's fault was not proved. The appellate court, however, found that it was the erroneous survey by
Engineer Quedding that triggered these discrepancies. And it was this survey that respondent Winston Go relied upon in
constructing his house on his father's land. He built his house in the belief that it was entirely within the parameters of his father's
land. In short, respondents Go had no knowledge that they encroached petitioners' lot. They are deemed builders in good
faith 22 until the time petitioner Ballatan informed them of their encroachment on her property. 23

Respondent Li Ching Yao built his house on his lot before any of the other parties did. 24 He constructed his house in 1982,
respondents Go in 1983, and petitioners in 1985. 25 There is no evidence, much less, any allegation that respondent Li Ching
Yao was aware that when he built his house he knew that a portion thereof encroached on respondents Go's adjoining land.
Good faith is always presumed, and upon him who alleges bad faith on the part of a possessor rests the burden of proof. 26

All the parties are presumed to have acted in good faith. Their rights must, therefore, be determined in accordance with the
appropriate provisions of the Civil Code on property.

Art. 448 of the Civil Code provides:

Art. 448. The owner of the land on which anything has been built, sown or planted in good faith, shall have the
right to appropriate as his own the works, sowing or planting, after payment of the indemnity provided for in
Articles 546 and 548, 27 or to oblige the one who built or planted to pay the price of the land, and the one who
sowed the proper rent. However, the builder or planter cannot be obliged to buy the land if its value is
considerably more than that of the building or trees. In such case, he shall pay reasonable rent, if the owner of
the land does not choose to appropriate the building or trees after proper indemnity. The parties shall agree
upon the terms of the lease and in case of disagreement, the court shall fix the terms thereof.
CivRev (Property) Assignment 2
The owner of the land on which anything has been built, sown or planted in good faith shall have the right to appropriate
as his own the building, planting or sowing, after payment to the builder, planter or sower of the necessary and useful
expenses, and in the proper case, expenses for pure luxury or mere pleasure. The owner of the land may also oblige
the builder, planter or sower to purchase and pay the price of the land. If the owner chooses to sell his land, the builder,
planter or sower must purchase the land, otherwise the owner may remove the improvements thereon. The builder,
planter or sower, however, is not obliged to purchase the land if its value considerably more than the building, planting
or sowing. In such case, the builder, planter or sower must pay rent to the owner of the land. If the parties cannot come
to terms over the conditions of the lease, the court must fix the terms thereof. The right to choose between appropriating
the improvement or selling the land on which the improvement stands to the builder, planter or sower, is given to the
owner of the land. 28

Art. 448 has been applied to improvements or portions of improvements built by mistaken belief on land belonging to the
adjoining owner. 29 The facts of the instant case are similar to those in Cabral v. Ibanez, 30 to wit:

[P]laintiffs Geronima Zabala and her husband Justino Bernardo, constructed their house in the belief that it was
entirely within the area of their own land without knowing at that time that part of their house was occupying a
14-square meter portion of the adjoining lot belonging to the defendants, and that the defendants Bernardo M.
Cabral and Mamerta M. Cabral were likewise unaware of the fact that a portion of plaintiff's house was
extending and occupying a portion of their lot with an area of 14 square meters. The parties came to know of
the fact that part of the plaintiff's house was occupying part of defendant's land when the construction of
plaintiff's house was about to be finished, after a relocation of the monuments of the two properties had been
made by the U.S. Army through the Bureau of Lands, according to their "Stipulation of Facts," dated August 17,
1951.

On the basis of these facts, we held that:

The court, therefore, concludes that the plaintiffs are builders in good faith and the relative rights of the
defendant Mamerta Cabral as owner of the land and of the plaintiffs as owners of the building is governed by
Article 361 of the Civil Code (Co Tao v. Joaquin Chan Chico, 46 Off. Gaz.5514). Article 361 of the old Civil
Code has been reproduced with an additional provision in Article 448 of the new Civil Code, approved June 18,
1949. 31

Similarly, in Grana and Torralba v. Court of Appeals,32 we held that:

Although without any legal and valid claim over the land in question, petitioners, however, were found by the
Court of Appeals to have constructed a portion of their house thereon in good faith. Under Article 361 of the old
Civil Code (Article 448 of the new), the owner of the land on which anything has been built in good faith shall
have the right to appropriate as his own the building, after payment to the builder of necessary or useful
expenses, and in the proper case, expenses for pure luxury or mere pleasure, or to oblige the builder to pay the
price of the land. Respondents, as owners of the land, have therefore the choice of either appropriating the
portion of petitioners' house which is on their land upon payment of the proper indemnity to petitioners, or
selling to petitioners that part of their land on which stands the improvement. It may here be pointed out that it
would be impractical for respondents to choose to exercise the first alternative, i.e., buy that portion of the
house standing on their land, for in that event the whole building might be rendered useless. The more
workable solution, it would seem, is for respondents to sell to petitioners that part of their land on which was
constructed a portion of the latter's house. If petitioners are unwilling or unable to buy, then they must vacate
the land and must pay rentals until they do so. Of course, respondents cannot oblige petitioners to buy the land
if its value is considerably more than that of the aforementioned portion of the house. If such be the case, then
petitioners must pay reasonable rent. The parties must come to an agreement as to the conditions of the lease,
and should they fail to do so, then the court shall fix the same. 33

In light of these rulings, petitioners, as owners of Lot No. 24, may choose to purchase the improvement made by respondents
Go on their land, or sell to respondents Go the subject portion. If buying the improvement is impractical as it may render the Go's
house useless, then petitioners may sell to respondents Go that portion of Lot No. 24 on which their improvement stands. If the
Go's are unwilling or unable to buy the lot, then they must vacate the land and, until they vacate, they must pay rent to
petitioners. Petitioners, however, cannot compel respondents Go to buy the land if its value is considerably more than the
portion of their house constructed thereon. If the value of the land is much more than the Go's improvement, the respondents Go
must pay reasonable rent. If they do not agree on the terms of the lease, then they may go to court to fix the same.

In the event that petitioners elect to sell to respondents Go the subject portion of their lot, the price must be fixed at the
prevailing market value at the time of payment. The Court of Appeals erred in fixing the price at the time of taking, which is the
time the improvements were built on the land. The time of taking is determinative of just compensation in expropriation
proceedings. The instant case is not for expropriation. It is not a taking by the state of private property for a public purpose upon
payment of just compensation. This is a case of an owner who has been paying real estate taxes on his land but has been
deprived of the use of a portion of this land for years. It is but fair and just to fix compensation at the time of payment. 34

Art. 448 and the same conditions abovestated also apply to respondents Go as owners and possessors of their land and
respondent Li Ching Yao as builder of the improvement that encroached on thirty-seven (37) square meters of respondents Go's
land.

IN VIEW WHEREOF, the decision of respondent Court of Appeals is modified as follows:

(1) Petitioners are ordered to exercise within thirty (30) days from finality of this decision their option to either buy the portion of
respondents Go's improvement on their Lot No. 24, or sell to said respondents the portion of their land on which the
CivRev (Property) Assignment 2
improvement stands. If petitioners elect to sell the land or buy the improvement, the purchase price must be at the prevailing
market price at the time of payment. If buying the improvement will render respondents Go's house useless, then petitioners
should sell the encroached portion of their land to respondents Go. If petitioners choose to sell the land but respondents Go are
unwilling or unable to buy, then the latter must vacate the subject portion and pay reasonable rent from the time petitioners
made their choice up to the time they actually vacate the premises. But if the value of the land is considerably more than the
value of the improvement, then respondents Go may elect to lease the land, in which case the parties shall agree upon the
terms, the lease. Should they fail to agree on said terms, the court of origin is directed to fix the terms of the lease.

From the moment petitioners shall have exercised their option, respondents Go shall pay reasonable monthly rent up to the time
the parties agree on the terms of the lease or until the court fixes such terms.

(2) Respondents Go are likewise directed to exercise their rights as owners of Lots Nos. 25 and 26, vis-a-vis respondent Li
Ching Yao as builder of the improvement that encroached on thirty seven (37) square meters of respondents Go 's land in
accordance with paragraph one abovementioned.

(3) The Decision of the Court of Appeals ordering Engineer Quedding, as third-party defendant, to pay attorney's fees of
P5,000.00 to respondents Go is affirmed. The additional filing fee on the damages constitutes a lien on this award.

(4) The Decision of the Court of Appeals dismissing third-party complaint against Araneta Institute of Agriculture is affirmed.

SO ORDERED.
CivRev (Property) Assignment 2
THIRD DIVISION

G.R. Nos. 154391-92             September 30, 2004

Spouses ISMAEL and TERESITA MACASAET, petitioners,


vs.
Spouses VICENTE and ROSARIO MACASAET, respondents.

DECISION

PANGANIBAN, J.:

The present case involves a dispute between parents and children. The children were invited by the parents to occupy the
latter’s two lots, out of parental love and a desire to foster family solidarity. Unfortunately, an unresolved conflict terminated this
situation. Out of pique, the parents asked them to vacate the premises. Thus, the children lost their right to remain on the
property. They have the right, however, to be indemnified for the useful improvements that they constructed thereon in good
faith and with the consent of the parents. In short, Article 448 of the Civil Code applies.

The Case

Before us is a Petition for Review1 under Rule 45 of the Rules of Court, assailing the March 22, 2002 Decision 2 and the June 26,
2002 Resolution3 of the Court of Appeals (CA) in CA-GR SP Nos. 56205 & 56467. The challenged Decision disposed as follows:

"WHEREFORE, the assailed Decision is AFFIRMED with the following MODIFICATIONS:

‘1. Vicente and Rosario should reimburse Ismael and Teresita one-half of the value of the useful improvements
introduced in the premises prior to demand, which is equivalent to ₱475,000.00. In case the former refuse to
reimburse the said amount, the latter may remove the improvements, even though the land may suffer damage
thereby. They shall not, however, cause any more impairment upon the property leased than is necessary.

‘2. The award of attorney’s fees is DELETED.

‘3. The records of these consolidated cases are REMANDED to the Court of origin for further proceedings to
determine the option to be taken by Vicente and Rosario and to implement the same with dispatch." 4

The assailed Resolution denied petitioners’ Motion for Reconsideration.

The Facts

Petitioners Ismael and Teresita5 Macasaet and Respondents Vicente and Rosario Macasaet are first-degree relatives. Ismael is
the son of respondents, and Teresita is his wife.6

On December 10, 1997, the parents filed with the Municipal Trial Court in Cities (MTCC) of Lipa City an ejectment suit against
the children.7 Respondents alleged that they were the owners of two (2) parcels of land covered by Transfer Certificate of Title
(TCT) Nos. T-78521 and T-103141, situated at Banay-banay, Lipa City; that by way of a verbal lease agreement, Ismael and
Teresita occupied these lots in March 1992 and used them as their residence and the situs of their construction business; and
that despite repeated demands, petitioners failed to pay the agreed rental of ₱500 per week. 8

Ismael and Teresita denied the existence of any verbal lease agreement. They claimed that respondents had invited them to
construct their residence and business on the subject lots in order that they could all live near one other, employ Marivic (the
sister of Ismael), and help in resolving the problems of the family. 9 They added that it was the policy of respondents to allot the
land they owned as an advance grant of inheritance in favor of their children. Thus, they contended that the lot covered by TCT
No. T-103141 had been allotted to Ismael as advance inheritance. On the other hand, the lot covered by TCT No. T-78521 was
allegedly given to petitioners as payment for construction materials used in the renovation of respondents’ house. 10

The MTCC11 ruled in favor of respondents and ordered petitioners to vacate the premises. It opined that Ismael and Teresita had
occupied the lots, not by virtue of a verbal lease agreement, but by tolerance of Vicente and Rosario. 12 As their stay was by
mere tolerance, petitioners were necessarily bound by an implied promise to vacate the lots upon demand. 13 The MTCC
dismissed their contention that one lot had been allotted as an advance inheritance, on the ground that successional rights were
inchoate. Moreover, it disbelieved petitioners’ allegation that the other parcel had been given as payment for construction
materials.14

On appeal, the regional trial court15 (RTC) upheld the findings of the MTCC. However, the RTC allowed respondents to
appropriate the building and other improvements introduced by petitioners, after payment of the indemnity provided for by Article
448 in relation to Articles 546 and 548 of the Civil Code.16 It added that respondents could oblige petitioners to purchase the
land, unless its value was considerably more than the building. In the latter situation, petitioners should pay rent if respondents
would not choose to appropriate the building.17

Upon denial of their individual Motions for Reconsideration, the parties filed with the CA separate Petitions for Review, which
were later consolidated.18

Ruling of the Court of Appeals


CivRev (Property) Assignment 2
The CA sustained the finding of the two lower courts that Ismael and Teresita had been occupying the subject lots only by the
tolerance of Vicente and Rosario.19 Thus, possession of the subject lots by petitioners became illegal upon their receipt of
respondents’ letter to vacate it.20

Citing Calubayan v. Pascual,21 the CA further ruled that petitioners’ status was analogous to that of a lessee or a tenant whose
term of lease had expired, but whose occupancy continued by tolerance of the owner. 22 Consequently, in ascertaining the right
of petitioners to be reimbursed for the improvements they had introduced on respondents’ properties, 23 the appellate court
applied the Civil Code’s provisions on lease. The CA modified the RTC Decision by declaring that Article 448 of the Civil Code
was inapplicable. The CA opined that under Article 1678 of the same Code, Ismael and Teresita had the right to be reimbursed
for one half of the value of the improvements made.24

Not satisfied with the CA’s ruling, petitioners brought this recourse to this Court. 25

The Issues

Petitioners raise the following issues for our consideration:

"1. a) Whether or not Section 17[,] Rule 70 of the Rules of Court on Judgment should apply in the rendition of the
decision in this case;

b) Whether or not the Complaint should have been dismissed;

c) Whether or not damages including attorney’s fees should have been awarded to herein petitioners;

"2. a) Whether or not the rule on appearance of parties during the Pretrial should apply on appearance of parties during
Preliminary Conference in an unlawful detainer suit;

b) Whether or not the case of Philippine Pryce Assurance Corporation vs. Court of Appeals (230 SCRA 164) is
applicable to appearance of parties in an unlawful detainer suit;

"3. Whether or not Article 1678 of the Civil Code should apply to the case on the matters of improvements, or is it Article
447 of the Civil Code in relation to the Article 453 and 454 thereof that should apply, if ever to apply the Civil Code;

"4. Whether or not the [D]ecision of the Court of Appeals is supported by evidence, appropriate laws, rules and
jurisprudence;

"5. Whether or not Assisting Judge Norberto Mercado of the MTCC Lipa City should be held accountable in rendering
the MTCC [D]ecision;

"6. Whether or not Atty. Glenn Mendoza and Atty. Andrew Linatoc of the same [l]aw office should be held accountable
for pursuing the [e]jectment case[.]" 26

The Court’s Ruling

The Petition is partly meritorious.

First Issue:

Ejectment

Who is entitled to the physical or material possession of the premises? At the outset, we stress that this is the main issue in
ejectment proceedings.27 In the present case, petitioners failed to justify their right to retain possession of the subject lots, which
respondents own. Since possession is one of the attributes of ownership, 28 respondents clearly are entitled to physical or
material possession.

Allegations of the Complaint

Petitioners allege that they cannot be ejected from the lots, because respondents based their Complaint regarding the
nonpayment of rentals on a verbal lease agreement, which the latter failed to prove. 29 Petitioners contend that the lower courts
erred in using another ground (tolerance of possession) to eject them.

In actions for unlawful detainer, possession that was originally lawful becomes unlawful upon the expiration or termination of the
defendant’s right to possess, arising from an express or implied contract. 30 In other words, the plaintiff’s cause of action comes
from the expiration or termination of the defendant’s right to continue possession. 31 The case resulting therefrom must be filed
within one year from the date of the last demand.

To show a cause of action in an unlawful detainer, an allegation that the defendant is illegally withholding possession from the
plaintiff is sufficient. The complaint may lie even if it does not employ the terminology of the law, provided the said pleading is
couched in a language adequately stating that the withholding of possession or the refusal to vacate has become unlawful. 32 It is
equally settled that the jurisdiction of the court, as well as the nature of the action, is determined from the averments of the
complaint.33
CivRev (Property) Assignment 2
In the present case, the Complaint alleged that despite demands, petitioners "refused to pay the accrued rentals and [to] vacate
the leased premises."34 It prayed that judgment be rendered "[o]rdering [petitioners] and all those claiming rights under them to
vacate the properties x x x and remove the structures x x x constructed thereon." 35 Effectively then, respondents averred that
petitioners’ original lawful occupation of the subject lots had become unlawful.

The MTCC found sufficient cause to eject petitioners. While it disbelieved the existence of a verbal lease agreement, it
nevertheless concluded that petitioners’ occupation of the subject lots was by mere tolerance of respondents. Basing its
conclusion on the fact that the parties were close relatives, the MTCC ruled thus:

"x x x [T]he parties herein are first degree relatives. Because of this relationship, this Court takes judicial notice of the
love, care, concern and protection imbued upon the parents towards their [children], i.e., in the instant case, the love,
care, concern and protection of the [respondents] to the [petitioners]. With this in mind, this Court is inclined to believe
the position of the [petitioners] that there was no such verbal lease agreement between the parties herein that took
place in 1992. x x x.

"From the allegations of the [petitioners], this Court is convinced that their stay and occupancy of the subject premises
was by mere tolerance of the [respondents], and not by virtue of a verbal lease agreement between them." 36

Having found a cause of action for unlawful detainer, the MTCC (as well as the RTC and the CA) did not err in ordering the
ejectment of petitioners as prayed for by respondents. There was no violation of Section 17 of Rule 70 37 of the Rules of Court.
As earlier explained, unlawful detainer was sufficiently alleged in the Complaint and duly proven during the trial. Significantly, the
issue of whether there was enough ground to eject petitioners was raised during the preliminary conference. 38

Not Merely Tolerated

Possession

Petitioners dispute the lower courts’ finding that they occupied the subject lots on the basis of mere tolerance. They argue that
their occupation was not under such condition, since respondents had invited, offered and persuaded them to use those
properties.39

This Court has consistently held that those who occupy the land of another at the latter’s tolerance or permission, without any
contract between them, are necessarily bound by an implied promise that the occupants will vacate the property upon
demand.40 A summary action for ejectment is the proper remedy to enforce this implied obligation. 41 The unlawful deprivation or
withholding of possession is to be counted from the date of the demand to vacate. 42

Toleration is defined as "the act or practice of permitting or enduring something not wholly approved of." 43 Sarona v.
Villegas44 described what tolerated acts means, in this language:

"Professor Arturo M. Tolentino states that acts merely tolerated are ‘those which by reason of neighborliness or
familiarity, the owner of property allows his neighbor or another person to do on the property; they are generally those
particular services or benefits which one’s property can give to another without material injury or prejudice to the owner,
who permits them out of friendship or courtesy.’ x x x. And, Tolentino continues, even though ‘this is continued for a
long time, no right will be acquired by prescription." x x x. Further expounding on the concept, Tolentino writes: ‘There is
tacit consent of the possessor to the acts which are merely tolerated. Thus, not every case of knowledge and silence on
the part of the possessor can be considered mere tolerance. By virtue of tolerance that is considered as an
authorization, permission or license, acts of possession are realized or performed. The question reduces itself to the
existence or non-existence of the permission." 45

We hold that the facts of the present case rule out the finding of possession by mere tolerance. Petitioners were able to
establish that respondents had invited them to occupy the subject lots in order that they could all live near one other and help in
resolving family problems.46 By occupying those lots, petitioners demonstrated their acceptance of the invitation. Hence, there
was a meeting of minds, and an agreement regarding possession of the lots impliedly arose between the parties.

The occupancy of the subject lots by petitioners was not merely "something not wholly approved of" by respondents. Neither did
it arise from what Tolentino refers to as "neighborliness or familiarity." In point of fact, their possession was upon the invitation of
and with the complete approval of respondents, who desired that their children would occupy the premises. It arose from familial
love and a desire for family solidarity, which are basic Filipino traits.

Right to Use the Lots Terminated

That Ismael and Teresita had a right to occupy the lots is therefore clear. The issue is the duration of possession. In the absence
of a stipulation on this point, Article 1197 of the Civil Code allows the courts to fix the duration or the period.

"Article 1197. If the obligation does not fix a period, but from its nature and the circumstances it can be inferred that a
period was intended, the courts may fix the duration thereof.

"The courts shall also fix the duration of the period when it depends upon the will of the debtor.

"In every case the courts shall determine such period as may under the circumstances have been probably
contemplated by the parties. Once fixed by the courts, the period cannot be changed by them."
CivRev (Property) Assignment 2
Article 1197, however, applies to a situation in which the parties intended a period. Such qualification cannot be inferred from the
facts of the present case.

To repeat, when Vicente and Rosario invited their children to use the lots, they did so out of parental love and a desire for
solidarity expected from Filipino parents. No period was intended by the parties. Their mere failure to fix the duration of their
agreement does not necessarily justify or authorize the courts to do so. 47

Based on respondents’ reasons for gratuitously allowing petitioners to use the lots, it can be safely concluded that the
agreement subsisted as long as the parents and the children mutually benefited from the arrangement. Effectively, there is a
resolutory condition in such an agreement. 48 Thus, when a change in the condition existing between the parties occurs -- like a
change of ownership, necessity, death of either party or unresolved conflict or animosity -- the agreement may be deemed
terminated. Having been based on parental love, the agreement would end upon the dissipation of the affection.

When persistent conflict and animosity overtook the love and solidarity between the parents and the children, the purpose of the
agreement ceased.49 Thus, petitioners no longer had any cause for continued possession of the lots. Their right to use the
properties became untenable. It ceased upon their receipt of the notice to vacate. And because they refused to heed the
demand, ejectment was the proper remedy against them. Their possession, which was originally lawful, became unlawful when
the reason therefor -- love and solidarity -- ceased to exist between them.

No Right to Retain

Possession

Petitioners have not given this Court adequate reasons to reverse the lower courts’ dismissal of their contention that Lots T-
78521 and T-103141, respectively, were allegedly allotted to them as part of their inheritance and given in consideration for past
debts.

The right of petitioners to inherit from their parents is merely inchoate and is vested only upon the latters’ demise. Indisputably,
rights of succession are transmitted only from the moment of death of the decedent. 50 Assuming that there was an "allotment" of
inheritance, ownership nonetheless remained with respondents. Moreover, an intention to confer title to certain persons in the
future is not inconsistent with the owners’ taking back possession in the meantime for any reason deemed sufficient. 51 Other
than their self-serving testimonies and their affidavits, petitioners offered no credible evidence to support their outlandish claim of
inheritance "allocation."

We also agree with the lower courts that petitioners failed to prove the allegation that, through a dation in payment, Lot T-78521
had been transferred to the latter as payment for respondents’ debts. 52 The evidence presented by petitioners related only to the
alleged indebtedness of the parents arising from the latter’s purported purchases and advances. 53 There was no sufficient proof
that respondents had entered into a contract of dation to settle the alleged debt. Petitioners even stated that there was a
disagreement in the accounting of the purported debt, 54 a fact that disproves a meeting of the minds with the parents.

Petitioners also admitted that a portion of the alleged debt is the subject matter of a collection case against respondents (Civil
Case No. 0594-96).55 Thus, the former’s allegation that the indebtedness has been paid through a dation cannot be given
credence, inconsistent as it is with their action to recover the same debt.

Despite their protestations, petitioners recognized the right of the parents to recover the premises when they admitted in their
Position Paper filed with the MTCC that respondents had a title to the lots.

"The [respondents] want to get their property because the title is theirs, the [petitioners] do not object but what is due
the [petitioners] including the reparation for the tarnish of their dignity and honor must be given the [petitioners] for the
benefits of their children before the premises will be turned over." 56

As a rule, the right of ownership carries with it the right of possession.

Second Issue:

Appearance at the Preliminary Conference

Section 8 of Rule 70 of the Rules of Court requires the appearance of the plaintiff and the defendant during the preliminary
conference. On the basis of this provision, petitioners claim that the MTCC should have dismissed the case upon the failure of
respondents to attend the conference. However, petitioners do not dispute that an attorney-in-fact with a written authorization
from respondents appeared during the preliminary conference. 57 The issue then is whether the rules on ejectment allow a
representative to substitute for a party’s personal appearance.

Unless inconsistent with Rule 70, the provisions of Rule 18 on pretrial applies to the preliminary conference. 58 Under Section 4 of
this Rule, the nonappearance of a party may be excused by the showing of a valid cause; or by the appearance of a
representative, who has been fully authorized in writing to enter into an amicable settlement, to submit to alternative modes of
dispute resolution, and to enter into stipulations or admissions of facts and of documents. 59

Section 4 of Rule 18 may supplement Section 8 of Rule 70. Thus, the spirit behind the exception to personal appearance under
the rules on pretrial is applicable to the preliminary conference. If there are valid reasons or if a representative has a "special
authority," a party’s appearance may be waived. As petitioners are challenging only the applicability of the rules on pretrial to the
rule on preliminary conference, the written authorization from respondents can indeed be readily considered as a "special
authorization."
CivRev (Property) Assignment 2
Third Issue:

Rights of a Builder in Good Faith

As applied to the present case, accession refers to the right of the owner to everything that is incorporated or attached to the
property.60 Accession industrial -- building, planting and sowing on an immovable -- is governed by Articles 445 to 456 of the
Civil Code.

Articles 447 and 1678 of the

Civil Code Inapplicable

To buttress their claim of reimbursement for the improvements introduced on the property, petitioners cite Article 447. 61 They
allege that the CA erred in applying Article 1678, since they had no lease agreement with respondents.

We clarify. Article 447 is not applicable, because it relates to the rules that apply when the owner of the property uses the
materials of another. It does not refer to the instance when a possessor builds on the property of another, which is the factual
milieu here.

In view of the unique factual setting of the instant case, the contention of petitioners regarding the inapplicability of Article 1678
deserves attention. The CA applied the provisions on lease, because it found their possession by mere tolerance comparable
with that of a lessee, per the pronouncement in Calubayan v. Pascual, 62 from which we quote:

"x x x. It has been held that a person who occupies the land of another at the latter’s tolerance or permission, without
any contract between them, is necessarily bound by an implied promise that he will vacate upon demand, failing which
a summary action for ejectment is the proper remedy against them. The status of defendant is analogous to that of a
lessee or tenant whose term of lease has expired but whose occupancy continued by tolerance of the owner. In such a
case, the unlawful deprivation or withholding of possession is to be counted from the date of the demand to
vacate."63 (Emphasis in the original.)

As explained earlier, Ismael and Teresita’s possession of the two lots was not by mere tolerance, a circumstance that negates
the applicability of Calubayan.

Article 448 Applicable

On the other hand, when a person builds in good faith on the land of another, the applicable provision is Article 448, which
reads:64

"Article 448. The owner of the land on which anything has been built, sown or planted in good faith, shall have the right
to appropriate as his own the works, sowing or planting, after payment of the indemnity provided for in Articles 546 and
548, or to oblige the one who built or planted to pay the price of the land, and the one who sowed, the proper rent.
However, the builder or planter cannot be obliged to buy the land if its value is considerably more than that of the
building or trees. In such case, he shall pay reasonable rent, if the owner of the land does not choose to appropriate the
building or trees after proper indemnity. The parties shall agree upon the terms of the lease and in case of
disagreement, the court shall fix the terms thereof."

This Court has ruled that this provision covers only cases in which the builders, sowers or planters believe themselves to be
owners of the land or, at least, to have a claim of title thereto.65 It does not apply when the interest is merely that of a holder,
such as a mere tenant, agent or usufructuary. 66 From these pronouncements, good faith is identified by the belief that the land is
owned; or that -- by some title -- one has the right to build, plant, or sow thereon. 67

However, in some special cases, this Court has used Article 448 by recognizing good faith beyond this limited definition. Thus, in
Del Campo v. Abesia,68 this provision was applied to one whose house -- despite having been built at the time he was still co-
owner -- overlapped with the land of another. 69 This article was also applied to cases wherein a builder had constructed
improvements with the consent of the owner. The Court ruled that the law deemed the builder to be in good faith. 70 In Sarmiento
v. Agana,71 the builders were found to be in good faith despite their reliance on the consent of another, whom they had
mistakenly believed to be the owner of the land.72

Based on the aforecited special cases, Article 448 applies to the present factual milieu. The established facts of this case show
that respondents fully consented to the improvements introduced by petitioners. In fact, because the children occupied the lots
upon their invitation, the parents certainly knew and approved of the construction of the improvements introduced
thereon.73 Thus, petitioners may be deemed to have been in good faith when they built the structures on those lots.

The instant case is factually similar to Javier v. Javier.74 In that case, this Court deemed the son to be in good faith for building
the improvement (the house) with the knowledge and consent of his father, to whom belonged the land upon which it was built.
Thus, Article 44875 was applied.

Rule on Useful Expenses

The structures built by petitioners were "useful" improvements, because they augmented the value or income of the bare
lots.76 Thus, the indemnity to be paid by respondents under Article 448 is provided for by Article 546, which we quote:
CivRev (Property) Assignment 2
"Art. 546. Necessary expenses shall be refunded to every possessor; but only the possessor in good faith may retain
the thing until he has been reimbursed therefor.

"Useful expenses shall be refunded only to the possessor in good faith with the same right of retention, the person who
has defeated him in the possession having the option of refunding the amount of the expenses or of paying the increase
in value which the thing may have acquired by reason thereof."

Consequently, respondents have the right to appropriate -- as their own -- the building and other improvements on the subject
lots, but only after (1) refunding the expenses of petitioners or (2) paying the increase in value acquired by the properties by
reason thereof. They have the option to oblige petitioners to pay the price of the land, unless its value is considerably more than
that of the structures -- in which case, petitioners shall pay reasonable rent.

In accordance with Depra v. Dumlao,77 this case must be remanded to the trial court to determine matters necessary for the
proper application of Article 448 in relation to Article 546. Such matters include the option that respondents would take and the
amount of indemnity that they would pay, should they decide to appropriate the improvements on the lots. We disagree with the
CA’s computation of useful expenses, which were based only on petitioners’ bare allegations in their Answer. 78

Ruling on Improvement Justified

While, ordinarily, the jurisdiction of the MTCC on ejectment proceedings is limited to the issue of physical or material possession
of the property in question, this Court finds it necessary to abbreviate the issue on the improvements in relation to Article 448.
First, the determination of the parties’ right to those improvements is intimately connected with the MTCC proceedings in the
light of the ejectment of petitioners. Second, there is no dispute that while they constructed the improvements, respondents
owned the land. Third, both parties raised no objection when the RTC and the CA ruled accordingly on this matter.

Equitable considerations compel us to settle this point immediately, pro hoc vice, to avoid needless delay. Both parties have
already been heard on this issue; to dillydally or equivocate would not serve the cause of substantial justice.

Other Issues Raised

Given the foregoing rulings, it is no longer necessary to address petitioners’ allegation that the MTCC judge and respondents’
lawyers should be respectively held personally accountable for the Decision and for filing the case. 79 The insinuation of
petitioners that the lawyers manipulated the issuance of a false barangay certification is unavailing. 80 Their contention that
respondents did not attend the barangay conciliation proceedings was based solely on hearsay, which has little or no probative
value.81

WHEREFORE, the assailed Decision and Resolution of the Court of Appeals are AFFIRMED with the
following MODIFICATIONS:

1. The portion requiring Spouses Vicente and Rosario Macasaet to reimburse one half of the value of the useful
improvements, amounting to ₱475,000, and the right of Spouses Ismael and Rosita Macasaet to remove those
improvements (if the former refuses to reimburse) is DELETED.

2. The case is REMANDED to the court of origin for further proceedings to determine the facts essential to the proper
application of Articles 448 and 546 of the Civil Code, specifically to the following matters:

a. Spouses Vicente and Rosario Macasaet’s option to appropriate -- as their own -- the improvements on the
lots, after paying the indemnity, as provided under Article 546 in relation to Article 448 of the Civil Code; or in
requiring Spouses Ismael and Rosita Macasaet to pay for the value of the lots, unless it is considerably more
than that of the improvements, in which case petitioners shall pay reasonable rent based upon the terms
provided under the Civil Code

b. The value of the useful expenses incurred by Spouses Ismael and Rosita Macasaet in the construction of the
improvements on the lots

c. The increase in value acquired by the lots by reason of the useful improvements

d. Spouses Vicente and Rosario Macasaet’s choice of type of indemnity to be paid (whether b or c)

e. Whether the value of the lots is considerably more than that of the improvements built thereon

No pronouncement as to costs.

SO ORDERED.
CivRev (Property) Assignment 2
SECOND DIVISION

July 3, 2017

G.R. No. 211170

SPOUSES MAXIMO ESPINOZA and WINIFREDA DE VERA, Petitioners


vs.
SPOUSES ANTONIO MAYANDOC and ERLINDA CAYABYAB MAYANDOC, Respondents

DECISION

PERALTA, J.:

Before this Court is the Petition for Review on Certiorari  under Rule 45, dated March 21, 2014, of petitioners-spouses Maximo
Espinoza and Winifreda De Vera, that seeks to reverse and set aside the Decision 1 dated September 17, 2013 and Resolution
dated January 28, 2014, both of the Court of Appeals (CA) which, in turn, affirmed with modifications the Decision2 dated
February 18, 2011 of the Regional Trial Court (RTC), Branch 42, Dagupan City, in a complaint for useful expenses under
Articles 4483 and 5464 of the New Civil Code of the Philippines.

The facts follow.

A parcel of land located in Dagupan City was originally owned by Eusebio Espinoza. After the death of Eusebio, the said parcel
of land was divided among his heirs, namely: Pastora Espinoza, Domingo Espinoza and Pablo Espinoza. Petitioner Maximo is
the son of Domingo Espinoza, who died on November 3, 1965, and Agapita Cayabyab, who died on August 11, 1963.

Thereafter, on May 25, 1972, Pastora Espinoza executed a Deed of Sale conveying her share of the same property to
respondents and Leopoldo Espinoza. However, on that same date, a fictitious deed of sale was executed by petitioner Maximo's
father, Domingo Espinoza, conveying the three-fourth (3/4) share in the estate in favor of respondent Erlinda Cayabyab
Mayandoc's parents; thus, TCT No. 28397 was issued in the names of the latter.

On July 9, 1977, a fictitious deed of sale was executed by Nemesio Cayabyab, Candida Cruz, petitioners-spouses Maximo
Espinoza and Winifreda De Vera and Leopoldo Espinoza over the land in favor of respondents- spouses Antonio and Erlinda
Mayandoc; thus, TCT No. 37403 was issued under the names of the latter.

As a result of the foregoing, petitioners filed an action for annulment of document with prayer for the nullification of TCT No.
37403 and, on August 16, 1999, the RTC, Branch 40, Dagupan City rendered a Decision in favor of petitioners and ordering
respondents to reconvey the land in dispute and to pay attorney's fees and the cost of the suit.

Respondents appealed, but the CA, in its Decision dated February 6, 2004, affirmed the RTC with modifications that the award
of attorney's fees and litigation expenses be deleted for lack of factual basis. The said CA Decision became final and executory
on March 8, 2004.

Thus, respondents filed a complaint for reimbursement for useful expenses, pursuant to Articles 448 and 546 of the New Civil
Code, alleging that the house in question was built on the disputed land in good faith sometime in 1995 and was finished in
1996. According to respondents, they then believed themselves to be the owners of the land with a claim of title thereto and
were never prevented by the petitioners in constructing the house. They added that the new house was built after the old house
belonging to respondent Erlinda Mayandoc's father was torn down due to termite infestation and would not have reconstructed
the said house had they been aware of the defect in their title. As such, they claimed that they are entitled to reimbursement of
the construction cost of the house in the amount of ₱800,000.00. They further asserted that at the time that their house was
constructed, they were possessors in good faith, having lived over the land in question for many years and that petitioners
questioned their ownership and possession only in 1997 when a complaint for nullity of documents was filed by the latter.

Petitioners, in their Answer, argued that respondents can never be considered as builders in good faith because the latter were
aware that the deeds of sale over the land in question were fictitious and, therefore, null and void; thus, as builders in bad faith,
they lose whatever has been built over the land without right to indemnity.

Respondents, on January 5, 2011, manifested their option to buy the land where the house stood, but petitioners expressed that
they were not interested to sell the land or to buy the house in question.

The RTC, on February 18, 2011, rendered its Decision with the following dispositive portion:

WHEREFORE, judgment is hereby rendered requiring the defendants to sell the land, where the plaintiffs' house stands, to the
latter at a reasonable price based on the zonal value determined by the Bureau of Internal Revenue (BIR).

SO ORDERED.5

Petitioners appealed to the CA, but the latter, in its Decision dated September 17, 2013, affirmed the decision of the RTC with
modifications. The dispositive portion of the Decision reads:

WHEREFORE, the Decision dated February 18, 2011 by the Regional Trial Court, Branch 42 of Dagupan City, in Civil Case No.
2005- 0271-D is hereby AFFIRMED with MODIFICATIONS.
CivRev (Property) Assignment 2
Let the case be REMANDED to the aforementioned trial court for further proceedings consistent with the proper application of
Articles 448, 546 and 548 of the New Civil Code and to render a complete judgment of the case.

SO ORDERED.6

The motion for reconsideration of petitioners were subsequently denied by the CA in its Resolution dated January 28, 2014.

Hence, the present petition.

Petitioners raise the following issues:

I.

WHETHER THE HONORABLE COURT OF APPEALS ERRED IN RULING THAT THE PETITIONERS WERE NOT ABLE TO
PROVE BAD FAITH ON THE PART OF THE RESPONDENTS.

II.

WHETHER OR NOT THE HONORABLE COURT OF APPEALS ERRED IN RULING THAT RES JUDJCATA  DOES NOT
APPLY IN THE INST ANT CASE.

According to petitioners, whether or not respondents were in bad faith in introducing improvements on the subject land is already
moot, since the judgment rendered by the RTC of Dagupan City, Branch 40 and affirmed by the CA, that declared the two
Deeds of Definite/ Absolute Sale dated May 25, 1972 and July 9, 1977 as null and void, had long become final and executory on
March 8, 2004. They also argue that respondents had not successfully shown any right to introduce improvements on the said
land as their claim of laches and acquisitive prescription have been rejected by the CA on appeal; thus, it follows that the
respondents were builders in bad faith because knowing that the land did not belong to them and that they had no right to build
thereon, they still caused the house to be erected. They further insist that respondents are deemed builders in bad faith because
their house has been built and reconstructed into a bigger one after respondent Erlinda's parents forged a fictitious sale. Finally,
they claim that the principle of res judicata in the mode of "conclusiveness of judgment" applies in this case.

The petition lacks merit.

The findings of facts of the Court of Appeals are conclusive and binding on this Court 7 and they carry even more weight when
the said court affirms the factual findings of the trial court. 8 Stated differently, the findings of the Court of Appeals, by itself, which
are supported by substantial evidence, are almost beyond the power of review by this Court. 9 Although this rule is subject to
certain exceptions, this Court finds none that is applicable in this case. Nevertheless, the petition still fails granting that an
exception obtains.

To be deemed a builder in good faith, it is essential that a person asserts title to the land on which he builds, i.e., that he be a
possessor in the concept of owner, and that he be unaware that there exists in his title or mode of acquisition any flaw which
invalidates it.10 The RTC, as affirmed by the CA, found respondents to be builders in good faith, thus:

The plaintiffs are builders in good faith. As asserted by plaintiffs and not rebutted by defendants, the house of plaintiffs was built
on the lot owned by defendants in 1995. The complaint for nullity of documents and reconveyance was filed in 1997, about two
years after the subject conjugal house was constructed. Defendants-spouses believed that at the time when they constructed
their house on the lot of defendants, they have a claim of title. Art. 526, New Civil Code, states that a possessor in good faith is
one who has no knowledge of any flaw or defect in his title or mode of acquisition. This determines whether the builder acted in
good faith or not. Surely, plaintiffs would not have constructed the subject house which plaintiffs claim to have cost them
₱800,000.00 to build if they knew that there is a flaw in their claim of title. Nonetheless, Art. 527, New Civil Code, states clearly
that good faith is always presumed, and upon him who alleges bad faith on the part of the possessor lies the burden of proof.
The records do not show that the burden of proof was successfully discharged by the defendants.

xxxx

Plaintiffs are in good faith in building their conjugal house in 1995 on the lot they believed to be their own by purchase. They also
have in their favor the legal presumption of good faith. It is the defendants who had the burden to prove otherwise. They failed to
discharge such burden until the Regional Trial Court, Br. 40, Dagupan City, promulgated an adverse ruling in Civil Case No. 97-
0187-D. Thus, Art. 448 comes in to protect the plaintiffs-owners of their improvement without causing injustice to the lot owner.
Art. 448 comes in to protect the plaintiff-owners of their improvement without causing injustice to the lot owner. Art. 448 provided
a just resolution of the resulting "forced-ownership" by giving the defendants lot owners the option to acquire the conjugal house
after payment of the proper indemnity or to oblige the builder plaintiffs to pay for the lot. It is the defendants-lot owners who are
authorized to exercise the option as their right is older, and under the principle of accession where the accessory (house) follows
the principal. x x x.11

The settled rule is bad faith should be established by clear and convincing evidence since the law always presumes good
faith.12 In this particular case, petitioners were not able to prove that respondents were in bad faith in constructing the house on
the subject land. Bad faith does not simply connote bad judgment or negligence. 13 It imports a dishonest purpose or some moral
obliquity and conscious doing of a wrong.14 It means breach of a known duty through some motive, interest or ill will that
partakes of the nature of fraud.15 For anyone who claims that someone is in bad faith, the former has the duty to prove such.
Hence, petitioners err in their argument that respondents failed to prove that they are builders in good faith in spite of the
findings of the RTC and the CA that they are.
CivRev (Property) Assignment 2
As such, Article 44816 of the Civil Code must be applied. It applies when the builder believes that he is the owner of the land or
that by some title he has the right to build thereon, 17 or that, at least, he has a claim of title thereto. 18 In Tuatzs v. Spouses Escol,
et al.,19 this Court ruled that the seller (the owner of the land) has two options under Article 448: (1) he may appropriate the
improvements for himself after reimbursing the buyer (the builder in good faith) the necessary and useful expenses under
Articles 54620 and 54821 of the Civil Code; or (2) he may sell the land to the buyer, unless its value is considerably more than that
of the improvements, in which case, the buyer shall pay reasonable rent, thus:

The rule that the choice under Article 448 of the Civil Code belongs to the owner of the land is in accord with the principle of
accession, i.e., that the accessory follows the principal and not the other way around. Even as the option lies with the
landowner, the grant to him, nevertheless, is preclusive. The landowner cannot refuse to exercise either option and compel
instead the owner of the building to remove it from the land.

The raison d'etre  for this provision has been enunciated thus: Where the builder, planter or sower has acted in good faith, a
conflict of rights arises between the owners, and it becomes necessary to protect the owner of the improvements without
causing injustice to the owner of the land. In view of the impracticability of creating a state of forced co-ownership, the law has
provided a just solution by giving the owner of the land the option to acquire the improvements after payment of the proper
indemnity, or to oblige the builder or planter to pay for the land and the sower the proper rent. He cannot refuse to exercise
either option. It is the owner of the land who is authorized to exercise the option, because his right is older, and because, by the
principle of accession, he is entitled to the ownership of the accessory thing. 22

The CA, therefore, did not err in its ruling that instead of requiring the petitioners to sell the land, the R TC must determine the
option which the petitioners would choose.1âwphi1 As aptly ruled by the CA:

The rule that the right of choice belongs to the owner of the land is in accordance with the principle of accession. However, even
if this right of choice is exclusive to the land owner, he cannot refuse to exercise either option and demand, instead for the
removal of the building.

Instead of requiring defendants-appellants to sell the land, the court a quo  must determine the option which they would choose.
The first option to appropriate the building upon payment of indemnity or the second option, to sell the land to the plaintiffs-
appellees. Moreover, the court a quo  should also ascertain: (a) under the first option, the amount of indemnification for the
building; or (b) under the second option, the value of the subject property vis-a-vis that of the building, and depending thereon,
the price of, or the reasonable rent for, the subject prope1iy.

Hence, following the ruling in the recent case of Briones v. Macabagdal,  this case must be remanded to the court a quo for the
conduct of further proceedings to assess the current fair market of the land and to determine other matters necessary for the
proper application of Article 448, in relation to Articles 546 and 548 of the New Civil Code. 23

Therefore, this Court agrees with the CA that there is a need to remand the case to the RTC for further proceedings, specifically,
in assessing the current fair market value of the subject land and other matters that are appropriate in the application of Article
448, in relation to Articles 546 and 548 of the New Civil Code.

As to the issue of res judicata,  the CA is correct in its ruling that there is no identity of subject matter and cause of action
between the prior case of annulment of document and the present case, thus:

In the instant case, res judicata  will not apply since there is no identity of subject matter and cause of action. The first case is for
annulment of document, while the instant case is for reimbursement of useful expenses as builders in good faith under article
448 in relation to Articles 546 and 548 of the New Civil Code.

Moreover, We are not changing or reversing any findings of the RTC and by this Court in Our 6 February 2004 decision. The
Court is still bound by this judgment insofar as it found the Deeds of Absolute Sale null and void, and that defendants-appellants
are the rightful owners of the lot in question.

However, if the court a quo  did not take cognizance of the instant case, plaintiffs-appellees shall lose ownership of the building
worth Php316,400.00 without any compensation. While, the defendant-appellants not only will recover the land but will also
acquire a house without payment of indemnity. The fairness of the rules enunciated in Article 448 is explained by the Supreme
Court in the case of Depra v. Dumlao, viz.:

Where the builder, planter or sower has acted in good faith, a conflict of rights arises between the owners, and it becomes
necessary to protect the owner of the improvements without causing injustice to the owner of the land. In view of the
impracticability of creating a state of forced ownership, the law has provided a just solution by giving the owner of the land the
option to acquire the improvements after payment of the proper indemnity, or to oblige the builder or planter to pay for the land
and the sower to pay the proper rent. It is the owner of the land who is authorized to exercise the option, because his right is
older, and because, by the principle of accession, he is entitled to the ownership of the accessory thing.

Finally, "the decision of the court a quo should not be viewed as a denigration of the doctrine of immutability of final judgments,
but a recognition of the equally sacrosanct doctrine that a person should not be allowed Io profit or enrich himself inequitably at
anal her's expense."24

The well-settled rule is that the principle or rule of res judicata is primarily one of public policy. It is based on the policy against
multiplicity of suits,26 whose primary objective is to avoid unduly burdening the dockets of the courts. 27 In this case, however,
such principle is inapplicable.
CivRev (Property) Assignment 2
WHEREFORE, the Petition for Review on Certiorari  under Rule 45, dated March 21, 2014, of petitioners-spouses Maximo
Espinoza and Winifreda De Vera, is DENIED. Consequently, the Decision dated September 17, 2013 and Resolution dated
January 28, 2014, both of the Court of Appeals are AFFIRMED.

SO ORDERED.
CivRev (Property) Assignment 2
FIRST DIVISION

G.R. No. 146608             October 23, 2003

SPOUSES CONSTANTE FIRME AND AZUCENA E. FIRME, petitioners,


vs.
UKAL ENTERPRISES AND DEVELOPMENT CORPORATION, respondent.

DECISION

CARPIO, J.:

The Case

This is a petition for review on certiorari of the Decision1 dated 3 January 2001 of the Court of Appeals in CA-G.R. CV No.
60747. The Court of Appeals reversed the Decision 2 of the Regional Trial Court, Branch 223, Quezon City ("trial court"), which
held that there was no perfected contract of sale since there was no consent on the part of the seller.

The Facts

Petitioner Spouses Constante and Azucena Firme ("Spouses Firme") are the registered owners of a parcel of land 3 ("Property")
located on Dahlia Avenue, Fairview Park, Quezon City. Renato de Castro ("De Castro"), the vice president of Bukal Enterprises
and Development Corporation ("Bukal Enterprises") authorized his friend, Teodoro Aviles ("Aviles"), a broker, to negotiate with
the Spouses Firme for the purchase of the Property.

On 28 March 1995, Bukal Enterprises filed a complaint for specific performance and damages with the trial court, alleging that
the Spouses Firme reneged on their agreement to sell the Property. The complaint asked the trial court to order the Spouses
Firme to execute the deed of sale and to deliver the title to the Property to Bukal Enterprises upon payment of the agreed
purchase price.

During trial, Bukal Enterprises presented five witnesses, namely, Aviles, De Castro, Antonio Moreno, Jocelyn Napa and Antonio
Ancheta.

Aviles testified that De Castro authorized him to negotiate on behalf of Bukal Enterprises for the purchase of the Property.
According to Aviles, he met with the Spouses Firme on 23 January 1995 and he presented them with a draft deed of sale 4 ("First
Draft") dated February 1995. The First Draft of the deed of sale provides:

DEED OF ABSOLUTE SALE

KNOW ALL MEN BY THESE PRESENTS:

This DEED OF ABSOLUTE SALE made and executed by and between the Spouses CONSTANTE FIRME and AZUCENA E.
FIRME, both of legal age, Filipino citizens and with postal address at No. 1450 Union, Paco, City of Manila, hereinafter called
the VENDOR, and

BUKAL ENTERPRISES and DEVELOPMENT CORPORATION, a corporation duly organized and registered in accordance with
Philippine Laws, with business address at Dahlia Avenue, Fairview Park, Quezon City, herein represented by its PRESIDENT,
MRS. ZENAIDA A. DE CASTRO, hereinafter called the VENDEE.

WITNESSETH:

That the VENDOR is the absolute and registered owner of a certain parcel of land located at Fairview Park, Quezon City, and
more particularly described as follows:

A parcel of land (Lot 4, Block 33 of the consolidation-subdivision plan (LRC) Pcs-8124, Sheet No. I, being a portion of the
consolidation of Lots 41-B-2-A and 41-B-2-C, Psd-1136 and Lot (LRC) Pcs-2665, (LRC) GLRO) Record. No. 1037), situated in
Quezon City, Island of Luzon. Bounded on the NE., points 2 to 5 by Road Lot 24, of the consolidation-subdivision plan.
Beginning at a point marked "1" on plan, being S. 67 deg. 23’W., 9288.80 m. from BLLM I, Mp of Montalban, Rizal; thence N. 85
deg. 35’E., 17.39 m. to point 2; thence S. 54 deg. 22’E., 4.00 m. to point 3; thence S. 14 deg. 21’E., 17.87 m. to point 4; thence 3
deg. 56’E., 17.92 m. to point 5; thence N. 85 deg. 12’ W., 23.38 m. to point 6; thence N. 4 deg. 55’ W., 34.35 m. to the point of
beginning; containing an area of EIGHT HUNDRED AND SIX (806) SQUARE METERS, more or less.

VENDOR’S title thereto being evidenced by Transfer Certificate of Title No. 264243 issued by the Register of Deeds of Quezon
City;

That the VENDOR, for and in consideration of the sum of THREE MILLION TWO HUNDRED TWENTY FOUR THOUSAND
PESOS (₱3,224,000.00) Philippine Currency, to them in hand paid and receipt whereof is hereby acknowledged, do hereby
SELL, TRANSFER and CONVEY unto the said VENDEE, its assigns, transferees and successors in interest the above
described property, free from all liens and encumbrances whatsoever;
CivRev (Property) Assignment 2
It is hereby mutually agreed that the VENDEE shall bear all the expenses for the capital gains tax, documentary stamps,
documentation, notarization, removal and relocation of the squatters, registration, transfer tax and other fees as may be required
by law;

That the VENDOR shall pay the real estate tax for the current year and back real estate taxes, charges and penalties if there are
any.

IN WITNESS WHEREOF, we have hereunto affixed our signatures this ____ day of February, 1995, at Quezon City, Philippines.

CONSTANTE FIRME BUKAL ENTERPRISES AND


DEVELOPMENT CORP.

BY:
AZUCENA E. FIRME ZENAIDA A. DE CASTRO
VENDOR President
xxx

The Spouses Firme rejected this First Draft because of several objectionable conditions, including the payment of capital gains
and other government taxes by the seller and the relocation of the squatters at the seller’s expense. During their second
meeting, Aviles presented to the Spouses Firme another draft deed of sale 5 ("Second Draft") dated March 1995. The Spouses
Firme allegedly accepted the Second Draft in view of the deletion of the objectionable conditions contained in the First Draft.
According to Aviles, the Spouses Firme were willing to sell the Property at ₱4,000 per square meter. They then agreed that
payment would be made at the Far East Bank and Trust Company ("FEBTC"), Padre Faura Branch, Manila. However, the
scheduled payment had to be postponed due to problems in the transfer of funds. The Spouses Firme later informed Aviles that
they were no longer interested in selling the Property. 6

De Castro testified that he authorized Aviles to negotiate for Bukal Enterprises the purchase of the Property owned by the
Spouses Firme. The Property was located beside the Dahlia Commercial Complex owned by Bukal Enterprises. Aviles informed
him that the Spouses Firme agreed to sell the Property at ₱4,000 per square meter, payable in cash for a lump sum of
₱3,224,000. Furthermore, Bukal Enterprises agreed to pay the taxes due and to undertake the relocation of the squatters on the
Property. For this purpose, Bukal Enterprises applied for a loan of ₱4,500,000 which FEBTC granted. Bukal Enterprises then
relocated the four families squatting on the Property at a cost of ₱60,000 per family. After the squatters vacated the Property,
Bukal Enterprises fenced the area, covered it with filling materials, and constructed posts and riprap. Bukal Enterprises spent
approximately ₱300,000 for these improvements. In a letter 7 dated 7 March 1995, Bukal Enterprises offered to pay the purchase
price of ₱3,224,000 to the Spouses Firme upon execution of the transfer documents and delivery of the owner’s duplicate copy
of TCT No. 264243. The Spouses Firme did not accept this offer but instead sent Bukal Enterprises a letter demanding that its
workers vacate the Property. Bukal Enterprises then filed a complaint for specific performance and damages. 8

Antonio Moreno, one of the alleged squatters on the Property, testified that he constructed his house on the Property sometime
in 1982. On 26 February 1995, he was summoned together with the other squatters to a meeting with Aviles regarding their
relocation. They agreed to relocate provided they would be given financial assistance of ₱60,000 per family. Thus, on 6 March
1995, the squatter families were each paid ₱60,000 in the presence of De Castro and Aviles. Thereafter, they voluntarily
demolished their houses and vacated the Property. 9

Jocelyn Mapa, the manager of FEBTC, Padre Faura Branch, testified that Bukal Enterprises has been their client since 1994.
According to her, Bukal Enterprises applied for a loan of ₱4,500,000 on the third week of February 1995 allegedly to buy a lot in
Fairview. FEBTC approved the loan on the last week of February and released the proceeds on the first week of March. 10

Antonio Ancheta ("Ancheta"), barangay captain of Barangay Fairview, testified that he was present when one of the officers of
Bukal Enterprises, a certain Renato, paid each of the four squatter families around ₱60,000 to ₱100,000. Ancheta informed Dr.
Constante Firme that he told the squatters to leave considering that they already received payment for their relocation.
According to Ancheta, Dr. Constante Firme must have misunderstood him and thought that the squatters left through Ancheta’s
own efforts.11

On the other hand, Dr. Constante Firme ("Dr. Firme") was the sole witness for the defendant spouses.

Dr. Firme testified that on 30 January 1995, he and his wife met with Aviles at the Aristocrat Restaurant in Quezon City. Aviles
arranged the meeting with the Spouses Firme involving their Property in Fairview. Aviles offered to buy the Property at ₱2,500
per square meter. The Spouses Firme did not accept the offer because they were reserving the Property for their children. On 6
February 1995, the Spouses Firme met again with Aviles upon the latter’s insistence. Aviles showed the Spouses Firme a copy
of a draft deed of sale12 ("Third Draft") which Aviles prepared. The Third Draft of the deed of sale provides:

CONRACT OF SALE

KNOW ALL MEN BY THESE PRESENTS:

This AGREEMENT, executed this ___ day of February, 1995, by and between the Spouses CONSTANTE FIRME and
AZUCENA E. FIRME, both of legal age, Filipino citizen and with postal address at __________, Quezon City, hereinafter
referred to as the VENDORS, and BUKAL ENTERPRISES and DEVELOPMENT CORPORATION, a corporation duly organized
and registered in accordance with Philippine Laws, with postal address at Fairview Park, Quezon City, herein represented by its
President and Chief Executive Officer, hereinafter referred to as the VENDEE.
CivRev (Property) Assignment 2
WITNESSETH:

That for and in consideration of the sum of THREE MILLION TWO HUNDRED TWENTY FOUR THOUSAND PESOS
(₱3,224,000.00), Philippine Currency, payable in the form hereinafter expressed, agreed to sell to the VENDEE and the
VENDEE has agreed to buy from the VENDORS, a parcel of land situated at Dahlia Avenue corner Rolex Street, Fairview Park,
Quezon City, containing an area of 806 Square Meters more or less, of which the VENDORS are the absolute registered owners
in accordance with the Land Registration Act, as evidenced by Transfer Certificate of Title No. 264243 issued by the Register of
Deeds of Quezon City, more particularly described and bounded as follows:

(DESCRIPTION AND BOUNDARIES OF PROPERTY)

THE FURTHER TERMS AND CONDITIONS OF THE CONTRACT ARE AS FOLLOWS:

1. The VENDEE agrees to pay the VENDORS upon execution of this Contract the sum of ONE MILLION
PESOS (₱1,000,000.00), Philippine Currency, as downpayment and agrees to pay the balance of TWO
MILLION TWO HUNDRED TWENTY FOUR THOUSAND PESOS (₱2,224,000.00) at the post office address of
the VENDORS in Quezon City, or such other place or Office as the VENDORS may designate within a period
of sixty (60) days counted from the date of this Contract;

2. The VENDORS have hereunto authorized the VENDEE to mortgage the property and submit this Contract,
together with a certified true copy of the TCT, Tax Declaration, Tax Clearance and Vicinity/Lot Plan, with their
Lending Bank. The proceeds of the VENDEE’S Loan shall directly be paid and remitted by the Bank to the
VENDORS;

3. The said parcel of land shall remain in the name of the VENDORS until the Lending Bank of the VENDEE
shall have issued a Letter Guaranty Payment in favor of the VENDORS, at which time the VENDORS agree to
execute a Deed of Absolute Sale in favor of the VENDEE and cause the issuance of the Certificate of Title in
the name of the latter. The Capital Gains Tax and Documentary Stamps shall be charged from the VENDORS
in accordance with law;1awphi1.nét

4. The payment of the balance of ₱2,224,000.00 by the VENDEE to the VENDORS shall be within a period of
sixty (60) days effective from the date of this Contract. After the lapse of 60 days and the loan has not yet been
released due to fortuitous events the VENDEE shall pay an interest of the balance a monthly interest based on
existing bank rate until said fortuitous event is no longer present;

5. The VENDEE shall remove and relocate the Squatters, however, such actual, reasonable and necessary
expenses shall be charged to the VENDORS upon presentation of receipts and documents to support the act;

6. The VENDEE shall be allowed for all legal purposes to take possession of the parcel of land after the
execution of this Contract and payment of the downpayment;

7. The VENDEE shall shoulder all expenses like the documentation, registration, transfer tax and relocation of
the property.

IN WITNESS WHEREOF, we have hereunto affixed our signatures this ____ day of February, 1995, at Quezon City, Philippines.

CONSTANTE E. FIRME BUKAL ENTERPRISES DEV. CORP.


VENDOR VENDEE
BY:
AZUCENA E. FIRME ________________________
VENDOR President & Chief Executive Officer
xxx

The Spouses Firme did not accept the Third Draft because they found its provisions one-sided. The Spouses Firme particularly
opposed the provision on the delivery of the Property’s title to Bukal Enterprises for the latter to obtain a loan from the bank and
use the proceeds to pay for the Property. The Spouses Firme repeatedly told Aviles that the Property was not for sale when
Aviles called on 2 and 4 March 1995 regarding the Property. On 6 March 1995, the Spouses Firme visited their Property and
discovered that there was a hollow block fence on one side, concrete posts on another side and bunkers occupied by workers of
a certain Florante de Castro. On 11 March 1995, Spouses Firme visited the Property again with a surveyor. Dr. Firme talked with
Ancheta who told him that the squatters had voluntarily demolished their shanties. The Spouses Firme sent a letter 13 dated 20
March 1995 to Bukal Enterprises demanding removal of the bunkers and vacation by the occupants of the Property. On 22
March 1995, the Spouses Firme received a letter 14 dated 7 March 1995 from Bukal Enterprises demanding that they sell the
Property.15

On 7 August 1998, the trial court rendered judgment against Bukal Enterprises as follows:

WHEREFORE, in the light of the foregoing premises, the above-entitled case [is] hereby DISMISSED and plaintiff BUKAL
ENTERPRISES DEVELOPMENT CORPORATION is hereby ordered to pay the defendants Spouses Constante and Azucena
Firme:
CivRev (Property) Assignment 2
1. the sum of Three Hundred Thirty Five Thousand Nine Hundred Sixty Four and 90/100 (₱335,964.90) as and by way
of actual and compensatory damages;

2. the sum of Five Hundred Thousand Pesos (₱500,000.00) as and by way of moral damages;

3. the sum of One Hundred Thousand Pesos (₱100,000.00) as and by way of attorney’s fees; and

4. the costs of the suit.

SO ORDERED.16

Bukal Enterprises appealed to the Court of Appeals, which reversed and set aside the decision of the trial court. The dispositive
portion of the decision reads:

WHEREFORE, premises considered, the Decision, dated August 7, 1998, is hereby REVERSED and SET ASIDE. The
complaint is granted and the appellees are directed to henceforth execute the Deed of Absolute Sale transferring the ownership
of the subject property to the appellant immediately upon receipt of the purchase price of ₱3,224,000.00 and to perform all such
acts necessary and proper to effect the transfer of the property covered by TCT No. 264243 to appellant. Appellant is directed to
deliver the payment of the purchase price of the property within sixty days from the finality of this judgment. Costs against
appellees.

SO ORDERED.17

Hence, the instant petition.1a\^/phi1.net

The Ruling of the Trial Court

The trial court held there was no perfected contract of sale. Bukal Enterprises failed to establish that the Spouses Firme gave
their consent to the sale of the Property. The parties did not go beyond the negotiation stage and there was no evidence of
meeting of the minds between the parties. Furthermore, Aviles had no valid authority to bind Bukal Enterprises in the sale
transaction. Under Sections 23 and 36 (No. 7) of the Corporation Code, the corporate power to purchase a specific property is
exercised by the Board of Directors of the corporation. Without an authorization from the Board of Directors, Aviles could not
validly finalize the purchase of the Property on behalf of Bukal Enterprises. There is no basis to apply the Statute of Frauds
since there was no perfected contract of sale.

The Ruling of the Court of Appeals

The Court of Appeals held that the lack of a board resolution authorizing Aviles to act on behalf of Bukal Enterprises in the
purchase of the Property was cured by ratification. Bukal Enterprises ratified the purchase when it filed the complaint for the
enforcement of the sale.

The Court of Appeals also held there was a perfected contract of sale. The appellate court ruled that the Spouses Firme
revealed their intent to sell the Property when they met with Aviles twice. The Spouses Firme rejected the First Draft because
they considered the terms unacceptable. When Aviles presented the Second Draft without the objectionable provisions, the
Spouses Firme no longer had any cause for refusing to sell the Property. On the other hand, the acts of Bukal Enterprises in
fencing the Property, constructing posts, relocating the squatters and obtaining a loan to purchase the Property are
circumstances supporting their claim that there was a perfected contract of sale.

The Spouses Firme allowed Bukal Enterprises to exercise acts of ownership over the Property when the latter introduced
improvements on the Property and evicted the squatters. These acts constitute partial performance of the contract of sale that
takes the oral contract out of the scope of the Statute of Frauds.

The Issues

The Spouses Firme raise the following issues:

1. WHETHER THE COURT OF APPEALS ERRED IN FINDING THAT THERE WAS A PERFECTED CONTRACT OF
SALE BETWEEN PETITIONERS AND RESPONDENT DESPITE THE ADDUCED EVIDENCE PATENTLY TO THE
CONTRARY;

2. WHETHER THE COURT OF APPEALS ERRED IN NOT FINDING THAT THE ALLEGED CONTRACT OF SALE IS
ENFORCEABLE DESPITE THE FACT THAT THE SAME IS COVERED BY THE STATUTE OF FRAUDS;

3. WHETHER THE COURT OF APPEALS ERRED IN DISREGARDING THE FACT THAT IT WAS NOT LEGALLY
AND FACTUALLY POSSIBLE FOR RESPONDENT TO PERFECT A CONTRACT OF SALE; AND

4. THE COURT OF APPEALS ERRED IN RULING THAT THE AWARD BY THE TRIAL COURT OF MORAL AND
COMPENSATORY DAMAGES TO PETITIONERS IS IMPROPER. 18

The Ruling of the Court


CivRev (Property) Assignment 2
The petition is meritorious.

The fundamental question for resolution is whether there was a perfected contract of sale between the Spouses Firme and
Bukal Enterprises. This requires a review of the factual and legal issues of this case. As a rule, only questions of law are
appealable to this Court under Rule 4519 of the Rules of Civil Procedure. The findings of fact by the Court of Appeals are
generally conclusive and binding on the parties and are not reviewable by this Court. 20 However, when the factual findings of the
Court of Appeals are contrary to those of the trial court or when the inference made is manifestly mistaken, this Court has the
authority to review the findings of fact.21 Likewise, this Court may review findings of fact when the judgment of the Court of
Appeals is premised on a misapprehension of facts.22 This is the situation in this case.

Whether there was a perfected contract of sale

We agree with the finding of the trial court that there was no perfected contract of sale. Clearly, the Court of Appeals
misapprehended the facts of the case in ruling otherwise.

First, the records indubitably show that there was no consent on the part of the Spouses Firme. Aviles did not present any draft
deed of sale during his first meeting with the Spouses Firme on 30 January 1995. 23 Dr. Firme was consistent in his testimony that
he and his wife rejected the provisions of the Third Draft presented by Aviles during their second meeting on 6 February 1995.
The Spouses Firme found the terms and conditions unacceptable and told Aviles that they would not sell the property. 24 Aviles
showed them only one draft deed of sale (Third Draft) during their second and last meeting on 6 February 1995. 25 When shown a
copy of the First Draft, Dr. Firme testified that it was not the deed of sale shown to them by Aviles during their second
meeting26 and that the Third Draft was completely different from the First Draft. 27

On the other hand, Aviles gave conflicting testimony as to what transpired during the two meetings with the Spouses Firme. In
his direct examination, Aviles testified that during his first meeting with the Spouses Firme on 23 January 1995, he showed them
the First Draft which the Spouses Firme rejected. 28 On their second meeting, Aviles showed the Spouses Firme the Second
Draft, which the Spouses Firme allegedly approved because the objectionable conditions contained in the First Draft were
already deleted. However, a perusal of the First Draft and the Second Draft would show that both deeds of sale contain exactly
the same provisions. The only difference is that the date of the First Draft is February 1995 while that of the Second Draft is
March 1995.

When Aviles testified again as rebuttal witness, his testimony became more confusing. Aviles testified that during his first
meeting with the Spouses Firme on 30 January 1995, he showed them the Third Draft, which was not acceptable to the
latter.29 However, upon further questioning by his counsel, Aviles concurred with Dr. Firme’s testimony that he presented the
Third Draft (Exh. "5"; Exh. "L") to the Spouses Firme only during their second meeting. He also stated that he prepared and
presented to the Spouses Firme the First Draft (Exh. "C") and the Second Draft (Exh. "C-1") during their first or second meeting.
He testified:

ATTY. MARQUEDA:

Q: On page 11 of the tsn dated August 5, 1997 a question was posed "How did you find this draft the Contract of Sale
which was presented to you by Mr. Aviles on the second meeting?" The answer is "On the first meeting(sic), we find it
totally unacceptable, sir."30 What can you say on this? Before that, Mr. Witness, what is this Contract of Sale that you
presented to Mr. Aviles on the second meeting? Is this different from the Contract of Sale that was marked as Exhibit
"5-L"?

Q: May I see the document Exhibit 5 – L?31

INTERPRETER:

Witness going over the record.

ATTY. MARQUEDA:

Q: Is that the same document that was presented by you to Mr. Firme on the second meeting or there is a different
contract?

A: This is the same document – draft of the document that I submitted to them during our second meeting. That was
February. This was the draft.

Q: What about Exhibit C and C-1 [which] were identified by you. When was this presented to Dr. Firme?

A: This is the same.

Q: Exhibit C and C-1?

A: Yes because I prepared two documents during our meeting. One already with notarial, the one without notarial page
and the other one with notarial page already, so I prepared two documents but with the same contents both were dated
February of 1995.32

Q: So, you are referring now to Exhibit C and C-1 for the plaintiff?
CivRev (Property) Assignment 2
A: C-1 is already in the final form because we agreed already as to the date of the payment, so I prepared already
another document which is dated March 1995.33 (Emphasis supplied)

In his cross-examination, Aviles again changed his testimony. According to him, he presented the Third Draft to the Spouses
Firme during their first meeting.34 However, when he went over the records, he again changed his answer and stated that he
presented the Third Draft during their second meeting. 35

In his re-direct examination, Aviles gave another version of what he presented to the Spouses Firme during the two meetings.
According to him, he presented the Third Draft during the first meeting. On their second meeting, he presented the First and the
Second Drafts to the Spouses Firme.36

Furthermore, Aviles admitted that the first proposal of Bukal Enterprises was at ₱2,500 per square meter for the Property. 37 But
the First, Second and Third Drafts of the deed of sale prepared by Aviles all indicated a purchase price of ₱4,000 per square
meter or a lump sum of ₱3,224,000 (₱4,000 per sq.m. x 806 sq.m. = ₱3,224,000) for the Property. Hence, Aviles could not have
presented any of these draft deeds of sale to the Spouses Firme during their first meeting.

Considering the glaring inconsistencies in Aviles’ testimony, it was proper for the trial court to give more credence to the
testimony of Dr. Firme.

Even after the two meetings with Aviles, the Spouses Firme were firm in their decision not to sell the Property. Aviles called the
Spouses Firme twice after their last meeting. The Spouses Firme informed Aviles that they were not selling the Property. 38 Aviles
himself admitted this during his testimony, thus:

Q. Now, the next question which states: "But did you not have any occasion to talk to him after that second meeting?"
and the answer of Dr. Firme is "He called up a month after, that’s March 2, 1995." What can you say on this?

A. I called him to inform him that the loan was already transferred from Makati to Padre Faura Branch of the Far East
Bank, so I scheduled already the payment of their property.

Q. When?

A. On March 4, 1995.

Q. And then the next question which also states: "What did you talked (sic) about over the telephone?" The answer of
Dr. Firme was "When I found out that he was calling, I told him that the property is not for sale." What can you say on
this?

A. He mentioned that they are no longer interested to sell their property, perhaps they would like a higher price of the
property. They did not mention to me. I do not know what was their reason.

Q. The next question "So, what happened next?" The answer is "He called up two days later, March 4 and my wife
answered the telephone and told him that the property is not for sale, sir." What can you say on this?

A. That is true. That is what Mrs. Firme told me during our conversation on the telephone that they are no longer
interested to sell the property for obvious reason.

Q. When was that?

A. March 4, 1995, your honor.39 (Emphasis supplied)

Significantly, De Castro also admitted that he was aware of the Spouses Firme’s refusal to sell the Property. 40

The confusing testimony of Aviles taken together with De Castro’s admission that he was aware of the Spouses Firme’s refusal
to sell the Property reinforces Dr. Firme’s testimony that he and his wife never consented to sell the Property.

Consent is one of the essential elements of a valid contract. The Civil Code provides:

Art. 1318. There is no contract unless the following requisites concur:

1. Consent of the contracting parties;

2. Object certain which is the subject matter of the contract;

3. Cause of the obligation which is established.

The absence of any of these essential elements will negate the existence of a perfected contract of sale. 41 Thus, where there is
want of consent, the contract is non-existent. 42 As held in Salonga, et al. v. Farrales, et al.: 43

It is elementary that consent is an essential element for the existence of a contract, and where it is wanting, the contract is non-
existent. The essence of consent is the conformity of the parties on the terms of the contract, the acceptance by one of
CivRev (Property) Assignment 2
the offer made by the other. The contract to sell is a bilateral contract. Where there is merely an offer by one party, without the
acceptance of the other, there is no consent. (Emphasis supplied)

In this case, the Spouses Firme flatly rejected the offer of Aviles to buy the Property on behalf of Bukal Enterprises. There was
therefore no concurrence of the offer and the acceptance on the subject matter, consideration and terms of payment as would
result in a perfected contract of sale.44 Under Article 1475 of the Civil Code, the contract of sale is perfected at the moment there
is a meeting of minds on the thing which is the object of the contract and on the price.

Another piece of evidence which supports the contention of the Spouses Firme that they did not consent to the contract of sale
is the fact they never signed any deed of sale. If the Spouses Firme were already agreeable to the offer of Bukal Enterprises as
embodied in the Second Draft, then the Spouses Firme could have simply affixed their signatures on the deed of sale, but they
did not.

Even the existence of a signed document purporting to be a contract of sale does not preclude a finding that the contract is
invalid when the evidence shows that there was no meeting of the minds between the seller and buyer. 45 In this case, what were
offered in evidence were mere unsigned deeds of sale which have no probative value. 46 Bukal Enterprises failed to show the
existence of a perfected contract of sale by competent proof.1ªvvphi1.nét

Second, there was no approval from the Board of Directors of Bukal Enterprises as would finalize any transaction with the
Spouses Firme. Aviles did not have the proper authority to negotiate for Bukal Enterprises. Aviles testified that his friend, De
Castro, had asked him to negotiate with the Spouses Firme to buy the Property. 47 De Castro, as Bukal Enterprises’ vice
president, testified that he authorized Aviles to buy the Property. 48 However, there is no Board Resolution authorizing Aviles to
negotiate and purchase the Property on behalf of Bukal Enterprises. 49

It is the board of directors or trustees which exercises almost all the corporate powers in a corporation. Thus, the Corporation
Code provides:

SEC. 23. The board of directors or trustees. — Unless otherwise provided in this Code, the corporate powers of all corporations
formed under this Code shall be exercised, all business conducted and all property of such corporations controlled and held by
the board of directors or trustees to be elected from among the holders of stock, or where there is no stock, from among the
members of the corporation, who shall hold office for one (1) year and until their successors are elected and qualified. x x x

SEC. 36. Corporate powers and capacity. — Every corporation incorporated under this Code has the power and capacity:

xxx

7. To purchase, receive, take or grant, hold, convey, sell, lease, pledge, mortgage and otherwise deal with such real and
personal property, including securities and bonds of other corporations, as the transaction of a lawful business of the corporation
may reasonably and necessarily require, subject to the limitations prescribed by the law and the Constitution.

xxx

Under these provisions, the power to purchase real property is vested in the board of directors or trustees. While a corporation
may appoint agents to negotiate for the purchase of real property needed by the corporation, the final say will have to be with
the board, whose approval will finalize the transaction.50 A corporation can only exercise its powers and transact its business
through its board of directors and through its officers and agents when authorized by a board resolution or its by-laws. 51 As held
in AF Realty & Development, Inc. v. Dieselman Freight Services, Co.:52

Section 23 of the Corporation Code expressly provides that the corporate powers of all corporations shall be exercised by the
board of directors. Just as a natural person may authorize another to do certain acts in his behalf, so may the board of directors
of a corporation validly delegate some of its functions to individual officers or agents appointed by it. Thus, contracts or acts of a
corporation must be made either by the board of directors or by a corporate agent duly authorized by the board. Absent such
valid delegation/authorization, the rule is that the declarations of an individual director relating to the affairs of the corporation,
but not in the course of, or connected with, the performance of authorized duties of such director, are held not binding on the
corporation. (Emphasis supplied)

In this case, Aviles, who negotiated the purchase of the Property, is neither an officer of Bukal Enterprises nor a member of the
Board of Directors of Bukal Enterprises. There is no Board Resolution authorizing Aviles to negotiate and purchase the Property
for Bukal Enterprises. There is also no evidence to prove that Bukal Enterprises approved whatever transaction Aviles made
with the Spouses Firme. In fact, the president of Bukal Enterprises did not sign any of the deeds of sale presented to the
Spouses Firme. Even De Castro admitted that he had never met the Spouses Firme. 53 Considering all these circumstances, it is
highly improbable for Aviles to finalize any contract of sale with the Spouses Firme.

Furthermore, the Court notes that in the Complaint filed by Bukal Enterprises with the trial court, Aviles signed 54 the verification
and certification of non-forum shopping.55 The verification and certification of non-forum shopping was not accompanied by proof
that Bukal Enterprises authorized Aviles to file the complaint on behalf of Bukal Enterprises.

The power of a corporation to sue and be sued is exercised by the board of directors. "The physical acts of the corporation, like
the signing of documents, can be performed only by natural persons duly authorized for the purpose by corporate by-laws or by
a specific act of the board of directors." 56

The purpose of verification is to secure an assurance that the allegations in the pleading are true and correct and that it is filed in
good faith.57 True, this requirement is procedural and not jurisdictional. However, the trial court should have ordered the
CivRev (Property) Assignment 2
correction of the complaint since Aviles was neither an officer of Bukal Enterprises nor authorized by its Board of Directors to act
on behalf of Bukal Enterprises.

Whether the Statute of Frauds is applicable

The Court of Appeals held that partial performance of the contract of sale takes the oral contract out of the scope of the Statute
of Frauds. This conclusion arose from the appellate court’s erroneous finding that there was a perfected contract of sale. The
records show that there was no perfected contract of sale. There is therefore no basis for the application of the Statute of
Frauds. The application of the Statute of Frauds presupposes the existence of a perfected contract. 58 Article 1403 of the Civil
Code provides:

Art. 1403. The following contracts are unenforceable, unless they are ratified:

(1) Those entered into in the name of another person by one who has been given no authority or legal representation,
or who has acted beyond his powers;

(2) Those that do not comply with the Statute of Frauds as set forth in this number. In the following cases an agreement
hereafter made shall be unenforceable by action, unless the same, or some note or memorandum thereof, be in writing
and subscribed by the party charged or by his agent; evidence, therefore, of the agreement cannot be received without
the writing, or a secondary evidence of its contents:

xxx

(e) An agreement for the leasing for a longer period than one year, or for the sale of real property or of an interest
therein;

xxx

Whether Bukal Enterprises is a builder in good faith

Bukal Enterprises is not a builder in good faith. The Spouses Firme did not accept Aviles’ offer to purchase the Property. Aviles
testified that when he called the Spouses Firme on 2 March 1995, Dr. Firme informed him that they were no longer interested in
selling the Property. On 4 March 1995, Aviles called again and this time Mrs. Firme told him that they were not selling the
Property. Aviles informed De Castro of the refusal of the Spouses Firme to sell the Property. However, Bukal Enterprises still
proceeded in relocating the squatters and constructing improvements on the Property. De Castro testified:

ATTY. EJERCITO:

Q: The truth of the matter, Mr. Witness, is that the post was constructed sometime late 1994. Is that not correct?

A: No, sir. It is not true.

Q: When was it constructed?

A: That March.

Q: When in March?

A: 1995.

Q: When in March 1995?

A: From the period of March 2, 1995 or two (2) weeks after the removal of the squatters.

Q: When were the squatters removed?

WITNESS:

A: March 6 and 7 because there were four (4) squatters.

ATTY. EJERCITO:

Q: When did you find out that the Spouses Firme did not want to sell the same?

A: First week of March 1995.

Q: In your Complaint you said you find out on March 3, 1995. Is that not correct?

A: I cannot exactly remember, sir.


CivRev (Property) Assignment 2
ATTY. MARQUEDA:

In the Complaint it does not state March 3. Maybe counsel was thinking of this Paragraph 6 which states, "When the
property was rid of the squatters on March 2, 1995 for the documentation and payment of the sale, xxx".

ATTY. EJERCITO:

Q: So, you found out on March 2, 1995 that the defendants were no longer interested in selling to you the property. Is
that correct?

A: Yes, sir, because Mr. Aviles relayed it to me.

Q: Mr. Aviles relayed to you that the Spouses Firme were no longer interested in selling to you the property in March 2,
1995. Is that correct?

A: Yes, sir. Mr. Aviles told me.

Q: In so many words, Mr. Witness, you learned that the Spouses Firme were no longer interested in selling the property
before you spent allegedly all the sum of money for the relocation of squatters for all this construction that you are
telling this Court now?

WITNESS:

A: The refusal to sell is not yet formal and the lawyer sent a letter tendering full payment of the purchase price.

ATTY. EJERCITO:

Q: You mean to say that you did not believe Mr. Aviles when he told you that the Spouses Firme were no longer selling
the property?

A: No, sir.

Q: Was there anything formal when you say the Spouses Firme agreed to sell the property?

A: None, sir.

Q: And yet that time you believe Mr. Aviles when he verbally told you that the Sps. Firme agreed to sell the property? At
what point of the transaction with the Spouses Firme were you advised by your lawyer?

WITNESS:

A: At the time when they refused to sell the lot.

ATTY. EJERCITO:

Q: Was that before the squatters were relocated allegedly by Bukal Enterprises?

A: Yes, sir.

Q: In fact, it was the lawyer who advised you to relocate the squatters. Is it not true?

A: No, sir.59 (Emphasis supplied)

Bukal Enterprises is obviously a builder in bad faith. No deed of sale has been executed in this case. Despite the refusal of the
Spouses Firme to sell the Property, Bukal Enterprises still proceeded to introduce improvements on the Property. Bukal
Enterprises introduced improvements on the Property without the knowledge and consent of the Spouses Firme. When the
Spouses Firme learned about the unauthorized constructions made by Bukal Enterprises on the Property, they advised the latter
to desist from further acts of trespass on their Property. 60

The Civil Code provides:

Art. 449. He who builds, plants or sows in bad faith on the land of another, loses what is built, planted or sown without right of
indemnity.

Art. 450. The owner of the land on which anything has been built, planted or sown in bad faith may demand the demolition of the
work, or that the planting or sowing be removed, in order to replace things in their former condition at the expense of the person
who built, planted or sowed; or he may compel the builder or planter to pay the price of the land, and the owner the proper rent.

Under these provisions the Spouses Firme have the following options: (1) to appropriate what Bukal Enterprises has built
without any obligation to pay indemnity; (2) to ask Bukal Enterprises to remove what it has built; or (3) to compel Bukal
CivRev (Property) Assignment 2
Enterprises to pay the value of the land.61 Since the Spouses Firme are undoubtedly not selling the Property to Bukal
Enterprises, they may exercise any of the first two options. They may appropriate what has been built without paying indemnity
or they may ask Bukal Enterprises to remove what it has built at Bukal Enterprises’ own expense.

Bukal Enterprises is not entitled to reimbursement for the expenses incurred in relocating the squatters. Bukal Enterprises spent
for the relocation of the squatters even after learning that the Spouses Firme were no longer interested in selling the Property.
De Castro testified that even though the Spouses Firme did not require them to remove the squatters, they chose to spend for
the relocation of the squatters since they were interested in purchasing the Property. 62

Whether the Spouses Firme are entitled to compensatory and moral damages

The Court agrees with the Court of Appeals to delete the award for compensatory and moral damages. In awarding actual
damages, the trial court took into account the traveling expenses incurred by the Spouses Firme who are already residing in the
United States. However, the trial court failed to consider the testimony of Dr. Firme that they normally travel to the Philippines
more than once a year to visit their children.63 Thus, the expenses for the roundtrip tickets dated 1996-1997 could not be
attributed solely for the attendance of hearings in the case.

Nevertheless, an award of nominal damages of ₱30,000 is warranted since Bukal Enterprises violated the property rights of the
Spouses Firme.64 The Civil Code provides:

Art. 2221. Nominal damages are adjudicated in order that a right of the plaintiff, which has been violated or invaded by the
defendant, may be vindicated or recognized, and not for the purpose of indemnifying the plaintiff for any loss suffered by him.

Art. 2222. The court may award nominal damages in every obligation arising from any source enumerated in article 1157, or in
every case where any property right has been invaded.

The award of damages is also in accordance with Article 451 of the Civil Code which states that the landowner is entitled to
damages from the builder in bad faith.65

WHEREFORE, we SET ASIDE the Decision of the Court of Appeals and RENDER a new one:

1. Declaring that there was no perfected contract of sale;

2. Ordering Bukal Enterprises to pay the Spouses Firme ₱30,000 as nominal damages.

SO ORDERED.
CivRev (Property) Assignment 2
Republic of the Philippines
SUPREME COURT
Manila

SECOND DIVISION

G.R. No. 152423               December 15, 2010

SPOUSES MARCOS R. ESMAQUEL and VICTORIA SORDEVILLA, Petitioners,


vs.
MARIA COPRADA, Respondent.

DECISION

PERALTA, J.:

Before this Court is a petition for review on certiorari under Rule 45 of the Rules of Court seeking to set aside the Decision 1 and
the Resolution2 of the Court of Appeals, dated April 6, 2001 and February 15, 2002, respectively, (CA) in CA-G.R. SP No.
49994.

The antecedents are as follows:

On February 24, 1997, petitioners, spouses Marcos Esmaquel and Victoria Sordevilla (Victoria) filed an ejectment case 3 against
respondent Maria V. Coprada before the 2nd Municipal Circuit Trial Court (MCTC) of Magdalena, Liliw and Majayjay Laguna.
Petitioners claimed that they are the registered owners of a parcel of land situated in M.H. Del Pilar St., Barangay San Miguel,
Majayjay, Laguna, containing an area of Two Hundred Fifty-Three (253) square meters and covered by Transfer Certificate of
Title (TCT) No. T-93542. In 1945, respondent was able to persuade the petitioners to allow her and her family to use and occupy
the land for their residence, under the condition that they will vacate the premises should petitioners need to use the same.
Respondent and her family were allowed to construct their residential house. Since then, the petitioners never made an attempt
to drive them away out of pity, knowing that respondent and her eight children have no other place to live in. Also, respondent
and her family have been occupying the subject premises free of rent, including payment of realty taxes. Respondent's present
circumstances have completely improved, i.e., some of her children are already working; they are regularly sending her financial
assistance; and she has acquired her own residential house at Barangay Panglan, Majayjay, Laguna. Because of this,
petitioners verbally demanded that respondent vacate the subject land, but the latter refused. Thus, petitioners were forced to
send a demand letter dated August 22, 1996, giving respondent until November 30, 1996 to vacate the subject premises.
However, respondent still ignored said demand, which prompted petitioners to bring a complaint before the barangay authorities.
No settlement was reached, hence, a certification to file action in Court was issued. Petitioners were, therefore, constrained to
lodge an ejectment case against the respondent before the MCTC.

Respondent admitted that petitioners are the registered owners of the subject land. However, she averred that in 1945, it was
Emiliana Coprada (petitioner Victoria Sordevilla's mother and original owner of the subject land) and not the petitioners who
gave permission to her late husband Brigido Coprada to use the subject lot. Emiliana allowed her nephew Brigido and his family
to occupy the lot as their permanent abode, because of her love and affection for her nephew, and also, due to the fact that the
said lot is virtually a wasteland. Thereafter, Brigido and his family cleared the area and built therein a nipa hut to dwell in. When
Emiliana died, the ownership of the property was inherited by her only child, petitioner Victoria Sordevilla. Respondent alleged
that sometime in the early 1960's, petitioner Victoria offered the said lot for sale for ₱2,000.00 to respondent, who readily
agreed. The purchase price was paid in installments and was fully paid in 1962. Due to their close relationship, the agreement
was never reduced to writing. Respondent further maintained that since the execution of the oral sale of the subject lot, she has
been the one paying the realty taxes due on the property. After the sale, respondent built on the subject land a semi-concrete
structure. Respondent stated that petitioners' claim is barred by laches. Even granting, without admitting, that respondent's claim
of ownership over the property is improper because petitioners are the registered owners thereof, respondent argued that she is
a builder in good faith, because she was able to build the structure on the subject lot with the prior permission of the owner.

In its Decision4 dated September 11, 1997, the MCTC rendered judgment dismissing the complaint. It held that laches had
already set in which prevented petitioners from questioning the validity of the purported sale between Victoria and Maria.

On appeal, the Regional Trial Court (RTC) reversed the MCTC’s judgment. The RTC ruled that respondent's occupation of the
subject property was by virtue of petitioners' tolerance and permission. Hence, respondent is bound by an implied promise that
she will vacate the property upon demand. Thus, her possession over the subject property became unlawful after the petitioners
demanded her to vacate the property. The RTC found that respondent failed to prove the alleged oral sale and that petitioners
have adequately proven that they are entitled to the possession of the subject land as registered owners thereof. The RTC
ordered the respondent and all other persons claiming rights under her to vacate and surrender the possession of the subject
land to the petitioners and to remove any and all improvements she introduced on the parcel of land. 5

Respondent filed a Motion for Reconsideration, which was denied by the RTC in an Order 6 dated November 24, 1998. Obviously
dissatisfied by the Decision, respondent filed with the CA a petition for review with prayer for temporary restraining order and
preliminary injunction.7

In its Decision dated April 6, 2001, the CA granted respondent's petition, reversed the Decision of the RTC and affirmed in toto
the Decision of the MCTC. Petitioners filed a Motion for Reconsideration, which was denied by the CA in a Resolution 8 dated
February 15, 2002. Hence, the instant petition raising the following grounds:

I
CivRev (Property) Assignment 2
THE RIGHT OF THE REGISTERED OWNERS TO RECOVER POSSESSION IS NEVER BARRED BY LACHES AND/OR THE
PERSON WHO HAS A TORRENS TITLE OVER A PARCEL OF LAND IS ENTITLED TO THE POSSESSION THEREOF.

II

THE OWNERSHIP AND RIGHT OF PETITIONERS TO RECOVER POSSESSION OF THE SUBJECT PROPERTY CANNOT
BE DEFEATED BY UNPROVEN ORAL SALE.

III

LACHES HAD SET IN AGAINST [RESPONDENT].

IV

THE CERTIFICATE OF TITLE IS NOT SUBJECT TO COLLATERAL ATTACK. 9

The petition is meritorious.

The pertinent point of inquiry in this case is whether or not petitioners have a valid ground to evict respondent from the subject
property.

An action for forcible entry or unlawful detainer is governed by Section 1, Rule 70 of the Rules of Court, which provides:

SECTION 1. Who may institute proceedings, and when. - Subject to the provisions of the next succeeding section, a person
deprived of the possession of any land or building by force, intimidation, threat, strategy, or stealth, or a lessor, vendor, vendee,
or other person against whom the possession of any land or building is unlawfully withheld after the expiration or termination of
the right to hold possession by virtue of any contract, express or implied, or the legal representatives or assigns of any such
lessor, vendor, vendee, or other person, may, at any time within one (1) year after such unlawful deprivation or withholding of
possession, bring an action in the proper Municipal Trial Court against the person or persons unlawfully withholding or depriving
of possession, or any person or persons claiming under them, for the restitution of such possession, together with damages and
costs.

In unlawful detainer cases, the possession of the defendant was originally legal, as his possession was permitted by the plaintiff
on account of an express or implied contract between them. However, defendant's possession became illegal when the plaintiff
demanded that defendant vacate the subject property due to the expiration or termination of the right to possess under their
contract, and defendant refused to heed such demand. 10

The sole issue for resolution in an unlawful detainer case is physical or material possession of the property involved,
independent of any claim of ownership by any of the parties. Where the issue of ownership is raised by any of the parties, the
courts may pass upon the same in order to determine who has the right to possess the property. The adjudication is, however,
merely provisional and would not bar or prejudice an action between the same parties involving title to the property. 11 Since the
issue of ownership was raised in the unlawful detainer case, its resolution boils down to which of the parties' respective evidence
deserves more weight.

In the case at bar, petitioners' cause of action for unlawful detainer is based on their ownership of the land covered by TCT No.
T-93542 and on their claim that they merely tolerated respondent's stay thereat. Respondent's possession, as well as those
persons claiming right under her, became unlawful upon her refusal to vacate the premises. Petitioners contend that since they
are the registered owners of the subject land, they are entitled to the possession thereof and their right to recover possession
over it is never barred by laches. They maintain that respondent's claim of ownership is based on an unproven oral sale, which
does not exist. Further, respondent cannot rely on the Tax Declarations as she was paying taxes in the petitioners' name, as the
declared owners of the property. Moreover, she started paying the taxes only in 1984 despite her claim that the property was
sold to her in 1962. Even assuming that the sale took place in 1962, respondent is guilty of laches as she failed to take any
positive action for the delivery and conveyance to her of the portion of the property she is occupying. Finally, respondent cannot
collaterally attack the title of the petitioners to the subject land.

On her part, respondent, although admitting that the property is registered in petitioners' name, claimed that the 100-square-
meters portion of the property, where her house was erected, was already sold to her by petitioner Victoria. Thus, by virtue of
the sale, she and her family have the right to possess the said property. The non-presentation of receipt and deed of sale, non-
delivery of the owner's certificate of title, and her payment of the real property taxes in the name of the petitioners were due to
the close relationship between the parties and the existing practice of palabra de honor in their day to day transactions.
Respondent further alleged that she is not guilty of laches; rather, it is the registered owners' right to recover possession of their
property which is barred by laches.

In the present case, respondent failed to present evidence to substantiate her allegation that a portion of the land was sold to
her in 1962. In fact, when petitioners sent a letter12 to the respondent, demanding her to vacate the subject property, the
respondent, in reply13 to the said letter, never mentioned that she purchased the subject land in 1962. If the sale really took
place, the respondent should have immediately and categorically claimed that in her letter response. Clearly therefore,
respondent's submission that there was an oral sale is a mere afterthought.

On the other hand, it is undisputed that the subject property is covered by Transfer Certificate of Title No. T-93542, registered in
the name of the petitioners. As against the respondent's unproven claim that she acquired a portion of the property from the
petitioners by virtue of an oral sale, the Torrens title of petitioners must prevail. Petitioners' title over the subject property is
evidence of their ownership thereof. It is a fundamental principle in land registration that the certificate of title serves as evidence
CivRev (Property) Assignment 2
of an indefeasible and incontrovertible title to the property in favor of the person whose name appears therein. Moreover, the
age-old rule is that the person who has a Torrens title over a land is entitled to possession thereof. 14

Further, respondent's argument that petitioners are no longer the owners of a portion of the subject land because of the sale in
her favor is a collateral attack on the title of the petitioners, which is not allowed. The validity of petitioners' certificate of title
cannot be attacked by respondent in this case for ejectment. Under Section 48 of Presidential Decree No. 1529, a certificate of
title shall not be subject to collateral attack. It cannot be altered, modified or canceled, except in a direct proceeding for that
purpose in accordance with law. The issue of the validity of the title of the petitioners can only be assailed in an action expressly
instituted for that purpose. Whether or not the respondent has the right to claim ownership over the property is beyond the power
of the trial court to determine in an action for unlawful detainer. 15

In Rodriguez v. Rodriguez,16 citing the case of Co v. Militar,17 the Court held that:

[T]he Torrens System was adopted in this country because it was believed to be the most effective measure to guarantee the
integrity of land titles and to protect their indefeasibility once the claim of ownership is established and recognized.

It is settled that a Torrens Certificate of title is indefeasible and binding upon the whole world unless and until it has been
nullified by a court of competent jurisdiction. Under existing statutory and decisional law, the power to pass upon the validity of
such certificate of title at the first instance properly belongs to the Regional Trial Courts in a direct proceeding for cancellation of
title.

As the registered owner, petitioner had a right to the possession of the property, which is one of the attributes of ownership. x x x

Anent the issue on laches, the CA's ruling that petitioners' long inaction to assert their rights over the subject land bars them
from recovering the same is without basis. Also, the doctrine invoked by the appellate court that a registered owner may loose
his right to recover its possession by reason of laches is not applicable here.

Laches is the failure or neglect, for an unreasonable and unexplained length of time, to do that which, by exercising due
diligence, could or should have been done earlier; it is negligence or omission to assert a right within a reasonable time,
warranting the presumption that the party entitled to assert it either has abandoned or declined to assert it. 18 There is no
absolute rule as to what constitutes laches or staleness of demand; each case is to be determined according to its particular
circumstances, with the question of laches addressed to the sound discretion of the court. Because laches is an equitable
doctrine, its application is controlled by equitable considerations and should not be used to defeat justice or to perpetuate fraud
or injustice.19

Respondent first acquired possession of the subject lot by mere tolerance. From 1945 until the filing of the complaint for
ejectment in 1997, the nature of that possession has never changed. Petitioners allowed the respondent to possess the property
with the knowledge that the respondent will vacate the same upon demand. Hence, until such demand to vacate was
communicated by the petitioners to the respondent, petitioners are not required to do any act to recover the subject land,
precisely because they knew of the nature of the respondent's possession, i.e., possession by mere tolerance. Thus, it cannot
be said that petitioners are guilty of failure or neglect to assert a right within a reasonable time. Further, after the petitioners gave
a demand letter to the respondent giving the latter until November 30, 1996 to vacate the subject premises, which respondent
failed to heed, they immediately filed a complaint before the barangay authorities and, thereafter, lodged an ejectment case
before the MCTC on February 24, 1997. In sum, We find that petitioners are not guilty of laches as would bar their claim to the
property in question.

In contrast, respondent, who is claiming that a portion of the property was sold to her in 1962, has herself failed within a long
period of time to have that portion transferred in her name. Respondent had to wait for almost 35 years since 1962, and were it
not for the filing of the ejectment suit in 1997, she would not have bothered to assert her rights under the alleged sale.
Respondent's failure to assert that right only goes to prove that no sale ever transpired between the parties.

Moreover, as the registered owners, petitioners' right to eject any person illegally occupying their property is not barred by
laches. In Gaudencio Labrador, represented by Lulu Labrador Uson, as Attorney-in-Fact v. Spouses Ildefonso Perlas and
Pacencia Perlas and Spouse Rogelio Pobre and Melinda Fogata Pobre, 20 the Court held that:

x x x As a registered owner, petitioner has a right to eject any person illegally occupying his property. This right is
imprescriptible and can never be barred by laches. In Bishop v. Court of Appeals, we held, thus:

As registered owners of the lots in question, the private respondents have a right to eject any person illegally occupying their
property. This right is imprescriptible. Even if it be supposed that they were aware of the petitioners' occupation of the property,
and regardless of the length of that possession, the lawful owners have a right to demand the return of their property at any time
as long as the possession was unauthorized or merely tolerated, if at all. This right is never barred by laches.

Since respondent's occupation of the subject lot is by mere tolerance or permission of the petitioners, without any contract
between them, respondent is bound by an implied promise that she will vacate the same upon demand, failing which a summary
action for ejectment is the proper remedy against her.21

In respondent's Answer filed before the MCTC, she claimed that since she was able to build a structure on the subject lot with
the prior permission from the owner, she is a builder in good faith and thus entitled to be reimbursed the necessary and useful
expenses under Articles 546 and 548 of the Civil Code of the Philippines. Without such reimbursement, she has the right of
retention over the property and she cannot just be ejected from the premises.
CivRev (Property) Assignment 2
Respondent's argument does not hold water. Since respondent's occupation of the subject property was by mere tolerance, she
has no right to retain its possession under Article 448 of the Civil Code. She is aware that her tolerated possession may be
terminated any time and she cannot be considered as builder in good faith. 22 It is well settled that both Article 44823 and Article
54624 of the New Civil Code, which allow full reimbursement of useful improvements and retention of the premises until
reimbursement is made, apply only to a possessor in good faith, i.e., one who builds on land with the belief that he is the owner
thereof. Verily, persons whose occupation of a realty is by sheer tolerance of its owners are not possessors in good faith. 25 At
the time respondent built the improvements on the premises in 1945, she knew that her possession was by mere permission
and tolerance of the petitioners; hence, she cannot be said to be a person who builds on land with the belief that she is the
owner thereof.

Respondent's reliance on her payment of realty taxes on the property is unavailing. She started paying taxes only in 1984
despite her claim that she bought the property in 1962. Further, aside from the rule that tax declarations and corresponding tax
receipts cannot be used to prove title to or ownership of a real property inasmuch as they are not conclusive evidence of the
same,26 the RTC found that although the payment for said taxes were received from respondent, the declared owner was
petitioner Victoria.

It must be stressed, however, that the court's adjudication of ownership in an ejectment case is merely provisional, and
affirmance of the RTC's decision would not bar or prejudice an action between the same parties involving title to the property, if
and when such action is brought seasonably before the proper forum. 27

WHEREFORE, the petition is GRANTED. The Decision and the Resolution of the Court of Appeals, dated April 6, 2001 and
February 15, 2002, respectively, in CA-G.R. SP No. 49994, affirming the Decision of the 2nd Municipal Circuit Trial Court in Civil
Case No. 1875, are REVERSED and SET ASIDE. The Decision of the Regional Trial Court of Santa Cruz, Laguna, Branch 26,
in Civil Case No. SC-3580, is REINSTATED.

SO ORDERED.
CivRev (Property) Assignment 2
Republic of the Philippines
SUPREME COURT
Manila

FIRST DIVISION

G.R. No. 182754               June 29, 2015

SPOUSES CRISPIN AQUINO and TERESA V. AQUINO, herein represented by their Attorney-in-Fact, AMADOR D.
LEDESMA, Petitioners,
vs.
SPOUSES EUSEBIO AGUILAR and JOSEFINA V. AGUILAR, Respondents.

DECISION

SERENO, CJ:

In this Petition for Review on Certiorari1 filed under Rule 45 of the Rules of Court, Petitioner spouses Crispin and Teresa Aquino
(petitioners) assail the Court of Appeals (CA) Decision dated 25 April 2008 2 in CA-GR SP No. 92778. The CA modified the
Decisions of both the Metropolitan Trial Court (MeTC) and the Regional Trial Court (RTC). The CA ruled that although
respondent spouses Eusebio and Josefina Aguilar (respondents) cannot be considered builders in good faith, they should still be
reimbursed for the improvements they have introduced on petitioners' property. 3

THE FACTS

Teresa Vela Aquino (Teresa) and her husband, Crispin Aquino, are the owners of a house and lot located at No. 6948, Rosal
Street, Guadalupe

Since 1981, this property has been occupied by Teresa's sister, Josefina Vela Aguilar; Josefina's spouse Eusebio; and their
family.5 It appears from the record that respondents stayed on the property with the consent and approval of petitioners, who
were then residing in the United States.6

While respondents were in possession of the property, the house previously constructed therein was demolished, and a three-
storey building built in its place.7 Respondents occupied half of the third floor of this new building)for the next 20 years without
payment of rental.8

On 22 September 2003, petitioners sent a letter to respondents informing them that an immediate family member needed to use
the premises and demanding the surrender of the property within 10 days from notice. 9 Respondents failed to heed this demand,
prompting petitioners to file a Complaint for ejectment against them before the office of the barangay captain of Guadalupe
Viejo.10 The parties attempted to reach an amicable settlement in accordance with Section 412 of the Local Government Code,
but these efforts proved unsuccessful.11

On 19 November 2003, petitioner spouses Aquino filed a Complaint 12 with the MeTC of Makati City praying that respondents be
ordered to (a) vacate the portion of the building they were then occupying; and (b) pay petitioner a reasonable amount for the
use and enjoyment of the premises from the time the formal demand to vacate was made. 13

In their Answer with Counterclaim,14 respondents claimed that they had contributed to the improvement of the property and the
construction of the building, both in terms of money and management/supervision services. Petitioners purportedly agreed to let
them contribute to the costs of construction in exchange for the exclusive use of a portion of the building. Respondents averred:

2.3 That the construction of the three (3) storey building was also at the uncompensated supervision of defendant
Eusebio Aguilar, of which only r 2 Million was spent by plaintiffs while defendants spent around r 1 Million as
contribution to the construction cost. It was defendants who introduced improvements on subject lot because at the time
plaintiffs bought the property it was marshy which was filled up by defendants (sic) truck load with builders, adobe and
scumbro that elevated the ground;

2.4 The original agreement was for my client to contribute his share so that they will have the portion of the subject
building for their own exclusive use. It turned out later that the agreement they had was disowned by plaintiffs when
they saw the totality of the building constructed thereon coupled by the fact, that the value of the lot has tremendously
appreciated due to the commercialization of the vicinity which will command higher price and windfall profits should
plaintiffs sell the property which they are now contemplating on (sic);

2.5 The portion which plaintiffs want defendants to vacate is a portion which the latter built with their own money upon
your clients agreement and consent whom they built in good faith knowing and hoping that later on the same will be
theirs exclusively. It was never an act of generosity, liberality and tolerance. Conversely, it was one of the implied co-
ownership or partnership, because aside from the fact that defendants, who were then peacefully residing in Laguna,
made unquantifiable contributions in terms of money and services arising from his uncompensated management and
supervision over the entire subject property while plaintiffs are abroad. By legal implications he is an industrial partner
responsible for the development and improvements of the subject property. His contribution was never without the
consent of plaintiffs. Whatever contribution defendants introduced over the said property was made and built in good
faith;15
CivRev (Property) Assignment 2
Since they were allegedly co-owners of the building and builders in good faith, respondents claimed that they had the right to be
compensated for the current value of their contribution.16 Accordingly, they prayed for the dismissal of the Complaint and the
award of ₱5 million as compensation for their contributions to the construction of the building, as well as moral damages,
attorney's fees and costs of litigation.17

THE RULING OF THE METC

In a Decision18 dated 12 November 2004, the MeTC ruled in favor of petitioners, stating that they had the right to enjoy
possession of the property as the registered owners thereof. 19 Since the case was merely one for ejectment, the court held that it
was no longer proper to resolve respondents' claim of co-ownership over the building. 20

The MeTC also declared that respondents were builders in bad faith who were not entitled to recover their purported expenses
for the construction of the building.21 It emphasized that their occupation of the property was by mere tolerance of petitioners
and, as such, could be terminated at any time.22 The court further noted that in a letter dated 15 July 1983, petitioners had
already asked respondents to refrain from constructing improvements on the property because it was intended to be sold. 23

The dispositive portion of the MeTC Decision, which ordered respondents to vacate the property, reads:

WHEREFORE, premises considered, judgment is hereby rendered ordering defendants Eusebio & Josefina Aguilar and all
persons claiming rights under them to immediately vacate the subject property, and deliver peaceful possession thereof to the
plaintiffs. Defendants are likewise ordered to pay plaintiffs ₱7,000.00 monthly rental commencing 22 October 2003 until such
time that defendant finally vacate the premises, ₱10,000.00 as and by way of attorney's fees, and the cost of suit. 24

On 14 September 2005, respondents appealed the MeTC's Decision to the RTC. 25

THE RULING OF THE RTC

In their Memorandum on Appeal26 before the R TC, respondents assailed the MeTC's finding that petitioners, as the registered
owners of the land, were also the owners of the improvement constructed thereon. 27 Respondents asserted that they were co-
owners of the building since they built a portion thereof using their own funds, as evidenced by various receipts they presented
before the MeTC.28

Respondents also maintained that they were builders in good faith. They pointed out that petitioners never objected to the
construction of the improvement on their property.29 According to respondents, petitioners' letter dated 15 July 1983 was written
at a time when an old dilapidated house was still standing on the property. 30 Subsequently however, the house was demolished
and the new building was constructed thereon by respondents, with petitioners' knowledge and consent. 31

In a Decision32 dated 3 January 2006, the RTC denied the appeal and affirmed the MeTC's Decision. According to the court,
respondents did not become co-owners of the property although they may have contributed to the construction of the building
thereon.33 Hence, their stay in the premises remained to be by mere tolerance of the petitioners. 34

The RTC also ruled that respondents cannot be considered builders in good faith. 35 The court found that as early as 1983,
petitioners had informed respondents of the intention to eventually dispose of the property. 36 The RTC concluded that petitioners
never consented to the construction of any form of structure on the property. 37 Since respondents participated in the construction
of the building even after they had been notified that their occupation may be terminated anytime, the R TC ruled that they did
not build the structures in good faith.38 The RTC likewise noted that "the improvements in question as well as other personal
belongings of the appellants were removed from the premises through a writ of demolition, and these properties are now in their
possession."39

THE RULING OF THE CA

Aggrieved by the RTC Decision, respondents elevated the matter to the CA. They reiterated that they owned one-half of the third
floor of the building on the property, having spent their own funds for the construction thereof. Respondents also asserted that
because they built that portion in good faith, with no objection from petitioners, they were entitled to reimbursement of all
necessary and useful expenses incurred in the construction.

On 25 April 2008, the CA affirmed the conclusion of the lower courts that respondents could not be considered co-owners of the
property or builders in good faith.40 According to the appellate court, respondents were aware that their right to possess the
property had a limitation, because they were not the owners thereof. They knew that their occupation of the building was by
mere tolerance or permission of petitioners, who were the registered owners of the property. The CA likewise noted that
respondents failed to prove the alleged agreement between the parties with respect to the ownership of one-half of the third floor
of the improvement. There being no contract between them, respondents are necessarily bound to vacate the property upon
demand.41 The CA ruled:

The Supreme Court has consistently held that those who occupy the land of another at the latter's tolerance or permission,
without any contract between them, are necessarily bound by an implied promise that the occupants will vacate the property
upon demand. Based on the principles enunciated in Calubayan v. Pascual, the status of petitioners is analogous to that of a
lessee or a tenant whose term of lease has expired but whose occupancy continued by tolerance of the owner. In such a case,
the unlawful deprivation or withholding of possession is to be reckoned from the date of the demand to vacate. 42 (Citations
omitted)
CivRev (Property) Assignment 2
Nevertheless, the CA declared that respondents should be reimbursed for the necessary and useful expenses they had
introduced on petitioners' property, pursuant to Articles 1678 and 548 of the Civil Code. 43 The dispositive portion of the CA
Decision dated 25 April 200844 reads:

WHEREFORE, the assailed Decision is AFFIRMED with the following MODIFICATIONS:

1. The case is REMANDED to the court of origin for further proceedings to determine the facts essential to the
application of Article 1678 and Article 546 of the Civil Code, specifically on the following matters:

a) To determine the cost of necessary expenses incurred by petitioners during their period of possession.

b) To determine the cost of useful improvements introduced by petitioners in the construction of the building.

2. After said amounts shall have been determined by competent evidence:

a) Respondents Aquino are ordered to pay petitioners the costs of necessary improvements incurred during the
period of their occupation.

b) Petitioners Aguilar are to be reimbursed one half (1/2) of the amount they expended on the construction of
the building should respondents decided to appropriate the same. Should respondents refuse to reimburse the
costs of the improvements, petitioners may remove the improvements even though the principal thing may
suffer damage thereby.

c) In both instances, petitioners shall have no right of retention over the subject premises.

d) In any event, petitioners shall pay respondents the amount of Php7,000.00 as monthly rental commencing
22 October 2003 until such time that petitioners finally vacate the premises. No pronouncement as to costs.

SO ORDERED.45

Respondents no longer appealed the Decision of the CA. This time, petitioners elevated the matter to this Court through the
instant Petition for Review46 under Rule 45 of the Rules of Court.

PROCEEDINGS BEFORE THIS COURT

In their Petition, petitioners allege that the CA seriously erred in remanding the case to the court of origin for the purpose of
ascertaining the right of respondents to be reimbursed for the improvements introduced on the property. 47 They emphasize that
respondents were builders in bad faith, and, as such, are not entitled to reimbursement under Articles 449, 450 and 451 of the
Civil Code.

In their Comment,48 respondents assert that the CA correctly ruled that their status is akin to that of a lessee or tenant whose
term of lease has expired, but whose occupancy continues by virtue of the tolerance of the owner. They aver that the CA
properly upheld their entitlement to reimbursement pursuant to Articles 1678 49 and 54650 of the Civil Code.51

In their Reply,52 petitioners argue against supposed improvements constructed by respondents from 1999 to 2003 amounting to
₱995,995.94. Petitioners say this claim is highly ridiculous and unbelievable. 53

OUR RULING

Since respondents no longer appealed the Decision of the CA, 54 they are considered bound by its findings and conclusions.
These include its affirmation of the earlier findings of the MeTC and the RTC that respondents cannot be considered builders in
good faith:

Both the MeTC and the RTC have rejected the idea that petitioners are builders in good faith. We agree. The resolution of the
issues at bar calls for the application of the rules on accession under the Civil Code. The term "builder in good faith" as used in
reference to Article 448 of the Civil Code, refers to one who, not being the owner of the land, builds on that land believing
himself to be its owner and unaware of the land, builds on that land, believing himself to be its owner and unaware of the defect
in h is title or mode of acquisition. The essence of good faith lies in an honest belief in the validity of one's right, ignorance of a
superior claim, and absence of intention to overreach another.

In the instant case, the Spouses Aguilar cannot be considered as builders in good faith on account of their admission that the
subject lot belonged to the Spouses Aquino when they constructed the building. At the onset, petitioners were aware of a flaw in
their title and a limit to their right to possess the property. By law, one is considered in good faith if he is not aware that there
exists in his title or mode of acquisition any flaw which invalidates it. 55

Respondents are deemed to have acquiesced to the foregoing findings when they failed to appeal the CA Decision. A party who
does not appeal from a judgment can no longer seek the modification or reversal thereof. 56 Accordingly, the only issue left for
this Court to determine is that which is now raised by petitioners - whether the CA erred in remanding this case to the court of
origin for the determination of the necessary and useful expenses to be reimbursed to respondents pursuant to Articles 1678
and 546 of the Civil Code.

We resolve to PARTLY GRANT the Petition and modify the ruling of the CA.
CivRev (Property) Assignment 2
Article 1678 is not applicable to this case.

In its Decision, the CA found that respondents were occupants of the property by mere tolerance or generosity of petitioners and
were bound by an implied promise to vacate the premises upon demand. 57

Based on this finding, the CA held that "the status of petitioners is analogous to that of a lessee or a tenant whose term of lease
has expired but whose occupancy continued by tolerance of owner" 58 pursuant to this Court's ruling in Calubayan v.
Pascual,59 As a result, the CA concluded that Articles 1678 and 546 of the Civil Code must be applied to allow respondents to be
reimbursed for their necessary and useful expenses.

We disagree. By its express provision, Article 1678 of the Civil Code applies only to lessees who build useful improvements on
the leased property. It does not apply to those who possess property by mere tolerance of the owners, without a contractual
right.

A careful reading of the statement made by this Court in Calubayan would show that it did not, as it could not, modify the
express provision in Article 1678, but only noted an "analogous" situation. According to the Court, the analogy between a tenant
whose term of lease has expired and a person who occupies the land of another at the latter's tolerance lies in their implied
obligation to vacate the premises upon demand of the owner. The Court stated:

To begin with, it would appear that although the defendant is regarded by the plaintiffs as a "squatter" his occupancy of the
questioned premises had been permitted or tolerated even before the Philippine Realty Corporation sold the lots to the plaintiffs.
Otherwise, the latter would not have found him on the premises. It may be true that upon their acquisition of the parcels of land
in 1957, plaintiffs notified and .requested defendant to see them, but despite defendant's failure to heed these requests, plaintiffs
did not choose to bring an action in court but suffered the defendant instead to remain in the premises for almost six years. Only
on February 2, 1963, did the plaintiffs for the first time notify the defendant that "they now need the two parcels of land in
question" and requested him to vacate the same. In allowing several years to pass without requiring the occupant to vacate the
premises nor filing an action to eject him, plaintiffs have acquiesced to defendant's possession and use of the premises. It has
been held that a person who occupies the land of another at the latter's tolerance or permission, without any contract between
them, is necessarily bound by an implied promise that he will vacate upon demand, failing which a summary action for ejectment
is the proper remedy against them. The status of defendant is analogous to that of a lessee or tenant whose term of lease has
expired but whose occupancy continued by tolerance of the owner. In such a case, the unlawful deprivation or withholding of
possession is to be counted from the date of the demand to vacate. 60 (Emphasis in the original)

It is clear from the above that Calubayan is not sufficient basis to confer the status and rights of a lessee on those who occupy
property by mere tolerance of the owner.

In this case, there is absolutely no evidence of any lease contract between the parties. In fact, respondents themselves never
alleged that they were lessees of the lot or the building in question. Quite the opposite, they insisted that they were co-owners of
the building and builders in good faith under Article 448 of the Civil Code. For that reason, respondents argue that it was
erroneous for the CA to consider them as lessees and to determine their rights in accordance with Article 1678.

As builders in bad faith, respondents are


not entitled to reimbursement of useful expenses.

Furthermore, even if we were to subscribe to the CA' s theory that the situation of respondents is "analogous to that of a lessee
or tenant whose term of lease has expired but whose occupancy continued by tolerance," the absence of good faith on their part
prevents them from invoking the provisions of Article 1678.

As discussed above, the MeTC, the RTC and the CA all rejected the claims of respondents that they were builders in good faith.
This pronouncement is considered conclusive upon this Court, in view of respondents' failure to appeal from the CA decision.
This rule bars the application of Article 1678 as well as Articles 448 and 576 of the Civil Code and all other provisions requiring
good faith on the part of the builder.

We are aware that in some instances, this Court has allowed the application of Article 448 to a builder who has constructed
improvements on the land of another with the consent of the owner. 61 In those cases, the Court found that the owners knew and
approved of the construction of improvements on the property. Hence, we ruled therein that the structures were built in good
faith, even though the builders knew that they were constructing the improvement on land owned by another.

Although the factual circumstances in the instant case are somewhat similar, there is one crucial factor that warrants a departure
from the above-described rulings: the presence of evidence that petitioners prohibited respondents from building their own
structure on a portion of the property. Based on the findings of fact of the MeTC and the RTC, petitioners had already warned
respondents not to build a structure on the property as early as 1983. The MeTC explained: Likewise, in a letter dated 15 July
1983 sent by plaintiffs to the defendants marked as Exhibit "2" of defendants' Position Paper, Teresa Aquino made known to the
defendants not to construct on the premises as she planned to sell the same when the value of the property shall increase (sic).
Defendants are undoubtedly builders in bad faith for despite the prohibition made upon them, they continued their construction
activities upon respondents' property.62

This ruling was affirmed by the R TC in its Decision dated 3 January 2006, which reads:

An examination of appellants' Exhibit "2" which is a letter dated July 15, 1983, sent to appellant Josefina Aguilar, the sister of
appellee Teresa Aquino, abundantly shows that their occupancy of the premises in question is by tolerance of the appellees.
Thus, the letter expressly states that the appellants are advised not to put up a shop, as the appellees had plan (sic) then of
disposing the property (the land) in question for a reasonable profit after a period of three or four years, thereby placing on
CivRev (Property) Assignment 2
notice them (appellants) that their possession of the said property is temporary in nature and by mere generosity of the
appellees, they being sisters.

The letter likewise advised them to apply for a housing project so that by the time the property in question is sold, they have a
place to transfer to. All these undisputed antecedents which can be considered as judicially admitted by the appellants being
their own evidence marked as Exhibit "2", coupled with the fact that since the time they occupied the premises in 1983 up to the
time when the complaint was filed, they were not asked to pay any monthly rental for the use, enjoyment and occupancy of the
said property, ineluctably established the fact that their possession of the said property is by mere tolerance of the appellees. 63

xxxx

Their contention that pursuant to Article 453 of the Civil Code, they should be considered builders in good faith even if they have
acted in bad faith, since their act of introducing improvements to one-half of the third floor of the three storey building was with
knowledge and without opposition on the part of the appellants, cannot be sustained, principally on the ground that as stated
earlier, their Exhibit "2" is very limpid on the act that they were already forewarned as early as 1983 not to introduce any
improvements thereon as the property is slated to be sold as it was only bought for investment purposes. The fact that the
appellees did not thereafter remind them of this, is of no moment, as this letter was not likewise withdrawn by a subsequent one
or modified by the appellees.64

We find no reason to depart from the conclusions of the trial courts. Respondents were evidently prohibited by petitioners from
building improvements on the land because the latter had every intention of selling it. That this sale did not materialize is
irrelevant. What is crucial is that petitioners left respondents clear instructions not to build on the land.

We also agree with the RTC's ruling that the lack of constant reminders from petitioners about the "prohibition" expressed in the
1983 letter was immaterial. The prohibition is considered extant and continuing since there is no evidence that this letter was
ever withdrawn or modified. Moreover, no evidence was presented to show that petitioners were aware of what was happening:
that respondents were constructing a portion of the building with their own funds and for their exclusive use and ownership.
Neither were respondents able to present evidence that petitioners had agreed to share the expenses with them, or that the
former had given consent to the latter's contribution, if any.

In view of the foregoing, this Court's previous rulings on Article 448 cannot be applied to this case. Hence, we hold that
petitioners, as the owners of the land, have the right to appropriate what has been built on the property, without any obligation to
pay indemnity therefor;65 and that respondents have no right to a refund of any improvement built therein, 66 pursuant to Articles
449 and 450 of the Civil Code:

Art. 449. He who builds, plants or sows in bad faith on the land of another, loses what is built, planted or sown without right of
indemnity.

Art. 450. The owner of the land on which anything has been built, planted or sown in bad faith may demand the demolition of the
work, or that the planting or sowing be removed, in order to replace things in their former condition at the expense of the person
who built, planted or sowed; or he may compel the builder or planter to pay the price of the land, and the sower the proper rent.

Art. 451. In the cases of the two preceding articles, the landowner is entitled to damages from the builder, planter or sower.

Respondents may recover the


necessary expenses incurred for the
preservation of the property but
without the right of retention.

Pursuant to Article 452 of the Civil Code, a builder in bad faith is entitled to recoup the necessary expenses incurred for the
preservation of the land.67 The CA correctly ruled that respondents in this case are similarly entitled to this reimbursement.
However, being builders in bad faith, they do not have the right of retention over the premises. 68

While the evidence before this Court does not establish the amount of necessary expenses incurred by respondents during their
stay in the property, we note that even petitioners do not deny that such expenses were incurred. In fact, in a letter dated 15 July
1983, petitioners acknowledged that respondents had spent personal money for the maintenance of the property. Petitioners
even promised to reimburse them for those expenses.69 In this light, we find it proper to order the remand of this case to the
court a quo for the purpose of determining the amount of necessary expenses to be reimbursed to respondents.

With respect to the award of actual damages to petitioners, we find no reason to reverse or modify the ruling of the
CA.1âwphi1 This Court has consistently held that those who occupy the land of another at the latter's tolerance or permission,
even without any contract between them, are necessarily bound by an implied promise that the occupants would vacate the
property upon demand.70 Failure to comply with this demand renders the possession unlawful and actual damages may be
awarded to the owner from the date of the demand to vacate 71 until the actual surrender of the property.

Accordingly, we affirm the CA's award of actual damages to petitioners in the amount of ₱7 ,000 per month from the date of
demand (22 October 2003) until the subject properties are vacated. This amount represents a reasonable compensation for the
use and occupation of respondents' property 72 as determined by the RTC and the MeTC.

As to petitioners' prayer for attorney's fees, we find no cogent basis for the award. WHEREFORE, the Petition is PARTLY
GRANTED.
CivRev (Property) Assignment 2
The Court of Appeals Decision dated 25 April 2008 is REVERSED insofar as it ordered: (a) the reimbursement of the useful
expenses incurred by respondents while in possession of the property; and (b) the determination of the cost of these useful
improvements by the court of origin. The rest of the Decision of the Court of Appeals is hereby AFFIRMED.

Accordingly, this case is REMANDED to the court of origin for the determination of the necessary expenses of preservation of
the land, if any, incurred by respondent spouses Eusebio and Josefina Aguilar while they were in possession of the property,
which expenses shall be reimbursed to them by petitioner spouses Crispin and Teresa Aquino.

On the other hand, respondents and all persons claiming rights under them are ordered, upon finality of this Decision without
awaiting the resolution of the matter of necessary expenses by the trial court, to immediately VACATE the subject property and
DELIVER its peaceful possession to petitioners. Respondents are likewise ordered to PAY petitioners ₱7 ,000 as monthly rental
plus interest thereon at the rate of 6% per annum, to be computed from 22 October 2003 until the finality of this Decision.

No pronouncement as to costs.

SO ORDERED.
CivRev (Property) Assignment 2
Republic of the Philippines
SUPREME COURT
Manila

FIRST DIVISION

G.R. No. 77294 December 12, 1988

ANGELICA VIAJAR and CELSO VIAJAR, plaintiffs-appellants,


vs.
COURT OF APPEALS, LEONOR P. LADRIDO, LOURDES LADRIDO IGNACIO, EUGENIO P. LADRIDO and L P.
LADRIDO, defendants-appellees.

Ramon A. Gonzales for petitioner.

Miraflores Law Offices for respondents.

MEDIALDEA, J.:

This is a petition for review on certiorari of the decision of the Court of Appeals dated December 29, 1986, in CA-G.R. CV No.
69942 entitled, "ANGELICA VIAJAR, et. al., Plaintiffs-Appellants, versus LEONOR LADRIDO, et. al., Defendants-
Appellees," affirming the decision of the Court of First Instance (now Regional Trial Court) of Iloilo dated December 10, 1981.

The antecedent facts in the instant case are as follows: The spouses Ricardo Y. Ladrido and Leonor P. Ladrido were the owners
of Lot No. 7511 of the Cadastral Survey of Pototan situated in barangay Cawayan, Pototan, Iloilo. This lot contained an area of
154,267 square meters and was registered in the names of the spouses under Transfer Certificate of Title No. T-21940 of the
Register of Deeds of Iloilo.

Spouses Rosendo H. Te and Ana Te were also the registered owners of a parcel of land described in their title as Lot No. 7340
of the Cadastral Survey of Pototan.

On September 6, 1973, Rosendo H. Te, with the conformity of Ana Te, sold this lot to Angelica F. Viajar and Celso F. Viajar for
P5,000. A Torrens title was later issued in the names of Angelica F. Viajar and Celso F. Viajar.

Later, Angelica F. Viajar had Lot No. 7340 relocated and found out that the property was in the possession of Ricardo Y.
Ladrido. Consequently, she demanded its return but Ladrido refused.

On February 15, 1974, Angelica F. Viajar and Celso F. Viajar instituted a civil action for recovery of possession and damages
against Ricardo Y. Ladrido. This case was docketed as Civil Case No. 9660 of the Court of First Instance of Iloilo. Summoned to
plead, defendant Ladrido filed his answer with a counterclaim. Plaintiffs filed their reply to the answer.

Subsequently, the complaint was amended to implead Rosendo H. Te as another defendant. Plaintiffs sought the annulment of
the deed of sale and the restitution of the purchase price with interest in the event the possession of defendant Ladrido is
sustained. Defendant Te filed his answer to the amended complaint and he counter claimed for damages. Plaintiffs answered
the counterclaim.

During the pendency of the case, plaintiff Celso F. Viajar sold his rights over Lot No. 7340 to his mother and co-plaintiff, Angelica
F. Viajar. For this reason, plaintiff Angelica F. Viajar now appears to be the sole registered owner of this lot.

On May 25, 1978, defendant Ladrido died. He was substituted in the civil action by his wife, Leonor P. Ladrido, and children,
namely: Lourdes Ladrido-Ignacio, Eugenio P. Ladrido and Manuel P. Ladrido, as parties defendants.

The facts admitted by the parties during the pre-trial show that the piece of real property which used to be Lot No. 7340 of the
Cadastral Survey of Pototan was located in barangay Guibuanogan Pototan, Iloilo; that it consisted of 20,089 square meters;
that at the time of the cadastral survey in 1926, Lot No. 7511 and Lot No. 7340 were separated by the Suague River; that the
area of 11,819 square meters of what was Lot No. 7340 has been in the possession of the defendants; that the area of 14,036
square meters, which was formerly the river bed of the Suague River per cadastral survey of 1926, has also been in the
possession of the defendants; and that the plaintiffs have never been in actual physical possession of Lot No. 7340.

After trial on the merits, a second amended complaint which included damages was admitted.

The plaintiffs raised the following issues to be resolved:

1. Whether the change in the course of the Suague River was sudden as claimed by the
plaintiffs or gradual as contended by the defendants;

2. Assuming arguendo it was gradual, whether or not the plaintiffs are still entitled to Lot "B'
appearing in Exhibit "4" and to one-half (½) of Lot "A," also indicated in Exhibit "4;" and

3. Damages (pp. 12-13, Rollo).


CivRev (Property) Assignment 2
On December 10, 1981, the trial court rendered its decision, the dispositive portion of which reads:

WHEREFORE, judgment is hereby rendered in favor of the defendants and against the plaintiffs:

1. Dismissing the complaint of plaintiffs Angelica F. Viajar and Celso F. Viajar with costs
against them;

2. Declaring defendants Leonor P. Ladrido, Lourdes Ladrido-Ignacio, Eugenio P. Ladrido and


Manuel P. Ladrido as owner of the parcel of land indicated as Lots A and B in the sketch plan
(Exhs. 'C' as well as '4,' '4-B' and '4-C') situated in barangays Cawayan and Guibuanogan
Pototan, Iloilo, and containing an area of 25,855 square meters, more or less; and

3. Pronouncing that as owners of the land described in the preceding paragraph, the
defendants are entitled to the possession thereof.

Defendants' claim for moral damages and attorney's fees are dismissed.

SO ORDERED (p. 36, Rollo).

Not satisfied with the decision, the plaintiffs appealed to the Court of Appeals and assigned the following errors:

I.

THE LOWER COURT ERRED IN NOT HOLDING THAT PLAINTIFFS ARE ENTITLED TO LOT B
APPEARING IN EXHIBIT "4" AND TO ONE-HALF (½) OF LOT A IN THE SAID EXHIBIT "4."

II

THE LOWER COURT ERRED IN NOT AWARDING DAMAGES TO PLAINTIFFS (p. 42,  Rollo).

As earlier stated, the Court of Appeals affirmed the decision of the court a quo. Plaintiffs (the petitioners herein) now come to Us
claiming that the Court of Appeals palpably erred in affirming the decision of the trial court on the ground that the change in the
course of the Suague River was gradual and not sudden.

In the decision appealed from, the Court of Appeals held:

This appeal is not impressed with merit.

Article 457 of the New Civil Code provides that:

Art. 457. To the owners of lands adjoining the banks of rivers belong the accretion which they
gradually receive from the effects of the current of the waters.

The presumption is that the change in the course of the river was gradual and caused by accretion and erosion
(Martinez Canas vs. Tuason, 5 Phil. 668; Payatas Estate Improvement Co. vs. Tuason, 53 Phil. 55; C.H.
Hodges vs. Garcia, 109 Phil. 133). In the case at bar, the lower court correctly found that the evidence
introduced by the plaintiff to show that the change in the course of the Suague River was sudden or that it
occurred through avulsion is not clear and convincing.

Contrariwise, the lower court found that:

... the defendants have sufficiently established that for many years after 1926 a gradual accretion on the
eastern side of Lot No. 7511 took place by action of the current of the Suague River so that in 1979 an alluvial
deposit of 29,912 square meters (2.9912 hectares), more or less, had been added to Lot No. 7511. (Exhs. '1'
as well as Exhs. 'C' and '4'). Apropos it should be observed that the accretion consisted of Lot A with an area of
14,036 square meters; Lot B, 11,819 square meters; and Lot C, 4,057 square meters. (Exhs. '4-B,' '4-C' and '4-
D'). Only Lot C is not involved in this litigation. (See Pre-trial Order, supra)

The established facts indicate that the eastern boundary of Lot No. 7511 was the Suague River based on the
cadastral plan. For a period of more than 40 years (before 1940 to 1980) the Suague River overflowed its
banks yearly and the property of the defendant gradually received deposits of soil from the effects of the
current of the river. The consequent increase in the area of Lot No. 7511 due to alluvion or accretion was
possessed by the defendants whose tenants plowed and planted the same with coin and tobacco.

The quondam river bed had been filled by accretion through the years. The land is already plain and there is no
indication on the ground of any abandoned river bed. The river bed is definitely no longer discernible now.

What used to be the old river bed (Lot A) is in level with Lot No. 7511. So are the two other areas to the East.
(Lots B and C) Lots A, B and C are still being cultivated.
CivRev (Property) Assignment 2
Under the law, accretion which the banks or rivers may gradually receive from the effects of the current of the
waters becomes the property of the owners of the lands adjoining the banks. (Art. 366, Old Civil Code; Art. 457,
New Civil Code which took effect on August 30, 1950 [Lara v. Del Rosario, 94 Phil. 778]. Therefore, the
accretion to Lot No. 7511 which consists of Lots A and B (see Exhs. 'C' and '4') belongs to the defendants (pp.
34-35, Record on Appeal).

We find no cogent reason to disturb the foregoing finding and conclusion of the lower court.

The second assignment of error is a mere offshoot of the first assignment of error and does not warrant further
discussion (pp. 4244, Rollo).

The petition is without merit.

The petitioners contend that the first issue raised during the trial of the case on the merits in the Court of First Instance, that is,
"whether the change in the course of the Suague River was sudden as claimed by the plaintiffs or gradual as contended by the
defendants," was abandoned and never raised by them in their appeal to the Court of Appeals. Hence, the Court of Appeals, in
holding that the appeal is without merit, because of the change of the Suague River was gradual and not sudden, disposed of
the appeal on an issue that was never raised and, accordingly, its decision is void. In support of its contention, petitioners cite
the following authorities:

It is a well-known principle in procedure that courts of justice have no jurisdiction or power to decide a question
not in issue (Lim Toco vs. Go Fay, 80 Phil. 166).

A judgment going outside the issues and purporting to adjudicate something upon which the parties were not
heard, is not merely irregular, but extra-judicial and invalid ( Salvante vs. Cruz, 88 Phil. 236-244; Lazo vs.
Republic Surety & Insurance Co., Inc., 31 SCRA 329, 334).

The pivotal issue in the petitioners' appeal was whether the change in the course of the Suague River was gradual or sudden
because the trial court below resolved the same in its decision thus subjecting the same to review by respondent appellate court.
By simply abandoning this issue, the petitioners cannot hope that the affirmance of the decision wherein this issue was resolved
makes the decision of the Court of Appeals void. In effect, the petitioners are expounding a new procedural theory that to render
a questioned decision void, all that has to be done is to simply abandon on appeal the pivotal issue as resolved by the lower
court and when its decision is affirmed on appeal, attack the decision of the appellate court as void on the principle that a court
of justice has no jurisdiction or power to decide the question not in issue. This is not correct. Even the authorities cited by the
petitioners, more specifically the Salvante and Lazo cases, supra, do not support their contention. They were heard in the trial
court and they cannot complain that the proceeding below was irregular and hence, invalid.

The trial court found that the change in the course of the Suague River was gradual and this finding was affirmed by the
respondent Court of Appeals. We do not find any valid reason to disturb this finding of fact.

Article 457 of the New Civil Code (reproduced from Article 366 of the Old), the law applied by the courts a quo provides:

Art. 457. To the owners of the lands adjoining the banks of rivers belong the accretion which they gradually
receive from the effects of the current of the waters.

Petitioners contend that this article must be read together with Sections 45 an 46 of Act No. 496 which provides:

SEC. 45. 1 The obtaining of a decree of registration and the entry of a certificate of title shall be regarded as an
agreement running with the land, and binding upon the applicant and all successors in title that the land shall
be and always remain registered land, and subject to the provisions of this Act and all Acts amendatory thereof.

SEC. 46. 2 No title to registered land in derogation to that of the registered owner shall be acquired by
prescription or adverse possession.

As a result, petitioners contend, Article 457 of the New Civil Code must be construed to limit the accretion mentioned therein as
accretion of unregistered land to the riparian owner, and should not extend to registered land. Thus, the lot in question having
remained the registered land of the petitioners, then the private respondents cannot acquire title there in derogation to that of the
petitioners, by accretion, for that will defeat the indefeasibility of a Torrens Title.

The rule that registration under the Torrens System does not protect the riparian owner against the diminution of the area of his
registered land through gradual changes in the course of an adjoining stream is well settled. In Payatas Estate Improvement Co.
vs. Tuason,  53 Phil. 55, We ruled:

The controversy in the present cases seems to be due to the erroneous conception that Art. 366 of the Civil
Code does not apply to Torrens registered land. That article provides that "any accretions which the banks of
rivers may gradually receive from the effects of the current belong to the owners of the estates bordering
thereon." Accretions of that character are natural incidents to land bordering on running streams and are not
affected by the registration laws. It follows that registration does not protect the riparian owner against
diminution of the area of his land through gradual changes in the course of the adjoining stream.

In C.N. Hodges vs. Garcia, 109 Phil. 133, We also ruled:


CivRev (Property) Assignment 2
It clearly appearing that the land in question has become part of defendant's estate as a result of accretion, it
follows that said land now belongs to him. The fact that the accretion to his land used to pertain to plaintiffs
estate, which is covered by a Torrens Certificate of Title, cannot preclude him (defendant) from being the owner
thereof. Registration does not protect the riparian owner against the diminution of the area of his land through
gradual changes in the course of the adjoining stream. Accretions which the banks of rivers may gradually
receive from the effect of the current become the property of the owners of the banks (Art. 366 of the Old Civil
Code; Art. 457 of the New). Such accretions are natural incidents to land bordering on running streams and the
provisions of the Civil Code in that respect are not affected by the Registration Act.

We find no valid reason to review and abandon the aforecited rulings.

As the private respondents are the owners of the premises in question, no damages are recoverable from them.

ACCORDINGLY, the petition is DISMISSED for lack of merit without pronouncement as to costs.

SO ORDERED.
CivRev (Property) Assignment 2
Republic of the Philippines
SUPREME COURT
Manila

SECOND DIVISION

G.R. No. L-43346             March 20, 1991

MARIO C. RONQUILLO, petitioner
vs.
THE COURT OF APPEALS, DIRECTOR OF LANDS, DEVELOPMENT BANK OF THE PHILIPPINES, ROSENDO DEL
ROSARIO, AMPARO DEL ROSARIO and FLORENCIA DEL ROSARIO, respondents.*

Angara, Abello, Concepcion, Regala & Cruz for petitioner.

REGALADO, J.:

This petition seeks the review of the decision1 rendered by respondent Court of Appeals on September 25, 1975 in CA-G.R. No.
32479-R, entitled "Rosendo del Rosario, et al., Plaintiffs-Appellees, versus Mario Ronquillo, Defendant-Appellant," affirming in
toto the judgment of the trial court, and its amendatory resolution 2 dated January 28, 1976 the dispositive portion of which reads:

IN VIEW OF THE FOREGOING, the decision of this Court dated September 25, 1975 is hereby amended in the sense
that the first part of the appealed decision is set aside, except the last portion "declaring the plaintiffs to be the rightful
owners of the dried-up portion of Estero Calubcub which is abutting plaintiffs' property," which we affirm, without
pronouncement as to costs.

SO ORDERED.

The following facts are culled from the decision of the Court of Appeals:

It appears that plaintiff Rosendo del Rosario was a registered owner of a parcel of land known as Lot 34, Block 9,
Sulucan Subdivision, situated at Sampaloc, Manila and covered by Transfer Certificate of Title No. 34797 of the
Registry of Deeds of Manila (Exhibit "A"). The other plaintiffs Florencia and Amparo del Rosario were daughters of said
Rosendo del Rosario. Adjoining said lot is a dried-up portion of the old Estero Calubcub occupied by the defendant
since 1945 which is the subject matter of the present action.

Plaintiffs claim that long before the year 1930, when T.C.T. No. 34797 over Lot No. 34 was issued in the name of
Rosendo del Rosario, the latter had been in possession of said lot including the adjoining dried-up portion of the old
Estero Calubcub having bought the same from Arsenio Arzaga. Sometime in 1935, said titled lot was occupied by
Isabel Roldan with the tolerance and consent of the plaintiff on condition that the former will make improvements on the
adjoining dried-up portion of the Estero Calubcub. In the early part of 1945 defendant occupied the eastern portion of
said titled lot as well as the dried-up portion of the old Estero Calubcub which abuts plaintiffs' titled lot. After a relocation
survey of the land in question sometime in 1960, plaintiffs learned that defendant was occupying a portion of their land
and thus demanded defendant to vacate said land when the latter refused to pay the reasonable rent for its occupancy.
However, despite said demand defendant refused to vacate.

Defendant on the other hand claims that sometime before 1945 he was living with his sister who was then residing or
renting plaintiffs' titled lot. In 1945 he built his house on the disputed dried-up portion of the Estero Calubcub with a
small portion thereof on the titled lot of plaintiffs. Later in 1961, said house was destroyed by a fire which prompted him
to rebuild the same. However, this time it was built only on the called up portion of the old Estero Calubcub without
touching any part of plaintiffs titled land. He further claims that said dried-up portion is a land of public domain. 3

Private respondents Rosendo, Amparo and Florencia, all surnamed del Rosario (Del Rosarios), lodged a complaint with the
Court of First Instance of Manila praying, among others, that they be declared the rightful owners of the dried-up portion of
Estero Calubcub. Petitioner Mario Ronquillo (Ronquillo) filed a motion to dismiss the complaint on the ground that the trial court
had no jurisdiction over the case since the dried-up portion of Estero Calubcub is public land and, thus, subject to the disposition
of the Director of Lands. The Del Rosarios opposed the motion arguing that since they are claiming title to the dried-up portion of
Estero Calubcub as riparian owners, the trial court has jurisdiction. The resolution of the motion to dismiss was deferred until
after trial on the merits.

Before trial, the parties submitted the following stipulation of facts:

1. That the plaintiffs are the registered owners of Lot 34, Block 9, Sulucan Subdivision covered by Transfer Certificate of
Title No. 34797;

2. That said property of the plaintiffs abuts and is adjacent to the dried-up river bed of Estero Calubcub Sampaloc,
Manila;

3. That defendant Mario Ronquillo has no property around the premises in question and is only claiming the dried-up
portion of the old Estero Calubcub, whereon before October 23, 1961, the larger portion of his house was constructed;
CivRev (Property) Assignment 2
4. That before October 23, 1961, a portion of defendant's house stands (sic) on the above-mentioned lot belonging to
the plaintiffs;

5. That the plaintiffs and defendant have both filed with the Bureau of Lands miscellaneous sales application for the
purchase of the abandoned river bed known as Estero Calubcub and their sales applications, dated August 5, 1958 and
October 13, 1959, respectively, are still pending action before the Bureau of Lands;

6. That the parties hereby reserve their right to prove such facts as are necessary to support their case but not covered
by this stipulation of facts.4

On December 26, 1962, the trial court rendered judgment the decretal portion of which provides:

WHEREFORE, judgment is hereby rendered ordering the defendant to deliver to the plaintiffs the portion of the land
covered by Transfer Certificate of title No. 34797 which is occupied by him and to pay for the use and occupation of
said portion of land at the rate of P 5.00 a month from the date of the filing of the complaint until such time as he
surrenders the same to the plaintiffs and declaring plaintiffs to be the owners of the dried-up portion of estero Calubcub
which is abutting plaintiffs' property.

With costs to the defendant.

SO ORDERED.5

On appeal, respondent court, in affirming the aforequoted decision of the trial court, declared that since Estero Calubcub had
already dried-up way back in 1930 due to the natural change in the course of the waters, under Article 370 of the old Civil Code
which it considers applicable to the present case, the abandoned river bed belongs to the Del Rosarios as riparian owners.
Consequently, respondent court opines, the dried-up river bed is private land and does not form part of the land of the public
domain. It stated further that "(e)ven assuming for the sake of argument that said estero did not change its course but merely
dried up or disappeared, said dried-up estero would still belong to the riparian owner," citing its ruling in the case of Pinzon vs.
Rama.6

Upon motion of Ronquillo, respondent court modified its decision by setting aside the first portion of the trial court's decision
ordering Ronquillo to surrender to the Del Rosarios that portion of land covered by Transfer Certificate of Title No. 34797
occupied by the former, based on the former's representation that he had already vacated the same prior to the commencement
of this case. However, respondent court upheld its declaration that the Del Rosarios are the rightful owners of the dried-up river
bed. Hence, this petition.

On May 17, 1976, this Court issued a resolution7 requiring the Solicitor General to comment on the petition in behalf of the
Director of Lands as an indispensable party in representation of the Republic of the Philippines, and who, not having been
impleaded, was subsequently considered impleaded as such in our resolution of September 10, 1976. 8 In his Motion to Admit
Comment,9 the Solicitor General manifested that pursuant to a request made by this office with the Bureau of Lands to conduct
an investigation, the Chief of the Legal Division of the Bureau sent a communication informing him that the records of his office
"do not show that Mario Ronquillo, Rosendo del Rosario, Amparo del Rosario or Florencia del Rosario has filed any public land
application covering parcels of land situated at Estero Calubcub Manila as verified by our Records Division.

The position taken by the Director of Lands in his Comment10 filed on September 3, 1978, which was reiterated in the Reply
dated May 4, 1989 and again in the Comment dated August 17, 1989, explicates:

5. We do not see our way clear to subscribe to the ruling of the Honorable Court of Appeals on this point for Article 370
of the Old Civil Code, insofar as ownership of abandoned river beds by the owners of riparian lands are concerned,
speaks only of a situation where such river beds were abandoned  because of a natural change in the course of the
waters. Conversely, we submit that if the abandonment was for some cause other than the natural change in the course
of the waters, Article 370 is not applicable and the abandoned bed does not lose its character as a property of public
dominion not susceptible to private ownership in accordance with Article 502 (No. 1) of the New Civil Code. In the
present case, the drying up of the bed, as contended by the petitioner, is clearly caused by human activity and
undeniably not because of the natural change of the course of the waters (Emphasis in the original text).

In his Comment11 dated August 17, 1989, the Director of Lands further adds:

8. Petitioner herein and the private respondents, the del Rosarios, claim to have pending sales application(s) over the
portion of the dried up Estero Calubcub, as stated in pages 4-5, of the Amended Petition.

9. However, as stated in the Reply dated May 4, 1989 of the Director of Lands, all sales application(s) have been
rejected by that office because of the objection interposed by the Manila City Engineer's Office that they need the dried
portion of the estero for drainage purposes.

10. Furthermore, petitioner and private respondents, the del Rosarios having filed said sales application(s) are now
estopped from claiming title to the Estero Calubcub (by possession for petitioner and by accretion for respondents del
Rosarios) because for (sic) they have acknowledged that they do not own the land and that the same is a public land
under the administration of the Bureau of Lands (Director of Lands vs. Santiago, 160 SCRA 186, 194).

In a letter dated June 29, 197912 Florencia del Rosario manifested to this Court that Rosendo, Amparo and Casiano del Rosario
have all died, and that she is the only one still alive among the private respondents in this case.
CivRev (Property) Assignment 2
In a resolution dated January 20, 1988,13 the Court required petitioner Ronquillo to implead one Benjamin Diaz pursuant to the
former's manifestation14 that the land adjacent to the dried up river bed has already been sold to the latter, and the Solicitor
General was also required to inquire into the status of the investigation being conducted by the Bureau of Lands. In compliance
therewith, the Solicitor General presented a letter from the Director of Lands to the effect that neither of the parties involved in
the present case has filed any public land application.15

On April 3, 1989, petitioner filed an Amended Petition for Certiorari,16 this time impleading the Development Bank of the
Philippines (DBP) which subsequently bought the property adjacent to the dried-up river bed from Benjamin Diaz. In its
resolution dated January 10, 1990,17 the Court ordered that DBP be impleaded as a party respondent.

In a Comment18 filed on May 9, 1990, DBP averred that "[c]onsidering the fact that the petitioner in this case claims/asserts no
right over the property sold to Diaz/DBP by the del Rosarios; and considering, on the contrary, that Diaz and DBP claims/asserts
(sic) no right (direct or indirect) over the property being claimed by Ronquillo (the dried-up portion of Estero Calubcub), it follows,
therefore, that the petitioner Ronquillo has no cause of action against Diaz or DBP. A  fortiori from the viewpoint of the classical
definition of a cause of action, there is no legal justification to implead DBP as one of the respondents in this petition." DBP
thereafter prayed that it be dropped in the case as party respondent.

On September 13, 1990, respondent DBP filed a Manifestation/Compliance 19 stating that DBP's interest over Transfer Certificate
of Title No. 139215 issued in its name (formerly Transfer Certificate of Title No. 34797 of the Del Rosarios and Transfer
Certificate of Title No. 135170 of Benjamin Diaz) has been transferred to Spouses Victoriano and Pacita A. Tolentino pursuant to
a Deed of Sale dated September 11, 1990.

Petitioner Ronquillo avers that respondent Court of Appeals committed an error of law and gross abuse of discretion, acted
arbitrarily and denied petitioner due process of law (a) when it declared private respondents Del Rosarios the rightful owners of
the dried-up portion of Estero Calubcub by unduly relying upon decisional law in the case of Pinzon vs. Rama, ante, which case
was decided entirely on a set of facts different from that obtaining in this case; and (b) when it ignored the undisputed facts in
the present case and declared the dried-up portion of Estero Calubcub as a private property.

The main issue posed for resolution in this petition is whether the dried-up portion of Estero Calubcub being claimed by herein
petitioner was caused by a natural change in the course of the waters; and, corollary thereto, is the issue of the applicability of
Article 370 of the old Civil Code.

Respondent court, in affirming the findings of the trial court that there was a natural change in the course of Estero Calubcub
declared that:

The defendant claims that Article 370 of the old Civil Code is not applicable to the instant case because said Estero
Calubcub did not actually change its course but simply dried up, hence, the land in dispute is a land of public domain
and subject to the disposition of the Director of Land(s). The contention of defendant is without merit. As mentioned
earlier, said estero as shown by the relocation plan (Exhibit "D") did not disappear but merely changed its course by a
more southeasternly (sic) direction. As such, "the abandoned river bed belongs to the plaintiffs-appellees and said land
is private and not public in nature. Hence, further, it is not subject to a Homestead Application by the appellant." (Fabian
vs. Paculan CA-G.R. Nos. 21062-63-64-R, Jan. 25 1962). Even assuming for the sake of argument that said estero did
not change its course but merely dried up or disappeared, said dried-up estero would still belong to the riparian owner
as held by this Court in the case of Pinzon vs. Rama (CA-G.R. No. 8389, Jan. 8, 1943; 2 O.G. 307).20

Elementary is the rule that the jurisdiction of the Supreme Court in cases brought to it from the Court of Appeals in a petition
for certiorari under Rule 45 of the Rules of Court is limited to the review of errors of law, and that said appellate court's finding of
fact is conclusive upon this Court. However, there are certain exceptions, such as (1) when the conclusion is a finding grounded
entirely on speculation, surmises or conjectures; (2) when the inference made is manifestly absurd, mistaken or impossible; (3)
when there is grave abuse of discretion in the appreciation of facts; (4) when the judgment is premised on a misapprehension of
facts; (5) when the findings of fact are conflicting; and (6) when the Court of Appeals in making its findings went beyond the
issues of the case and the same is contrary to the admissions of both appellant and
appellee.21

A careful perusal of the evidence presented by both parties in the case at bar will reveal that the change in the course of Estero
Calubcub was caused, not by natural forces, but due to the dumping of garbage therein by the people of the surrounding
neighborhood. Under the circumstances, a review of the findings of fact of respondent court thus becomes imperative.

Private respondent Florencia del Rosario, in her testimony, made a categorical statement which in effect admitted that Estero
Calubcub changed its course because of the garbage dumped therein, by the inhabitants of the locality, thus:

Q When more or less what (sic) the estero fully dried up?

A By 1960 it is (sic) already dried up except for a little rain that accumulates on the lot when it rains.

Q How or why did the Estero Calubcub dried (sic) up?

A It has been the dumping place of the whole neighborhood. There is no street, they dumped all the garbage there. It is
the dumping place of the whole community, sir. 22

In addition, the relocation plan (Exhibit "D") which also formed the basis of respondent court's ruling, merely reflects the change
in the course of Estero Calubcub but it is not clear therefrom as to what actually brought about such change. There is nothing in
the testimony of lone witness Florencia del Rosario nor in said relocation plan which would indicate that the change in the
CivRev (Property) Assignment 2
course of the estero was due to the ebb and flow of the waters. On the contrary, the aforequoted testimony of the witness belies
such fact, while the relocation plan is absolutely silent on the matter. The inescapable conclusion is that the dried-up portion of
Estero Calubcub was occasioned, not by a natural change in the course of the waters, but through the active intervention of
man.

The foregoing facts and circumstances remove the instant case from the applicability of Article 370 of the old Civil Code which
provides:

Art. 370. The beds of rivers, which are abandoned because of a natural change in the course of the waters, belong to
the owners of the riparian lands throughout the respective length of each. If the abandoned bed divided tenements
belonging to different owners the new dividing line shall be equidistant from one and the other.

The law is clear and unambiguous. It leaves no room for interpretation.1âwphi1 Article 370 applies only if there is a natural
change in the course of the waters. The rules on alluvion do not apply to man-made or artificial accretions 23 nor to accretions to
lands that adjoin canals or esteros or artificial drainage systems. 24 Considering our earlier finding that the dried-up portion of
Estero Calubcub was actually caused by the active intervention of man, it follows that Article 370 does not apply to the case at
bar and, hence, the Del Rosarios cannot be entitled thereto supposedly as riparian owners.

The dried-up portion of Estero Calubcub should thus be considered as forming part of the land of the public domain which
cannot be subject to acquisition by private ownership. That such is the case is made more evident in the letter, dated April 28,
1989, of the Chief, Legal Division of the Bureau of Lands 25 as reported in the Reply of respondent Director of Lands stating that
"the alleged application filed by Ronquillo no longer exists in its records as it must have already been disposed of as a rejected
application for the reason that other applications "covering Estero Calubcub Sampaloc, Manila for areas other than that
contested in the instant case, were all rejected by our office because of the objection interposed by the City Engineer's office
that they need the same land for drainage purposes". Consequently, since the land is to be used for drainage purposes the
same cannot be the subject of a miscellaneous sales application.

Lastly, the fact that petitioner and herein private respondents filed their sales applications with the Bureau of Lands covering the
subject dried-up portion of Estero Calubcub cannot but be deemed as outright admissions by them that the same is public land.
They are now estopped from claiming otherwise.

WHEREFORE, the decision appealed from, the remaining effective portion of which declares private respondents Del Rosarios
as riparian owners of the dried-up portion of Estero Calubcub is hereby REVERSED and SET ASIDE.

SO ORDERED.
CivRev (Property) Assignment 2
Republic of the Philippines
SUPREME COURT
Manila

SECOND DIVISION

G.R. No. 95907 April 8, 1992

JOSE REYNANTE, petitioner,
vs.
THE HONORABLE COURT OF APPEALS, THE HON. VALENTIN CRUZ, as Presiding Judge, Regional Trial Court of
Bulacan, Branch VIII, and the HEIRS OF LEONCIO CARLOS and DOLORES A. CARLOS, and HEIRS OF GORGONIO
CARLOS and CONCEPCION CARLOS, respondents.

PARAS, J.:

This is a petition for review on certiorari which seeks the reversal of: a) decision 1 of the Court of Appeals dated February 28,
1990 in CA-G.R. No. 1917 entitled "JOSE REYNANTE versus HON. VALENTIN CRUZ, Judge, RTC of Malolos, Bulacan, and
HEIRS OF LEONCIO AND DOLORES CARLOS, et al.", affirming the decision 2 of the Regional Trial Court
of Malolos, Bulacan, Branch 8, Third Judicial Region which reversed the decision 3 of the Municipal Trial Court of Meycauayan,
Bulacan, Branch 1, Third Judicial Region in Civil Case No. 1526 entitled "HEIRS OF LEONCIO CARLOS & DOLORES A.
CARLOS and HEIRS OF GORGONIO A. CARLOS & CONCEPCION CARLOS versus JOSE REYNANTE: and b) the resolution
denying the motion for reconsideration.

The facts as culled from the records of the case are as follows:

More than 50 years ago, petitioner Jose Reynante was taken as tenant by the late Don Cosme Carlos, owner and father-in-law
of herein private respondents, over a fishpond located at Barrio Liputan, Meycauayan, Bulacan with an area of 188.711 square
meters, more or less and covered by Transfer Certificate of Title No. 25618, Land Registry of Bulacan.

During the tenancy, petitioner Jose Reynante constructed a nipa hut where he and his family lived and took care of the nipa
palms (sasahan) he had planted on lots 1 and 2 covering an area of 5,096 square meters and 6,011 square meters respectively.
These lots are located between the fishpond covered by TCT No. 25618 and the Liputan (formerly Meycauayan) River.
Petitioner harvested and sold said nipa palms without interference and prohibition from anybody. Neither did the late Don
Cosme Carlos question his right to plant the nipa palms near the fishpond or to harvest and appropriate them as his own.

After the death of Don Cosme Carlos, his heirs (private respondents' predecessors-in-interest) entered into a written agreement
denominated as "SINUMPAANG SALAYSAY NG PAGSASAULI NG KARAPATAN" dated November 29, 1984 with petitioner
Jose Reynante whereby the latter for and in consideration of the sum of P200,000.00 turned over the fishpond he was tenanting
to the heirs of Don Cosme Carlos and surrendered all his rights therein as caretaker or "bantay-kasama at tagapamahala"
(Rollo, p. 77).

Pursuant to the said written agreement, petitioner surrendered the fishpond and the two huts located therein to private
respondents. Private respondents thereafter leased the said fishpond to one Carlos de la Cruz. Petitioner continued to live in the
nipa hut constructed by him on lots 1 and 2 and to take care of the nipa palms he had planted therein.

On February 17, 1988, private respondents formally demanded that the petitioner vacate said portion since according to them
petitioner had already been indemnified for the surrender of his rights as a tenant. Despite receipt thereof, petitioner refused and
failed to relinquish possession of lots 1 and 2.

Hence, on April 22, 1988, private respondents filed a complaint for forcible entry with preliminary mandatory injunction against
petitioner alleging that the latter by means of strategy and stealth, took over the physical, actual and material possession of lots
1 and 2 by residing in one of the kubos or huts bordering the Liputan River and cutting off and/or disposing of the sasa or nipa
palms adjacent thereto.

On January 10, 1989, the trial court rendered its decision dismissing the complaint and finding that petitioner had been in prior
possession of lots 1
and 2.

Private respondents appealed to the Regional Trial Court and on August 8, 1989 it rendered its decision, the dispositive portion
of which reads as follows:

WHEREFORE, this Court renders judgment in favor of the plaintiffs and against defendant and hereby reverses
the decision of the Court a quo. Accordingly, the defendant is ordered to restore possession of that piece of
land particularly described and defined as Lots 1 & 2 of the land survey conducted by Geodetic Engineer
Restituto Buan on March 2, 1983, together with the sasa or nipa palms planted thereon. No pronouncement as
to attorney's fees. Each party shall bear their respective costs of the suit.
CivRev (Property) Assignment 2
SO ORDERED. (Rollo, p. 55; Decision, p. 4).

From said decision, petitioner filed with the Court of Appeals a petition for review (Rollo, p. 30; Annex "A"). On February 28,
1990, the Court of Appeals rendered its decision, the dispositive portion of which reads as follows:

WHEREFORE, the decision of the court a quo, being consistent with law and jurisprudence, is hereby
AFFIRMED in toto. The instant petition seeking to issue a restraining order is hereby denied.

SO ORDERED. (Rollo, p. 30; Decision, p. 3).

On November 5, 1990, the Court of Appeals denied the motion for reconsideration filed by petitioner (Rollo, p. 35; Annex "B").

Hence, this petition.

In its resolution dated May 6, 1991, the Second Division of this court gave due course to the petition and required both parties to
file their respective memoranda (Rollo, p. 93).

The main issues to be resolved in this case are: a) who between the petitioner and private respondents has prior physical
possession of lots 1 and 2; and b) whether or not the disputed lots belong to private respondents as a result of accretion.

An action for forcible entry is merely a quieting process and actual title to the property is never determined. A party who can
prove prior possession can recover such possession even against the owner himself. Whatever may be the character of
his prior possession, if he has in his favor priority in time, he has the security that entitles him to remain on the property until he
is lawfully ejected by a person having a better right by accion publiciana or accion reivindicatoria (German Management &
Services, Inc. v. Court of Appeals, G.R. No. 76216, September 14, 1989, 177 SCRA 495, 498, 499). On the other hand, if a
plaintiff cannot prove prior physical possession, he has no right of action for forcible entry and detainer even if he should be the
owner of the property (Lizo v. Carandang, 73 Phil. 469 [1942]).

Hence, the Court of Appeals could not legally restore private respondents' possession over lots 1 and 2 simply because
petitioner has clearly proven that he had  prior possession over lots 1 and 2.

The evidence on record shows that petitioner was in possession of the questioned lots for more than 50 years. It is undisputed
that he was the caretaker of the fishpond owned by the late Don Cosme Carlos for more than 50 years and that he constructed a
nipa hut adjacent to the fishpond and planted nipa palms therein. This fact is bolstered by the "SINUMPAANG SALAYSAY"
executed by Epifanio Lucero (Records, p. 66), Apolonio D. Morte (Records, p. 101) and Carling Dumalay (Records, p. 103), all
of whom are disinterested parties with no motive to falsify that can be attributed to them, except their desire to tell the truth.

Moreover, an ocular inspection was conducted by the trial court dated December 2, 1988 which was attended by the parties and
their respective counsels and the court observed the following:

The Court viewed the location and the distance of the constructed nipa hut and the subject "sasahan" which
appears exists (sic) long ago, planted and stands (sic) adjacent to the fishpond and the dikes which serves (sic)
as passage way of water river of lot 1 and lot 2. During the course of the hearing, both counsel observed
muniment of title embedded on the ground which is located at the inner side of the "pilapil" separating the
fishpond from the subject "sasa" plant with a height of 20 to 25 feet from water level and during the ocular
inspection it was judicially observed that the controversial premises is beyond the titled property of the plaintiffs
but situated along the Liputan, Meycauayan River it being a part of the public domain. (Rollo, p. 51; Decision, p.
12).

On the other hand, private respondents based their claim of possession over lots 1 and 2 simply on the written agreement
signed by petitioner whereby the latter surrendered his rights over the fishpond.

Evidently, the trial court did not err when it ruled that:

An examination of the document signed by the defendant (Exhibit "B"), shows that what was surrendered to the
plaintiffs was the fishpond and not the "sasahan" or the land on which he constructed his hut where he now
lives. That is a completely different agreement in which a tenant would return a farm or a fishpond to his
landlord in return for the amount that the landlord would pay to him as a disturbance compensation. There is
nothing that indicates that the tenant was giving other matters not mentioned in a document like Exhibit "B".
Moreover, when the plaintiffs leased the fishpond to Mr. Carlos de La Cruz there was no mention that the lease
included the hut constructed by the defendant and the nipa palms planted by him (Exhibit "1"), a circumstance
that gives the impression that the nipa hut and the nipa palms were not included in the lease to Mr. de la Cruz,
which may not belong to the plaintiffs. (Rollo, p. 49; Decision, p. 9).

With regard to the second issue, it must be noted that the disputed lots involved in this case are not  included in Transfer
Certificate of Title No. 25618 as per verification made by the Forest Management Bureau, Department of Environment and
Natural Resources. That tract of land situated at Barrio Liputan, Meycauayan, Bulacan containing an area of 1.1107 hectares as
described in the plan prepared and surveyed by Geodetic Engineer Restituto Buan for Jose Reynante falls within Alienable and
Disposable Land (for fishpond development) under Project No. 15 per B.F.L.C. Map No. 3122 dated May 8, 1987 (Rollo, p. 31;
Decision, p. 2).

The respondent Court of Appeals ruled that lots 1 and 2 were created by alluvial formation and hence the property of private
respondents pursuant to Article 457 of the New Civil Code, to wit:
CivRev (Property) Assignment 2
Art. 457. To the owners of lands adjoining the banks of rivers belong the accretion which they gradually receive
from the effects of the current of the waters.

Accretion benefits a riparian owner when the following requisites are present: (1) that the deposit be gradual and imperceptible;
(2) that it resulted from the effects of the current of the water; and (c) that the land where accretion takes place is adjacent to the
bank of a river (Republic v. Court of Appeals, G.R. No. L-61647, October 12, 1984, 132 SCRA 514, cited in Agustin v.
Intermediate Appellate Court, G.R. Nos. 66075-76, July 5, 1990, 187 SCRA 218).

Granting without conceding that lots 1 and 2 were created by alluvial formation and while it is true that accretions which the
banks of rivers may gradually receive from the effect of the current become the property of the owner of the banks, such
accretion to registered land does not preclude acquisition of the additional area by another person through prescription.

This Court ruled in the case of Ignacio Grande, et al. v. Hon. Court of Appeals, et al., G.R. No. L-17652, June 30, 1962, 115 Phil.
521 that:

An accretion does not automatically become registered land just because the lot which receives such accretion
is covered by a Torrens Title. Ownership of a piece of land is one thing; registration under the Torrens system
of that ownership is another. Ownership over the accretion received by the land adjoining a river is governed by
the Civil Code. Imprescriptibility of registered land is  provided  in the registration law. Registration under the
Land Registration and Cadastral Act does not vest or give title to the land, but merely confirms and, thereafter,
protects the title already possessed by the owner, making it imprescriptible by occupation of third parties. But to
obtain this protection, the land must be placed under the operation of the registration laws, wherein certain
judicial procedures have been provided.

Assuming private respondents had acquired the alluvial deposit (the lot in question), by accretion, still their failure to register said
accretion for a period of fifty (50) years subjected said accretion to acquisition through prescription by third persons.

It is undisputed that petitioner has been in possession of the subject lots for more than fifty (50) years and unless private
respondents can show a better title over the subject lots, petitioner's possession over the property must be respected.

PREMISES CONSIDERED, the decision of the respondent Court of Appeals dated February 28, 1990 is REVERSED and SET
ASIDE and the decision of the Municipal Trial Court of Meycauayan, Bulacan, Branch I, is hereby REINSTATED.

SO ORDERED.
CivRev (Property) Assignment 2
Republic of the Philippines
SUPREME COURT
Manila

SECOND DIVISION

G.R. No. 98045 June 26, 1996

DESAMPARADO VDA. DE NAZARENO and LETICIA NAZARENO TAPIA, petitioners,


vs.
THE COURT OF APPEALS, MR. & MRS. JOSE SALASALAN, MR. & MRS. LEO RABAYA, AVELINO LABIS, HON.
ROBERTO G. HILARIO, ROLLEO I. IGNACIO, ALBERTO M. GILLERA and HON. ABELARDO G. PALAD, JR., in their
official and/or private capacities, respondents.

ROMERO, J.:p

Petitioners Desamparado Vda. de Nazareno and Leticia Nazareno Tapia challenge the decision of the Court of Appeals which
affirmed the dismissal of petitioners' complaint by the Regional Trial Court of Misamis Oriental, Branch 22. The complaint was
for annulment of the verification, report and recommendation, decision and order of the Bureau of Lands regarding a parcel of
public land.

The only issue involved in this petition is whether or not petitioners exhausted administrative remedies before having recourse to
the courts.

The subject of this controversy is a parcel of land situated in Telegrapo, Puntod, Cagayan de Oro City. Said land was formed as
a result of sawdust dumped into the dried-up Balacanas Creek and along the banks of the Cagayan river.

Sometime in 1979, private respondents Jose Salasalan and Leo Rabaya leased the subject lots on which their houses stood
from one Antonio Nazareno, petitioners' predecessor-in-interest. In the latter part of 1982, private respondents allegedly stopped
paying rentals. As a result, Antonio Nazareno and petitioners filed a case for ejectment with the Municipal Trial Court of
Cagayan de Oro City, Branch 4. A decision was rendered against private respondents, which decision was affirmed by the
Regional Trial Court of Misamis Oriental, Branch 20.

The case was remanded to the municipal trial court for execution of judgment after the same became final and executory.
Private respondents filed a case for annulment of judgment before the Regional Trial Court of Misamis Oriental, Branch 24
which dismissed the same. Antonio Nazareno and petitioners again moved for execution of judgment but private respondents
filed another case for certiorari with prayer for restraining order and/or writ of preliminary injunction with the Regional Trial Court
of Misamis Oriental, Branch 25 which was likewise dismissed. The decision of the lower court was finally enforced with the
private respondents being ejected from portions of the subject lots they occupied..

Before he died, Antonio Nazareno caused the approval by the Bureau of Lands of the survey plan designated as Plan Csd-106-
00571 with a view to perfecting his title over the accretion area being claimed by him. Before the approved survey plan could be
released to the applicant, however, it was protested by private respondents before the Bureau of Lands.

In compliance with the order of respondent District Land Officer Alberto M. Gillera, respondent Land Investigator Avelino G.
Labis conducted an investigation and rendered a report to the Regional Director recommending that Survey Plan No. MSI-10-
06-000571-D (equivalent to Lot No. 36302, Cad. 237) in the name of Antonio Nazareno, be cancelled and that private
respondents be directed to file appropriate public land applications.

Based on said report, respondent Regional Director of the Bureau of Lands Roberto Hilario rendered a decision ordering the
amendment of the survey plan in the name of Antonio Nazareno by segregating therefrom the areas occupied by the private
respondents who, if qualified, may file public land applications covering their respective portions.

Antonio Nazareno filed a motion for reconsideration with respondent Rolleo Ignacio, Undersecretary of the Department of
Natural Resources and Officer-in-Charge of the Bureau of Lands who denied the motion. Respondent Director of Lands
Abelardo Palad then ordered him to vacate the portions adjudicated to private respondents and remove whatever improvements
they have introduced thereon. He also ordered that private respondents be placed in possession thereof.

Upon the denial of the late Antonio Nazareno's motion for reconsideration, petitioners Desamparado Vda. de Nazareno and
Leticia Tapia Nazareno, filed a case before the RTC, Branch 22 for annulment of the following: order of investigation by
respondent Gillera, report and recommendation by respondent Labis, decision by respondent Hilario, order by respondent
Ignacio affirming the decision of respondent Hilario and order of execution by respondent Palad. The RTC dismissed the
complaint for failure to exhaust administrative remedies which resulted in the finality of the administrative decision of the Bureau
of Lands.

On appeal, the Court of Appeals affirmed the decision of the RTC dismissing the complaint. Applying Section 4 of C.A. No. 141,
as amended, it contended that the approval of the survey plan belongs exclusively to the Director of Lands. Hence, factual
findings made by the Metropolitan Trial Court respecting the subject land cannot be held to be controlling as the preparation and
CivRev (Property) Assignment 2
approval of said survey plans belong to the Director of Lands and the same shall be conclusive when approved by the Secretary
of Agriculture and Natural resources. 1

Furthermore, the appellate court contended that the motion for reconsideration filed by Antonio Nazareno cannot be considered
as an appeal to the Office of the Secretary of Agriculture and Natural Resources, as mandated by C.A. No. 141 inasmuch as the
same had been acted upon by respondent Undersecretary Ignacio in his capacity as Officer-in-charge of the Bureau of Lands
and not as Undersecretary acting for the Secretary of Agriculture and Natural Resources. For the failure of Antonio Nazareno to
appeal to the Secretary of Agriculture and Natural Resources, the present case does not fall within the exception to the doctrine
of exhaustion of administrative remedies. It also held that there was no showing of oppressiveness in the manner in which the
orders were issued and executed..

Hence, this petition.

Petitioners assign the following errors:

I. PUBLIC RESPONDENT COURT OF APPEALS IN A WHIMSICAL, ARBITRARY AND CAPRICIOUS


MANNER AFFIRMED THE DECISION OF THE LOWER COURT WHICH IS CONTRARY TO THE
PREVAILING FACTS AND THE LAW ON THE MATTER;

II. PUBLIC RESPONDENT COURT OF APPEALS IN A WHIMSICAL, ARBITRARY AND CAPRICIOUS


MANNER AFFIRMED THE DECISION OF THE LOWER COURT DISMISSING THE ORIGINAL CASE WHICH
FAILED TO CONSIDER THAT THE EXECUTION ORDER OF PUBLIC RESPONDENT ABELARDO G.
PALAD, JR., DIRECTOR OF LANDS, MANILA, PRACTICALLY CHANGED THE DECISION OF PUBLIC
RESPONDENT ROBERTO HILARIO, REGIONAL DIRECTOR, BUREAU OF LANDS, REGION 10, THUS
MAKING THE CASE PROPER SUBJECT FOR ANNULMENT WELL WITHIN THE JURISDICTION OF THE
LOWER COURT.

The resolution of the above issues, however, hinges on the question of whether or not the subject land is public land. Petitioners
claim that the subject land is private land being an accretion to his titled property, applying Article 457 of the Civil Code which
provides:

To the owners of lands adjoining the banks of rivers belong the accretion which they gradually receive from the
effects of the current of the waters.

In the case of Meneses v. CA, 2 this Court held that accretion, as a mode of acquiring property under Art. 457 of the Civil Code,
requires the concurrence of these requisites : (1) that the deposition of soil or sediment be gradual and imperceptible; (2) that it
be the result of the action of the waters of the river (or sea); and (3) that the land where accretion takes place is adjacent to the
banks of rivers (or the sea coast). These are called the rules on alluvion which if present in a case, give to the owners of lands
adjoining the banks of rivers or streams any accretion gradually received from the effects of the current of waters.

For petitioners to insist on the application of these rules on alluvion to their case, the above-mentioned requisites must be
present. However, they admit that the accretion was formed by the dumping of boulders, soil and other filling materials on
portions of the Balacanas Creek and the Cagayan River bounding their land. 3 It cannot be claimed, therefore, that the
accumulation of such boulders, soil and other filling materials was gradual and imperceptible, resulting from the action of the
waters or the current of the Balacanas Creek and the Cagayan River. In Hilario v. City of Manila, 4 this Court held that the word
"current" indicates the participation of the body of water in the ebb and flow of waters due to high and low tide. Petitioners'
submission not having met the first and second requirements of the rules on alluvion, they cannot claim the rights of a riparian
owner.

In any case, this court agrees with private respondents that petitioners are estopped from denying the public character of the
subject land, as well as the jurisdiction of the Bureau of Lands when the late Antonio Nazareno filed his Miscellaneous Sales
Application MSA (G-6) 571. 5 The mere filing of said Application constituted an admission that the land being applied for was
public land, having been the subject of Survey Plan No. MSi-10-06-000571-D (Equivalent to Lot No. 36302, Cad-237) which was
conducted as a consequence of Antonio Nazareno's Miscellaneous Sales Application wherein said land was described as an
orchard. Said description by Antonio Nazareno was, however, controverted by respondent Labis in his investigation report to
respondent Hilario based on the findings of his ocular inspection that said land actually covers a dry portion of Balacanas Creek
and a swampy portion of Cagayan River. The investigation report also states that, except for the swampy portion which is fully
planted to nipa palms, the whole area is fully occupied by a part of a big concrete bodega of petitioners and several residential
houses made of light materials, including those of private respondents which were erected by themselves sometime in the early
part of 1978. 6

Furthermore, the Bureau of Lands classified the subject land as an accretion area which was formed by deposits of sawdust in
the Balacanas Creek and the Cagayan river, in accordance with the ocular inspection conducted by the Bureau of Lands. 7 This
Court has often enough held that findings of administrative agencies which have acquired expertise because their jurisdiction is
confined to specific matters are generally accorded not only respect but even finality. 8 Again, when said factual findings are
affirmed by the Court of Appeals, the same are conclusive on the parties and not reviewable by this Court. 9

It is this Court's irresistible conclusion, therefore, that the accretion was man-made or artificial. In Republic v. CA, 10 this Court
ruled that the requirement that the deposit should be due to the effect of the current of the river is indispensable. This excludes
from Art. 457 of the Civil Code all deposits caused by human intervention. Putting it differently, alluvion must be the exclusive
work of nature. Thus, in Tiongco v. Director of Lands, et al., 11 where the land was not formed solely by the natural effect of the
water current of the river bordering said land but is also the consequence of the direct and deliberate intervention of man, it was
deemed a man-made accretion and, as such, part of the public domain.
CivRev (Property) Assignment 2
In the case at bar, the subject land was the direct result of the dumping of sawdust by the Sun Valley Lumber Co. consequent to
its sawmill
operations. 12 Even if this Court were to take into consideration petitioners' submission that the accretion site was the result of
the late Antonio Nazareno's labor consisting in the dumping of boulders, soil and other filling materials into the Balacanas Creek
and Cagayan River bounding his land, 13 the same would still be part of the public domain.

Having determined that the subject land is public land, a fortiori, the Bureau of Lands, as well as the Office of the Secretary of
Agriculture and Natural Resources have jurisdiction over the same in accordance with the Public Land Law. Accordingly, the
court a quo  dismissed petitioners' complaint for non-exhaustion of administrative remedies which ruling the Court of Appeals
affirmed.

However, this Court agrees with petitioners that administrative remedies have been exhausted. Petitioners could not have
intended to appeal to respondent Ignacio as an Officer-In-Charge of the Bureau of Lands. The decision being appealed from
was the decision of respondent Hilario who was the Regional Director of the Bureau of Lands. Said decision was made "for and
by authority of the Director of Lands". 14 It would be incongruous to appeal the decision of the Regional Director of the Bureau of
Lands acting for the Director of the Bureau of Lands to an Officer-In-Charge of the Bureau of Lands.

In any case, respondent Rolleo Ignacio's official designation was "Undersecretary of the Department of Agriculture and Natural
Resources." He was only an "Officer-In-Charge" of the Bureau of Lands. When he acted on the late Antonio Nazareno's motion
for reconsideration by affirming or adopting respondent Hilario's decision, he was acting on said motion as an Undersecretary on
behalf of the Secretary of the Department. In the case of Hamoy v. Secretary of Agriculture and Natural Resources, 15 this Court
held that the Undersecretary of Agriculture and Natural Resources may modify, adopt, or set aside the orders or decisions of the
Director of Lands with respect to questions involving public lands under the administration and control of the Bureau of Lands
and the Department of Agriculture and Natural Resources. He cannot, therefore, be said to have acted beyond the bounds of his
jurisdiction under Sections 3, 4 and 5 of Commonwealth Act No. 141 16

As borne out by the administrative findings, the controverted land is public land, being an artificial accretion of sawdust. As such,
the Director of Lands has jurisdiction, authority and control over the same, as mandated under Sections 3 and 4 of the Public
Land Law (C.A. No. 141) which states, thus:

Sec. 3. The Secretary of Agriculture and Natural Resources shall be the exclusive officer charged with carrying
out the provisions of this Act through the Director of Lands who shall act under his immediate control.

Sec. 4. Subject to said control, the Director of Lands shall have direct executive control of the survey,
classification, lease, sale or any other form of concession or disposition and management of the lands of the
public domain, and his decisions as to questions of fact shall be conclusive when approved by the Secretary of
Agriculture and Natural Resources.

In connection with the second issue, petitioners ascribe whim, arbitrariness or capriciousness in the execution order of public
respondent Abelardo G. Palad, the Director of Lands. This Court finds otherwise since said decision was based on the
conclusive finding that the subject land was public land. Thus, this Court agrees with the Court of Appeals that the Director of
Lands acted within his rights when he issued the assailed execution order, as mandated by the aforecited provisions.

Petitioners' allegation that respondent Palad's execution order directing them to vacate the subject land practically changed
respondent Hilario's decision is baseless. It is incorrect for petitioners to assume that respondent Palad awarded portions of the
subject land to private respondents Salasalans and Rabayas as they had not yet been issued patents or titles over the subject
land. The execution order merely directed the segregation of petitioners' titled lot from the subject land which was actually being
occupied by private respondents before they were ejected from it. Based on the finding that private respondents were actually in
possession or were actually occupying the subject land instead of petitioners, respondent Palad, being the Director of Lands and
in the exercise of his administrative discretion, directed petitioners to vacate the subject land on the ground that private
respondents have a preferential right, being the occupants thereof.

While private respondents may not have filed their application over the land occupied by them, they nevertheless filed their
protest or opposition to petitioners' Miscellaneous Sales Application, the same being preparatory to the filing of an application as
they were in fact directed to do so. In any case, respondent Palad's execution order merely implements respondent Hilario's
order. It should be noted that petitioners' own application still has to be given due course. 17

As Director of Lands, respondent Palad is authorized to exercise executive control over any form of concession, disposition and
management of the lands of the public domain. 18 He may issue decisions and orders as he may see fit under the circumstances
as long as they are based on the findings of fact.

In the case of Calibo v. Ballesteros, 19 this Court held that where, in the disposition of public lands, the Director of Lands bases
his decision on the evidence thus presented, he clearly acts within his jurisdiction, and if he errs in appraising the evidence, the
error is one of judgment, but not an act of grave abuse of discretion annullable by certiorari. Thus, except for the issue of non-
exhaustion of administrative remedies, this Court finds no reversible error nor grave abuse of discretion in the decision of the
Court of Appeals.

WHEREFORE, the petition is DISMISSED for lack of merit.

SO ORDERED.
CivRev (Property) Assignment 2
SECOND DIVISION

G.R. No. 116290               December 8, 2000

DIONISIA P. BAGAIPO, petitioner,
vs.
THE HON. COURT OF APPEALS and LEONOR LOZANO, respondents.

QUISUMBING, J.:

This petition assails the decision dated June 30, 1994 of the Court of Appeals affirming the dismissal by the Regional Trial Court
of Davao City, Branch 8, in Civil Case No. 555-89, of petitioner’s complaint for recovery of possession with prayer for preliminary
mandatory injunction and damages.

The undisputed facts of the case are as follows:

Petitioner Dionisia P. Bagaipo is the registered owner of Lot No. 415, a 146,900 square meter agricultural land situated in Ma-a,
Davao City under Transfer Certificate of Title No. T-15757 particularly described as follows:

…Bounded on the NE., by Lots Nos. 419 and 416; on the SE by the Davao River; on the SE., (sic) by Lots Nos. 1092 and 1091;
and on the NW., by Lots Nos. 413 and 418…1

Respondent Leonor Lozano is the owner of a registered parcel of land located across and opposite the southeast portion of
petitioner’s lot facing the Davao River. Lozano acquired and occupied her property in 1962 when his wife inherited the land from
her father who died that year.

On May 26, 1989, Bagaipo filed a complaint2 for Recovery of Possession with Mandatory Writ of Preliminary Injunction and
Damages against Lozano for: (1) the surrender of possession by Lozano of a certain portion of land measuring 29,162 square
meters which is supposedly included in the area belonging to Bagaipo under TCT No. T-15757; and (2) the recovery of a land
area measuring 37,901 square meters which Bagaipo allegedly lost when the Davao River traversed her property. Bagaipo
contended that as a result of a change in course of the said river, her property became divided into three lots, namely: Lots 415-
A, 415-B and 415-C.

In January 1988, Bagaipo commissioned a resurvey of Lot 415 and presented before the trial court a survey plan 3 prepared by
Geodetic Engineer Gersacio A. Magno. The survey plan allegedly showed that: a) the area presently occupied by Bagaipo,
identified as Lot 415-A, now had an area of only 79,843 square meters; b) Lot 415-B, with an area measuring 37,901 square
meters, which cut across Bagaipo’s land was taken up by the new course of the Davao River; and c) an area of 29,162 square
meters designated as Lot 415-C was illegally occupied by respondent Lozano. The combined area of the lots described by
Engineer Magno in the survey plan tallied with the technical description of Bagaipo’s land under TCT No. T-15757. Magno
concluded that the land presently located across the river and parallel to Bagaipo’s property still belonged to the latter and not to
Lozano, who planted some 350 fruit-bearing trees on Lot 415-C and the old abandoned river bed.

Bagaipo also presented Godofredo Corias, a former barangay captain and long-time resident of Ma-a to prove her claim that the
Davao River had indeed changed its course. Corias testified that the occurrence was caused by a big flood in 1968 and a
bamboo grove which used to indicate the position of the river was washed away. The river which flowed previously in front of a
chapel located 15 meters away from the riverbank within Bagaipo’s property now flowed behind it. Corias was also present when
Magno conducted the relocation survey in 1988.

For his part, Lozano insisted that the land claimed by Bagaipo is actually an accretion to their titled property. He asserted that
the Davao River did not change its course and that the reduction in Bagaipo’s domain was caused by gradual erosion due to the
current of the Davao River. Lozano added that it is also because of the river’s natural action that silt slowly deposited and added
to his land over a long period of time. He further averred that this accretion continues up to the present and that registration
proceedings instituted by him over the alluvial formation could not be concluded precisely because it continued to increase in
size.

Lozano presented three witnesses: Atty. Pedro Castillo, his brother-in-law; Cabitunga Pasanday, a tenant of Atty. Castillo; and
Alamin Catucag, a tenant of the Lozanos.

Atty. Castillo testified that the land occupied by the Lozanos was transferred to his sister, Ramona when they extra-judicially
partitioned their parents’ property upon his father’s death. On September 9, 1973, Atty. Castillo filed a land registration case
involving the accretion which formed on the property and submitted for this purpose, a survey plan 4 approved by the Bureau of
Lands as well as tax declarations5 covering the said accretion. An Order of General Default6 was already issued in the land
registration case on November 5, 1975, but the case itself remained pending since the petition had to be amended to include the
continuing addition to the land area.

Mr. Cabitunga Pasanday testified that he has continuously worked on the land as tenant of the Castillos since 1925, tilling an
area of about 3 hectares. However, the land he tilled located opposite the land of the Lozanos and adjacent to the Davao River
has decreased over the years to its present size of about 1 hectare. He said the soil on the bank of the river, as well as coconut
trees he planted would be carried away each time there was a flood. This similar erosion occurs on the properties of Bagaipo
and a certain Dr. Rodriguez, since the elevation of the riverbank on their properties is higher than the elevation on Lozano’s
side.
CivRev (Property) Assignment 2
Alamin Catucag testified that he has been a tenant of the Castillos since 1939 and that the portion he occupies was given to
Ramona, Lozano’s wife. It was only 1 hectare in 1939 but has increased to 3 hectares due to soil deposits from the mountains
and river. Catucag said that Bagaipo’s property was reduced to half since it is in the curve of the river and its soil erodes and
gets carried away by river water.

On April 5, 1991, the trial court conducted an ocular inspection. It concluded that the applicable law is Article 457 7 . To the
owners of lands adjoining the banks of rivers belong the accretion which they gradually receive from the effects of the current of
the waters.7 of the New Civil Code and not Art. 4618 The reduction in the land area of plaintiff was caused by erosion and not by
a change in course of the Davao River. Conformably then, the trial court dismissed the complaint.

On appeal, the Court of Appeals affirmed the decision of the trial court and decreed as follows:

WHEREFORE, the decision appealed from is hereby affirmed, with costs against the plaintiff-appellant. 9

Hence, this appeal.

Petitioner asserts that the Court of Appeals erred in:

....NOT GIVING PROBATIVE VALUE TO THE RELOCATION SURVEY (EXHIBIT "B") PREPARED BY LICENSED
GEODETIC ENGINEER GERSACIO MAGNO. THE CASE OF "DIRECTOR OF LANDS VS. HEIRS OF JUANA
CAROLINA" 140 SCRA 396 CITED BY THE RESPONDENT COURT IN DISREGARDING EXHIBIT "B" IS NOT
APPLICABLE TO THE CASE AT BAR.

....NOT FINDING THAT ASSUMING WITHOUT ADMITTING THAT THE QUESTIONED LOT 415-C (EXHIBIT "B-1")
OCCUPIED BY RESPONDENT LEONOR LOZANO WAS THE RESULT OF AN ACCRETION, THE PRINCIPLE OF
ACCRETION CANNOT AND DOES NOT APPLY IN THE INSTANT CASE TO FAVOR SAID RESPONDENT
BECAUSE SAID LOT 415-C IS WITHIN AND FORM PART OF PETITIONER’S LAND DESCRIBED IN TCT NO. 15757
(EXHIBIT "A")

....FINDING PETITIONER GUILTY OF LACHES WHEN SHE INSTITUTED THE SUIT.

....NOT ORDERING RESPONDENT LEONOR LOZANO TO VACATE AND SURRENDER LOT 415-C IN FAVOR OF
PETITIONER AND FOR HIM TO PAY PETITIONER DAMAGES FOR ITS UNLAWFUL OCCUPATION THEREOF.

....NOT HOLDING PETITIONER ENTITLED TO THE ABANDONED RIVER BED. 10

For this Court’s resolution are the following issues: Did the trial court err in holding that there was no change in course of the
Davao River such that petitioner owns the abandoned river bed pursuant to Article 461 of the Civil Code? Did private respondent
own Lot 415-C in accordance with the principle of accretion under Article 457? Should the relocation survey prepared by a
licensed geodetic engineer be disregarded since it was not approved by the Director of Lands? Is petitioner’s claim barred by
laches?

On the first issue. The trial court and the appellate court both found that the decrease in land area was brought about by erosion
and not a change in the river’s course. This conclusion was reached after the trial judge observed during ocular inspection that
the banks located on petitioner’s land are sharp, craggy and very much higher than the land on the other side of the river.
Additionally, the riverbank on respondent’s side is lower and gently sloping. The lower land therefore naturally received the
alluvial soil carried by the river current.11 These findings are factual, thus conclusive on this Court, unless there are strong and
exceptional reasons, or they are unsupported by the evidence on record, or the judgment itself is based on a misapprehension
of facts.12 These factual findings are based on an ocular inspection of the judge and convincing testimonies, and we find no
convincing reason to disregard or disbelieve them.

The decrease in petitioner’s land area and the corresponding expansion of respondent’s property were the combined effect of
erosion and accretion respectively. Art. 461 of the Civil Code is inapplicable. Petitioner cannot claim ownership over the old
abandoned riverbed because the same is inexistent. The riverbed’s former location cannot even be pinpointed with particularity
since the movement of the Davao River took place gradually over an unspecified period of time, up to the present.

The rule is well-settled that accretion benefits a riparian owner when the following requisites are present: 1) That the deposit be
gradual and imperceptible; 2) That it resulted from the effects of the current of the water; and 3) That the land where accretion
takes place is adjacent to the bank of the river. 13 These requisites were sufficiently proven in favor of respondents. In the
absence of evidence that the change in the course of the river was sudden or that it occurred through avulsion, the presumption
is that the change was gradual and was caused by alluvium and erosion. 14

As to Lot 415-C, which petitioner insists forms part of her property under TCT No. T-15757, it is well to recall our holding in C.N.
Hodges vs. Garcia, 109 Phil. 133, 135:

… The fact that the accretion to his land used to pertain to plaintiff’s estate, which is covered by a Torrens certificate of title,
cannot preclude him (defendant) from being the owner thereof. Registration does not protect the riparian owner against the
diminution of the area of his land through gradual changes in the course of the adjoining stream. Accretions which the banks of
rivers may gradually receive from the effect of the current become the property of the owners of the banks (Art. 366 of the old
Civil Code; Art. 457 of the new). Such accretions are natural incidents to land bordering on running streams and the provisions
of the Civil Code in that respect are not affected by the Land Registration Act. 15
CivRev (Property) Assignment 2
Petitioner did not demonstrate that Lot 415-C allegedly comprising 29,162 square meters was within the boundaries of her titled
property. The survey plan commissioned by petitioner which was not approved by the Director of Lands was properly discounted
by the appellate court. In Titong vs. Court of Appeals16 we affirmed the trial court’s refusal to give probative value to a private
survey plan and held thus:

…the plan was not verified and approved by the Bureau of Lands in accordance with Sec. 28, paragraph 5 of Act No. 2259, the
Cadastral Act, as amended by Sec. 1862 of Act No. 2711. Said law ordains that private surveyors send their original field notes,
computations, reports, surveys, maps and plots regarding a piece of property to the Bureau of Lands for verification and
approval.1âwphi1 A survey plan not verified and approved by said Bureau is nothing more than a private writing, the due
execution and authenticity of which must be proven in accordance with Sec. 20 of Rule 132 of the Rules of Court. The
circumstance that the plan was admitted in evidence without any objection as to its due execution and authenticity does not
signify that the courts shall give probative value therefor. To admit evidence and not to believe it subsequently are not
contradictory to each other…

In view of the foregoing, it is no longer necessary now to discuss the defense of laches. It is mooted by the disquisition on the
foregoing issues.

WHEREFORE, the assailed decision dated June 30, 1994, of the Court of Appeals in C.A.-G. R. CV No. 37615, sustaining the
judgment of the court a quo, is AFFIRMED. Costs against petitioner.

SO ORDERED.
CivRev (Property) Assignment 2
THIRD DIVISION

[G.R. No. 142595. October 15, 2003.]

RACHEL C. CELESTIAL, Petitioner, v. JESSE CACHOPERO, Respondent.

DECISION

CARPIO MORALES, J.:

In the instant appeal by petition for review on certiorari, 1 petitioner Rachel Cachopero Celestial assails the February 15, 1999
Decision of the Court of Appeals in CA-G.R. SP No. 45927, "Jesse C. Cachopero v. Regional Executive Director of DENR,
Region XII and Rachel C. Celestial," which reversed and set aside the Order of the Regional Trial Court (RTC) of Midsayap,
Cotabato, Branch 18 dismissing respondent’s petition for certiorari, prohibition and mandamus, and mandated the Regional
Executive Director of the Department of Environment and Natural Resources (DENR), Region XII to process the Miscellaneous
Sales Application (MSA) of respondent Jesse Cachopero in DENR Claim No. XII-050-90 to which petitioner filed a
protest.chanrob1es virtua1 1aw 1ibrary

Respondent, brother of petitioner, filed an MSA (Plan No. (XII-6)-1669) with the Bureau of Lands covering a 415 square meter
parcel of land located at Barrio 8, Midsayap, Cotabato and formerly part of the Salunayan Creek in Katingawan, Midsayap.

In his MSA, respondent alleged that he had, since 1968, been occupying the land whereon he built a residential house and
introduced other improvements.

Petitioner filed a protest against respondent’s MSA, claiming preferential right over the land subject thereof since it is adjacent
to, and is the only outlet from, her residential house situated at Lot No. 2586-G-28 (LRC) Psd-105462, Poblacion 8, Midsayap.

Following an ocular inspection, the Bureau of Lands, finding the land subject of respondent’s MSA to be outside the commerce
of man, dismissed petitioner’s protest and denied respondent’s MSA, to wit:chanrob1es virtual 1aw library

In the ocular inspection, it was verified that the land in dispute with an area of 415 square meters was formerly a part of the
Salunayan Creek that became dry as a result of the construction of an irrigation canal by the National Irrigation Administration.
However, it was certified by Project Engineer Reynaldo Abeto of the said office in his certification dated May 19, 1982, that only
a portion of the same containing an area of 59.40 square meters more or less was taken as part of the National Irrigation
Administration service road. It was also ascertained that the P20,000.00 residential house wherein Jesse Cachopero and his
family are living is not within the 69-meters width of the national highway. However, per the certification of the local office of the
District Engineer for Public Works and Highways, the government may need the area where the house stands for expansion in
the future. Moreover, it was also certified by the Office of Municipal Mayor that the whole area covered by the miscellaneous
sales application of Jesse Cachopero is needed by the municipal government for future public improvements.

From the foregoing facts, it is clear that the subject land is outside the commerce of man and therefore, not susceptible of
private acquisition under the provision of the Public Land Act. However, in keeping with the policy of our compassionate society
in tilting the balance of social forces by favoring the disadvantaged in life, we may allow Jesse Cachopero to temporarily occupy
the land in dispute, after excluding therefrom the portion needed for the existing right of way being claimed by Rachel Celestial
to be [the] only adequate outlet to the public highway until such time that the land is needed by the government for expansion of
the road.

WHEREFORE, it is ordered that this case, be, as hereby it is, dismissed and this case (sic), dropped from the records. The
Miscellaneous Sales Application (New) of Jesse Cachopero is hereby rejected and in lieu thereof, he shall file a revocable
permit application for the land in question after excluding from the southern part of the land the area of five (5) meters for right of
way purposes as shown in the sketch drawn at the back of this order. The segregation survey of the area shall be at the pro-rata
expense of the parties.

SO ORDERED. 2 (Emphasis and Italics supplied)

Petitioner thereafter instituted an action for ejectment against respondent and his wife before the Municipal Trial Court of
Midsayap, Cotabato, docketed as Civil Case No. 711. A judgment based on a compromise was rendered in said case under the
following terms and conditions:chanrob1es virtual 1aw library

That Spouses Jesse Cachopero and Bema Cachopero, defendants in this case, are going to vacate the premises in question
and transfer the old house subject of this ejectment case at the back of Lot No. 2586-G-28 (LRC) Psd-105462, located at 8,
Midsayap, Cotabato, within eight (8) months from today, but not later than April 30, 1990;

x          x          x

That plaintiff is willing to give a two (2)-meter wide exit alley on the eastern portion of said lot as road-right-of-way up to the point
of the NIA road on the west of Lot No. 2586-G-28, (LRC) Psd-105462;

That defendants hereby promise to remove all their improvements introduced fronting the residence of the plaintiff before August
31, 1989; and the plaintiff shall likewise remove all her existing improvements on the same area;

x       x       x 3 (Emphasis supplied)

Subsequently or on May 21, 1991, respondent filed another MSA with the DENR Regional Office of Cotabato involving a portion
CivRev (Property) Assignment 2
of the same lot subject of his first MSA, covering an area of 334 square meters, more or less (the subject land), and docketed as
DENR-XII-Claim No. 050-90. This time, the MSA was supported by a certification 4 dated January 9, 1989 issued by the Office
of the Mayor of Midsayap and an Indorsement 5 dated January 16, 1989 by the District Engineer of the Department of Public
Works and Highways stating that the subject land is suitable for residential purposes and no longer needed by the municipal
government.

Petitioner likewise filed a protest against her brother-respondent’s second MSA, alleging a preferential right over the subject
land, she being the adjacent and riparian owner, and maintaining that it is her only access to the national highway. She thus
reiterated her demand for a five (5)-meter road right of way through the land.chanrob1es virtua1 1aw 1ibrary

After another investigation of the subject land, DENR Regional Executive Director Macorro Macumbal issued an Order dated
February 17, 1994 stating that it was suitable for residential purposes but that, in light of the conflicting interest of the parties, it
be sold at public auction. Respondent’s second MSA was accordingly dismissed, viz:chanrob1es virtual 1aw library

In the ocular investigation of the premises, it was established that the said property is a dried bed of Salunayan Creek resulting
from the construction of the irrigation canal by the National Irrigation Administration; that it is suitable for residential purpose . . .

x          x          x

It is evident that under the law, property of the public domain situated within the first (1st) to fourth class municipalities are
disposable by sales only. Since municipality of Midsayap, Cotabato is classified as third (3rd) class municipality and the property
in dispute, Lot no. (MSA-XII-6)-1669, is situated in the poblacion of Midsayap, Cotabato, and considering the conflicting interest
of the herein parties, it is therefore equitable to dispose the same by sale at a public auction pursuant to Section 67, C.A. No.
141, as amended, pertinent clause of which provides:chanrob1es virtual 1aw library

. . . sale shall be made through oral bidding; and adjudication shall be made to the highest bidder, . . .

WHEREFORE, in view of all the foregoing, it is ordered as hereby is ordered that the instant protest is dismissed and dropped
from the records, and the Miscellaneous Sales Application (New) of Jesse C. Cachopero is rejected and returned unrecorded.
Accordingly, the CENR Officer of CENRO XII-4B shall cause the segregation survey of a portion of five (5) meters in width
running parallel to line point C-1 of the approved survey plan (MSA-XII-6)-1669, sketch is shown at the dorsal side hereof, as a
permanent easement and access road for the occupants of Lot No. 2386-G-28, (LRC) Psd-105462 to the national highway.
Thereafter, and pursuant to paragraph G.2.3 of Department Administrative Order No. 38, Series of 1990, the CENRO XII 4B
shall dispose the remaining area of the lot in question through oral bidding.

SO ORDERED." 6 (Emphasis and Italics supplied)

Respondent filed a Motion for Reconsideration of the above-said order of the DENR Regional Executive Director, but it was
denied by Order of February 27, 1995 by the OIC Regional Executive Director of Region XII, Cotabato City in this
wise:chanrob1es virtual 1aw library

A meticulous scrutiny of the records disclosed that Civil Case No. 711 for ejectment, decided on the basis of compromise
agreement of the parties dated August 10, 1989, involved "transfer of the house from Lot No. MSA XII-6-1669 to the litigant’s
parents’ property situated at the back of protestant property, Lot No. 2586-G-28 (LRC), Psd-105462." Whereas the issue in
DENR XII Claim No. 050-90 involved the disposition of lot no. (MSA II-6)-1669 a residential public land being exclusively vested
with the Director of Lands (Sec. 4, C.A. 141).

The two (2) meters wide exit alley provided in the compromise agreement was established by the protestant from her private
property (Lot No. 2586-G-28 (LRC), Psd-105462) for the benefit of her brother, herein respondent, upon his transfer to their
parents property at the back of Lot No. 2586-G-28 (LRC), Psd-105462. Whereas the five (5) meters wide easement imposed on
Lot No. (MSA-XII-6)-1669, a public land, provided in the decision in DENR Claim No. 050-90 is in accordance with Article 670 of
the New Civil Code . . .

x          x          x

With all the above foregoing, we find no reversible error to reconsider our Order of February 17, 1994.

WHEREFORE, the instant motion for reconsideration is DENIED. 7

Respondent thereupon filed on April 3, 1995 with the RTC of Midsayap, Cotabato a petition for certiorari, prohibition and
mandamus with preliminary mandatory injunction and temporary restraining order assailing the Orders dated February 17, 1994
and February 27, 1995 of the DENR Regional Executive Director and OIC Regional Executive Director of Region XII, Cotabato,
attributing grave abuse of discretion in the issuance thereof.

Petitioner moved for the dismissal of the petition, alleging lack of jurisdiction and non-exhaustion of administrative remedies.

By Order of March 26, 1997, the RTC denied respondent’s petition for certiorari for lack of merit and non-exhaustion of
administrative remedies, as it did deny his motion for reconsideration.

The Court of Appeals, before which respondent assailed the RTC orders by petition for certiorari, prohibition and mandamus,
granted said petition, and accordingly reversed and set aside the assailed orders of the RTC and ordered the DENR to process
the MSA of Respondent. 8

Petitioner’s Motion for Reconsideration 9 of the appellate court’s decision having been denied by Resolution of March 2, 2000,
10 she lodged the present petition, alleging that the Court of Appeals acted contrary to law and jurisprudence 1) in holding that
CivRev (Property) Assignment 2
the RTC of Midsayap had jurisdiction over respondent’s petition, the doctrine of exhaustion of administrative remedies was not
applicable to the instant case, and the contested land is public land; and 2) in ordering the processing of respondent’s MSA
pursuant to R.A. 730. 11

Petitioner contends that the RTC of Midsayap had no jurisdiction over respondent’s petition for certiorari as (a) it "is in the nature
of an appeal" 12 falling within the jurisdiction of the Court of Appeals under Section 9(3) 13 of Batas Pambansa Blg. 129 (B.P.
129), as amended; and (b) respondent failed to exhaust administrative remedies when he failed to appeal the questioned Orders
to the Secretary of Environment and Natural Resources. 14

Petitioner’s petition fails.chanrob1es virtua1 1aw 1ibrary

Petitioner has apparently confused the separate and distinct remedies of an appeal (i.e. through a petition for review of a
decision of a quasi judicial agency under Rule 43 of the Rules of Court) and a special civil action for certiorari (i.e. through a
petition for review under Rule 65 of the Rules of Court). In Silverio v. Court of Appeals, 15 this Court, speaking through then
Chief Justice Claudio Teehankee, distinguished between these two modes of judicial review as follows:chanrob1es virtual 1aw
library

The provisions of the Rules of Court permit an aggrieved party, in the general types of cases, to take a cause and apply for relief
to the appellate courts by way of either of two distinctly different and dissimilar modes — through the broad process of appeal or
the limited special civil action of certiorari. An appeal brings up for review errors of judgment committed by a court with
jurisdiction over the subject of the suit and the persons of the parties or any such error committed by the court in the exercise of
its jurisdiction amounting to nothing more than an error of judgment. On the other hand, the writ of certiorari issues for the
correction of errors of jurisdiction only or grave abuse of discretion amounting to lack or excess of jurisdiction. The writ
of certiorari "cannot legally be used for any other purpose." In terms of its function, the writ of certiorari serves "to keep an
inferior court within the bounds of its jurisdiction or to prevent it from committing such a grave abuse of discretion amounting to
excess of jurisdiction" or to relieve parties from arbitrary acts of courts — acts which courts have no power or authority in law to
perform. 16 (Italics, emphasis and underscoring supplied)

Concomitantly, appellate jurisdiction is separate and distinct from the jurisdiction to issue the prerogative writ of certiorari. An
appellate jurisdiction refers to a process which is a continuation of the original suit and not a commencement of a new action. In
contrast, to invoke a court’s jurisdiction to issue the writ of certiorari requires the commencement of a new and original action
therefor, independent of the proceedings which gave rise to the questioned decision or order. 17 As correctly held by the Court
of Appeals, 18 the RTCs have concurrent jurisdiction with the Court of Appeals and the Supreme Court over original petitions
for certiorari, prohibition and mandamus 19 under Section 21 20 of B.P. 129.

A perusal of respondent’s Petition dated April 3, 1995 filed before the RTC clearly shows that it alleged that the DENR Regional
Executive Director and OIC Regional Executive Director acted with "grave abuse of discretion and without or in excess of
jurisdiction amounting to lack of jurisdiction" when they issued the questioned Orders dated February 17, 1994 and February 27,
1995. Evidently, respondent sought a judicial review of the questioned Orders through a special civil action for certiorari which,
as aforementioned, was within the jurisdiction of the RTC of Midsayap, Cotabato. 21

Additionally, this Court finds no reason to disturb the Court of Appeals’ conclusion that the instant case falls under the
recognized exceptions to the rule on exhaustion of administrative remedies, to wit:chanrob1es virtual 1aw library

The rule of exhaustion of administrative remedies is inapplicable if it should appear that an irreparable injury or damage will be
suffered by a party if he should await, before taking court action, the final action of the administrative official concerned on the
matter as a result of a patently illegal order (Vivo v. Cloribel, 18 SCRA 713; De Lara v. Cloribel, 14 SCRA 269); or where appeal
would not prove to be speedy and adequate remedy. 22

True, the doctrine of exhaustion of administrative remedies calls for resort first to the appropriate administrative authorities in the
resolution of a controversy falling under their jurisdiction before the same may be elevated to the courts of justice for review, and
non-observance thereof is a ground for the dismissal of the complaint, 23 the rationale being:chanrob1es virtual 1aw library

The thrust of the rule on exhaustion of administrative remedies is that the courts must allow the administrative agencies to carry
out their functions and discharge their responsibilities within the specialized areas of their respective competence. It is presumed
that an administrative agency, if afforded an opportunity to pass upon a matter, will decide the same correctly, or correct any
previous error committed in its forum. Furthermore, reasons of law, comity and convenience prevent the courts from entertaining
cases proper for determination by administrative agencies. Hence, premature resort to the courts necessarily becomes fatal to
the cause of action of the petitioner. 24

However, this requirement of prior exhaustion of administrative remedies is not absolute, there being instances when it may be
dispensed with and judicial action may be validly resorted to immediately, among which are: 1) when the question raised is
purely legal; 2) when the administrative body is in estoppel; 3) when the act complained of is patently illegal; 4) when there is
urgent need for judicial intervention; 5) when the claim involved is small; 6) when irreparable damage will be suffered; 7) when
there is no other plain, speedy and adequate remedy; 8) when strong public interest is involved; and 9) in quo warranto
proceedings.25cralaw:red

Hence, where the act complained of is patently illegal since the administrative body acted without or in excess of jurisdiction or
with such grave abuse of discretion as to be tantamount to lack of jurisdiction, as was alleged in respondent’s petition before the
RTC, prior exhaustion of administrative remedies is not required and resort to the courts through a special civil action
for certiorari under Rule 65 is permitted:chanrob1es virtual 1aw library

We hold that it was an error for the court a quo to rule that the petitioners should have exhausted its remedy of appeal from the
orders denying their application for waiver/suspension to the Board of Trustees and thereafter to the Court of Appeals pursuant
to the Rules. Certiorari is an appropriate remedy to question the validity of the challenged issuances of the HDMF which are
alleged to have been issued with grave abuse of discretion amounting to lack of jurisdiction.

Moreover, among the accepted exceptions to the rule on exhaustion of administrative remedies are: (1) where the question in
CivRev (Property) Assignment 2
dispute is purely a legal one; and (2) where the controverted act is patently illegal or was performed without jurisdiction or in
excess of jurisdiction. Moreover, while certiorari as a remedy may not be used as a substitute for an appeal, especially for a lost
appeal, this rule should not be strictly enforced if the petition is genuinely meritorious. It has been said that where the rigid
application of the rules would frustrate substantial justice, or bar the vindication of a legitimate grievance, the courts are justified
in exempting a particular case from the operation of the rules. 26 (Emphasis supplied)

To justify the issuance of the writ of certiorari, however, it must be clearly shown that there is a patent and grave abuse of
discretion amounting to an evasion of a positive duty or to a virtual refusal to perform a duty enjoined by law, or to act at all in
contemplation of law, as where the power is exercised in an arbitrary and despotic manner by reason of passion or personal
hostility. 27

The crux of the case at bar is, therefore, whether the DENR Regional Executive Director and OIC Regional Director acted with
grave abuse of discretion amounting to lack or excess of jurisdiction in issuing the questioned Orders dated February 17, 1994
and February 27, 1995, respectively.chanrob1es virtua1 1aw 1ibrary

In resolving respondent’s second MSA and petitioner’s protest thereto, the DENR Regional Executive Director, after considering
the conflicting interest of the parties, found it equitable to resolve the same by directing the sale of the subject land at public
auction pursuant to Section 67, C.A. No. 141, as amended.

Section 67 of Commonwealth Act No. 141, otherwise known as "The Public Land Act," provides the procedure for the disposition
of lands of the public domain which are open to disposition or concession and intended to be used for residential, commercial,
industrial or other productive purposes other than agricultural, to wit:chanrob1es virtual 1aw library

SEC. 67. The lease or sale shall be made through oral bidding; and adjudication shall be made to the highest bidder. However,
where an applicant has made improvements on the land by virtue of a permit issued to him by competent authority, the sale or
lease shall be made by sealed bidding as prescribed in Section twenty-six of this Act, the provisions of which shall be applied
wherever applicable. If all or part of the lots remain unleased or unsold, the Director of Lands shall from time to time announce in
the Official Gazette or in any other newspapers of general circulation, the lease or sale of those lots, if necessary. (Emphasis
supplied)

With the enactment of Republic Act No. 730 28 on June 18, 1952, however, an exception to the foregoing procedure was
created by authorizing disposition of lands of the public domain by private sale, instead of bidding, provided that: (1) the
applicant has in his favor the conditions specified therein and (2) the area applied for is not more than 1,000 square meters. 29
The pertinent provision of R.A. 730 thus provides:chanrob1es virtual 1aw library

SEC. 1. Notwithstanding the provisions of Sections 61 and 67 of Commonwealth Act No. 141, as amended by Republic Act No.
293, any Filipino citizen of legal age who is not the owner of a home lot in the municipality or city in which he resides and who
has in good faith established his residence on a parcel of the public land of the Republic of the Philippines which is not needed
for the public service, shall be given preference to purchase at a private sale of which reasonable notice shall be given to him
not more than one thousand square meters at a price to be fixed by the Director of Lands with the approval of the Secretary of
Agriculture and Natural Resources. It shall be an essential condition of this sale that the occupant has constructed his house on
the land and actually resided therein. Ten percent of the purchase price shall be paid upon the approval of the sale and the
balance may be paid in full, or in ten equal annual installments.

SEC. 2. Land acquired under the provisions of this Act shall not be subject to any restrictions against encumbrance or alienation
before and after the issuance of the patents thereon. 30

SEC. 3. The provisions of the Public Land Act with respect to the sale of lands for residential purposes which are not
inconsistent herewith shall be applicable.

SEC. 4. This Act shall take effect upon its approval.

Approved, June 18, 1952. (Emphasis supplied)

Given the foregoing provisions of R.A. 730 which took effect on June 18, 1952, and the DENR Regional Executive Director’s
February 17, 1994 finding that the subject land was "suitable for residential purposes," it was incumbent upon him to determine
whether the provisions of R.A. 730 were applicable to respondent’s MSA. As held by the Court of Appeals:chanrob1es virtual
1aw library

Finally, petitioner contends that the DENR Regional Executive Director and OIC Regional Executive Director gravely erred in
ordering the sale of the subject lot through oral bidding applying Section 67, Commonwealth Act No. 141 and not Republic Act
730 authorizing the sale of public land without bidding.

We agree with the petitioner.

x          x          x

Apropos is the case of Reyes v. Court of Appeals, 125 SCRA 785, ruling that:jgc:chanrobles.com.ph

"When public land lots of not more than 1,000 sq. ms. are used, or to be used as a residence . . . they can be sold on private
sales under the provisions of Republic Act No. 730."cralaw virtua1aw library

In Agura v. Serfino, Sr., (204 SCRA 569), the Supreme Court held that:jgc:chanrobles.com.ph

"R.A. 730 authorizes a sale by private sale, as an exception to the general rule that it should be by bidding, if the area applied for
does not exceed 1,000 square meters, . . ."cralaw virtua1aw library
CivRev (Property) Assignment 2

We see no reason why these ruling should not be applied in this case which involves 415 [should have been 334] square meters
only. 31

The Regional Director, however, summarily chose to apply Section 67 of the Public Land Act upon a finding that it was more
"equitable" in light of the "conflicting interest" of the parties. In his "Answer" to respondent’s petition before the RTC, the Director
justified his non-application of R.A. 730 in this wise:chanrob1es virtual 1aw library

. . . Republic Act No. 730 is not applicable to the case at bar, the land being disputed, Republic Act No. 730 requisite (sic) vas
not meet (sic) that for this law to apply to a particular case, the land must be in the first place not a land in conflict. There being a
pending protest for final adjudication, the said conflict continues to exist thus an impediment to the application of Republic Act
730 32 (Emphasis supplied)

which justification he reiterated in his Opposition 33 to respondent’s Motion for Reconsideration of the RTC decision.

The Director’s reliance on equity as basis for his action was misplaced, however. It is well-settled that "equity follows the law."
34 Described as "justice outside legality," it is applied only in the absence of, and never against, statutory law or legal
pronouncements. 35 Where pertinent positive rules are present, they should pre-empt and prevail over all abstract arguments
based only on equity. 36

A reading of R.A. 730 (or of the Public Land Act for that matter) shows nothing therein to support the Director’s contention that
the pendency of a protest is a bar to the application of R.A. 730 to an MSA. Indeed, that Section 1 of R.A. 730 gives a qualified
applicant preference to purchase alienable public land suitable for residential purposes implies that there may be more than one
party interested in purchasing it.chanrob1es virtua1 1aw library

What is more, under Section 91 of the Public Land Act, it is the duty of the Director of the Lands Management Bureau (formerly
the Director of Lands) to determine whether the material facts set forth in an MSA are true:chanrob1es virtual 1aw library

SEC. 91. The statements made in the application shall be considered as essential conditions and parts of any concession, title,
or permit issued on the basis of such application, and any false statement therein or omission of facts altering, changing, or
modifying the consideration of the facts set forth in such statements, and any subsequent modification, alteration, or change of
the material facts set forth in the application shall ipso facto produce the cancellation of the concession, title, or permit granted. It
shall be the duty of the Director of Lands, from time to time and whenever he may deem it advisable, to make the necessary
investigations for the purpose of ascertaining whether the material facts set out in the application are true, or whether they
continue to exist and are maintained and preserved in good faith, and for the purposes of such investigation, the Director of
Lands is hereby empowered to issue subpoenas and subpoenas duces tecum and, if necessary, to obtain compulsory process
from the courts. In every investigation made in accordance with this section, the existence of bad faith, fraud, concealment, or
fraudulent and illegal modification of essential facts shall be presumed if the grantee or possessor of the land shall refuse or fail
to obey a subpoena or subpoena duces tecum lawfully issued by the Director of Lands or his authorized delegates or agents, or
shall refuse or fail to give direct and specific answers to pertinent questions, and on the basis of such presumption, an order of
cancellation may issue without further proceedings. (Emphasis supplied)

Likewise, under Section 102 of the same Public Land Act, it is the duty of the Director of the Lands Management Bureau to, after
due hearing, verify whether the grounds of a protest or objection to an MSA are well founded, and, if so, to cancel the
MSA:chanrob1es virtual 1aw library

SEC. 102. Any person, corporation, or association may file an objection under oath to any application or concession under this
Act, grounded on any reason sufficient under this Act for the denial or cancellation of the application or the denial of the patent
or grant. If, after the applicant or grantee has been given suitable opportunity to be duly heard, the objection is found to be well
founded, the Director of Lands shall deny or cancel the application or deny patent or grant, and the person objecting shall, if
qualified, be granted a prior right of entry for a term of sixty days from the date of the notice. (Emphasis supplied)

There was thus clearly a positive duty on the part of the DENR Director to process respondent’s MSA, and to ascertain,
particularly in light of petitioner’s protest, whether respondent was qualified to purchase the subject land at a private sale
pursuant to R.A. 730. This, he did not do.

In fine, by abdicating his duty to process respondent’s MSA and summarily ordering, without factual or legal basis, that the
subject land be disposed of via oral bidding pursuant to Section 67 of the Public Land Act, the Director acted with patent grave
abuse of discretion amounting to lack or excess of jurisdiction. As the Court of Appeals held:chanrob1es virtual 1aw library

Considering that the assailed Orders of public respondent DENR Regional Executive Director applying Section 67 of
Commonwealth Act No. 141 and ordering the sale of the subject lot by oral bidding are patently erroneous, the authority of the
court to issue writs of certiorari, prohibition and mandamus is warranted. 37

The Director’s commission of grave abuse of discretion does not, however, mean that respondent automatically has the better
right to the subject land. As mandated by law, the Director must process respondent’s MSA, conduct an investigation, and
determine whether the material facts set forth therein are true to bring it within the coverage of R.A. 730.

A thorough investigation is all the more imperative considering that petitioner’s protest raises serious factual issues regarding
respondent’s qualification to purchase the subject land — in particular, whether he already owns a home lot in Midsayap and
whether he has, in good faith, constructed his house on the subject land and actually resided therein. These factual issues are
properly within the authority of the DENR and the Land Management Bureau, which are tasked with carrying out the provisions
of the Public Land Act and R.A. 730, 38 do determine, after both parties have been given an opportunity to fully present their
evidence.

As for petitioner’s claim of ownership over the subject land, admittedly a dried-up bed of the Salunayan Creek, based on (1) her
alleged long term adverse possession and that of her predecessor-in-interest, Marcelina Basadre, even prior to October 22,
1966, when she purchased the adjoining property from the latter, and (2) the right of accession under Art. 370 of the Spanish
CivRev (Property) Assignment 2
Civil Code of 1889 and/or Article 461 of the Civil Code, the same must fail.

Since property of public dominion is outside the commerce of man 39 and not susceptible to private appropriation and
acquisitive prescription, 40 the adverse possession which may be the basis of a grant of title in the confirmation of an imperfect
title refers only to alienable or disposable portions of the public domain. 41 It is only after the Government has declared the land
to be alienable and disposable agricultural land that the year of entry, cultivation and exclusive and adverse possession can be
counted for purposes of an imperfect title. 42

A creek, like the Salunayan Creek, is a recess or arm extending from a river and participating in the ebb and flow of the sea. 43
As such, under Articles 420(1) 44 and 502(1) 45 of the Civil Code, the Salunayan Creek, including its natural bed, is property of
the public domain which is not susceptible to private appropriation and acquisitive prescription. 46 And, absent any declaration
by the government, that a portion of the creek has dried-up does not, by itself, alter its inalienable character.chanrob1es virtua1
1aw 1ibrary

This, in fact, was the very reason behind the denial of respondent’s first MSA, the District Engineer having certified that the
government may need the subject land for future expansion, and the office of the Municipal Mayor having certified that it was
needed by t he municipal government for future public improvements. 47 Consequently, it was only after the same offices
subsequently certified 48 that the subject land was suitable for residential purposes and no longer needed by the municipal
government that it became alienable and disposable. Confronted with similar factual circumstances, this Court in Bracewell v.
Court of Appeals 49 held:chanrob1es virtual 1aw library

Clear from the above is the requirement that the applicant must prove that the land is alienable public land. On this score, we
agree with respondents that petitioner failed to show that the parcels of land subject of his application are alienable or
disposable. On the contrary, it was conclusively shown by the government that the same were only classified as alienable or
disposable on March 27, 1972. Thus, even granting that petitioner and his predecessors-in-interest had occupied the same
since 1908, he still cannot claim title thereto by virtue of such possession since the subject parcels of land were not yet alienable
land at that time nor capable of private appropriation. The adverse possession which may be the basis of a grant of title or
confirmation of an imperfect title refers only to alienable or disposable portions of the public domain. 50 (Emphasis supplied)

With respect to petitioner’s invocation of the principle of accession under either Article 370 of the Spanish Civil Code of 1889 or
Article 461 of the Civil Code, the same does not apply to vest her with ownership over subject land.

Under Article 370 51 of the Spanish Civil Code of 1889 which took effect in the Philippines on December 7, 1889, 52 the beds of
rivers which remain abandoned because the course of the water has naturally changed belong to the owners of the riparian
lands throughout their respective lengths. If the abandoned bed divided estates belonging to different owners, the new dividing
line shall run at equal distance therefrom. 53

When the present Civil Code took effect on August 30, 1950, 54 the foregoing rule was abandoned in favor of the present Article
461, which provides:chanrob1es virtual 1aw library

ART. 461. River beds which are abandoned through the natural change in the course of the waters ipso facto belong to the
owners whose lands are occupied by the new course in proportion to the area lost. However, the owners of the lands adjoining
the old bed shall have the right to acquire the same by paying the value thereof, which value shall not exceed the value of the
area occupied by the new bed. (Emphasis supplied)

Article 461 provides for compensation for the loss of the land occupied by the new bed since it is believed more equitable to
compensate the actual losers than to add land to those who have lost nothing. 55 Thus, the abandoned river bed is given to the
owner(s) of the land(s) onto which the river changed its course instead of the riparian owner(s). 56

Petitioner claims that on October 22, 1966, when she purchased the property adjoining the subject land from Marcelina Basadre,
the said subject land was already a dried-up river bed such that "almost one-half portion of the residential house . . . was so
already built and is still now situated at the said dried-up portion of the Salunayan Creek bed . . ." 57 She failed to allege,
however, when the subject portion of the Salunayan Creek dried up, a fact essential to determining whether the applicable law is
Article 370 of the Spanish Civil Code of 1889 or Article 461 of the Civil Code.chanrob1es virtua1 1aw 1ibrary

Had the disputed portion of the Salunayan Creek dried up after the present Civil Code took effect, the subject land would clearly
not belong to petitioner or her predecessor-in-interest since under the aforementioned provision of Article 461, "river beds which
are abandoned through the natural change in the course of the waters ipso facto belong to the owners of the land occupied by
the new course," and the owners of the adjoining lots have the right to acquire them only after paying their value. 58

And both Article 370 of the Old Code and Article 461 of the present Civil Code are applicable only when" [r]iver beds are
abandoned through the natural change in the course of the waters." It is uncontroverted, however, that, as found by both the
Bureau of Lands and the DENR Regional Executive Director, the subject land became dry as a result of the construction of an
irrigation canal by the National Irrigation Administration. Thus, in Ronquillo v. Court of Appeals, 59 this Court held:chanrob1es
virtual 1aw library

The law is clear and unambiguous. It leaves no room for interpretation. Article 370 applies only if there is a natural change in the
course of the waters. The rules on alluvion do not apply to man-made or artificial accretions nor to accretions to lands that adjoin
canals or esteros or artificial drainage systems. Considering our earlier finding that the dried-up portion of Estero Calubcub was
actually caused by the active intervention of man, it follows that Article 370 does not apply to the case at bar and, hence, the Del
Rosarios cannot be entitled thereto supposedly as riparian owners.

The dried-up portion of Estero Calubcub should thus be considered as forming part of the land of the public domain which
cannot be subject to acquisition by private ownership. . . 60 (Emphasis supplied)

Furthermore, both provisions pertain to situations where there has been a change in the course of a river, not where the river
simply dries up. In the instant Petition, it is not even alleged that the Salunayan Creek changed its course. In such a situation,
commentators are of the opinion that the dry river bed remains property of public dominion. 61
CivRev (Property) Assignment 2

Finally, while this Court notes that petitioner offered to purchase the subject land from the government, 62 she did so through an
informal letter dated August 9, 1989 63 instead of the prescribed form. By such move, she is deemed to have acknowledged that
the subject land is public land, for it would be absurd for her to have applied for its purchase if she believed it was hers. She is
thus stopped from claiming otherwise. 64

WHEREFORE, the petition is hereby DENIED for lack of merit.

SO ORDERED

You might also like